You are on page 1of 276

WBUHS (2008-2020)

rd
3 Prof. M.B.B.S, Part - II
Paper – II (SUPPLEMENTARY)

Answers of Surgery Question


Papers for M.B.B.S Students

Dr. Arkaprovo Roy


ASSOCIATE PROFESSOR
DEPARTMENT OF GENERAL SURGERY
MEDICAL COLLEGE, KOLKATA
The West Bengal University of Health Sciences

M.B.B.S. 3rd Professional Part – II Examination,

Paper – II, Supplementary Examination, 2020

Subject: Surgery Time: 21/2 hrs.

Paper: II Marks: 60

Attempt all questions

Group -A

1. Define gangrene. Discuss etiopathogenesis, clinical features, investigations and


management of gas gangrene.
3+3+3+3+3
Group - B
2. Answer any one of the following:
a) Classify testicular tumours. How will you manage a 60 year old man presenting
with seminoma testis. 5+10

b) Describe the clinical features and management of thyrotoxicosis. 8+7

Group - C
3. Write short notes on any three of the following: 3x5
a) Epidural anaesthesia.
b) Imperforate anus.
c) Breast abscess.
d) Basal cell carcinoma.
e) Spina bifida.
Group - D
4. Write short notes on any three of the following: 3x5
a) Bleeding from gum.
b) Hydrocephalus.
c) Glasgow Coma Scale.
d) P. E. T. Scan.
e) Cleft lip.

Group - A

1. Gas gangrene is most commonly caused by infection with the bacterium Clostridium
perfringens, which develops in an injury or surgical wound that's depleted of blood
supply. The bacterial infection produces toxins that release gas — hence the name
"gas" gangrene — and cause tissue death.

Causative organism: This is caused by C. perfringens. These Gram-positive, anaerobic, spore-


bearing bacilli are widely found in nature, particularly in soil and faeces.
Other common clostridial species that cause gas gangrene include Clostridium bifermentans,
Clostridium septicum, Clostridium sporogenes, Clostridium novyi, Clostridium fallax,
Clostridium histolyticum, and Clostridium tertium.

Patients at risk:
 This is found in military and traumatic surgery and colorectal operations.
 Patients who are immunocompromised, diabetic or have malignant disease are at greate
risk, particularly if they have wounds containing necrotic or foreign material, resulting
in anaerobic conditions.

Military wounds provide an ideal environment as the kinetic energy of high-velocity missiles
or shrapnel causes extensive tissue damage. The cavitation which follows passage of a
missile through the tissues causes a ‘sucking’ entry wound, leaving clothing and
environmental soiling in the wound in addition to devascularised tissue.

Bacterium of the Clostridial species produce two toxins: alpha and theta toxins, which cause
necrotizing damage to tissues.
Myonecrosis differs slightly from other types of necrosis. While the underlying causes are
almost identical, the type of affected tissue (in particular, muscle tissue) is significantly
more important for the patient's general health. Superficial necrosis is unsightly and can
lead to unattractive scarring, but otherwise does not affect the patient's likelihood of
survival or physical capability to the same extent. However, massive myonecrosis will likely
result in the loss of movement of the entire region. If the necrotic damage is allowed to
continue throughout an affected limb, then often that entire limb is lost permanently.
It is often difficult to identify the extent of muscle damage, as C. perfringens may be at work
in deeper fascial layers below the skin. Unlike other anaerobic infections, discharge in these
infections is often not purulent (filled with pus). Instead, the discharge is often described as
"sweetly putrid" or "dishwater pus" because it is much thinner than normal pus. This is due
to the lysis of neutrophils, a type of white blood cell, caused by the lecithinases and other
toxins released by Clostridium species.
Soil-borne anaerobes are particularly well-adapted to surviving harsh conditions. Often, a
scarcity of nutrition and competition for resources from numerous other species occurs.
Changes in pH and temperature are often significant, also. Bacteria often possess the ability
to create exotoxins to assist them in competing with other microbes in their natural
environments. When such bacteria are able to enter a living host, they encounter a vast
supply of nutrients, warm conditions, and an abundance of water. This enables the microbes
to rapidly proliferate, far in excess of the immune system's capability to defend,
as prokaryotic bacteria possess a far greater capacity for multiplication than the host's
immune system. The combination of bacterial load and ability to multiply is the basis for the
microbes' ability to cause massive infection. Alongside such rapid proliferation is a
corresponding mass-production of exotoxin that causes severe damage to local tissue in the
host. One such exotoxin is alpha toxin, which is produced by C. perfringens and is the
key virulence factor in its pathogenesis
Massive infection, gross injury, and depletion of the host's immune capability result in
system-wide sepsis. This is partly due to the burden on the immune system, its
corresponding release of inflammatory cytokines, and the distribution of bacterial toxins.
Massive infection is likely to result in death from a combination of system-wide septic
shock and the unintentionally damaging effects of the immune response. In animals,
disability and distress caused by all of these factors markedly increase the chance of
predation.

Clinical features:
 Gas gangrene wound infections are associated with severe local wound pain and crepitus
(gas in the tissues, which may also be noted on plain radiographs).
 The wound produces a thin, brown, sweet-smelling exudate, in which Gram staining will
reveal bacteria.
 Oedema and spreading gangrene due to release of collagenase, hyaluronidase, other
proteases and alpha toxin.
 Early systemic complications with circulatory collapse and multi organ systemic failure
follow if prompt action is not taken.
Diagnosis: Various diagnostic methods can be employed in the diagnosis of Gas gangrene.
Due to low incidence of myonecrosis it is easy to overlook diagnosis. As bacterial infections
mostly exhibit the same symptoms, early diagnosis of gas gangrene rarely occurs. The
ambiguous symptoms only contribute to poorer prognosis. Diagnostic methods include:

 Biopsy of affected tissue


 Cultures of fluids from inflicted area
 Resonance imaging to visualize necrotized subcutaneous tissues
 X-rays for air pockets in affected tissues
 Microscopy identification of strain of bacteria sampled from fluids of inflicted area.
 Gram stain

Treatment:
 Antibiotic prophylaxis should always be considered in patients at risk, especially when
amputations are performed for peripheral vascular disease with open necrotic
ulceration.
 Once gas gangrene infection is established, large doses of intravenous penicillin and
aggressive debridement of affected tissues are required.

Group – B

2. a) The WHO histopathological classification of testicular tumours


Germ cell tumours (90%)
Seminoma (48%)
Spermatocytic, classical, and anaplastic
subtypes
Non-seminomatous GCT (42%)
o Teratoma:
Differentiated/mature
Intermediate/immature
Undifferentiated/malignant
o Yolk sac tumour
o Choriocarcinoma
o Mixed NSGCT
Mixed GCT (10%)
Other tumours (7%)
 Epidermoid cyst (benign)
 Adenomatoid tumour
 Adenocarcinoma of the rete testis
 Carcinoid
 Lymphoma (5%)
 Metastatic, from another site (1%)
Sex cord stromal tumours (3%) (10% malignant)
 Leydig cell
 Sertoli cell
 Mixed or unclassified
Mixed germ cell/sex cord tumours (rare)
Tumorigenic model for germ cell tumours of the testis
Chemotherapy for seminomas:

Stage I seminomas: Chemotherapy: An option that works as well as radiation is 1 or 2 cycles


of chemotherapy with the drug carboplatin after surgery. Many experts prefer chemo over
radiation because it seems to be easier to tolerate. .

Stage IIA seminomas: Chemotherapy: Another option is chemo, with either 4 cycles of EP
(etoposide and cisplatin) or 3 cycles of BEP (bleomycin, etoposide, and cisplatin). You doctor
will watch you closely (every 3 to 6 months) to look for signs that the cancer has come back.

Stage IIB seminomas: Chemotherapy: This is the preferred treatment. You can get either 4
cycles of EP (etoposide and cisplatin) or 3 cycles of BEP (bleomycin, etoposide, and
cisplatin).

Stage IIC seminomas: Chemotherapy with 4 cycles of EP or 3 or 4 cycles of BEP. Radiation


therapy is generally not used for stage IIC seminoma.

b) Thyrotoxicosis is the clinical manifestation of excess thyroid hormone action at the tissue
level due to inappropriately high circulating thyroid hormone concentrations.
Hyperthyroidism, a subset of thyrotoxicosis, refers specifically to excess thyroid hormone
synthesis and secretion by the thyroid gland.

Primary thyrotoxicosis is common; it is due to disease in the thyroid, and thyroid-


stimulating hormone (TSH) levels are high. Secondary thyrotoxicosis is less common; it is
due to pituitary or hypothalamic disease, and TSH levels are low.
Signs and symptoms

Common symptoms of hyperthyroidism and thyrotoxicosis include the following:


 Nervousness
 Anxiety
 Increased perspiration
 Heat intolerance
 Hyperactivity
 Palpitations
Common signs of hyperthyroidism and thyrotoxicosis include the following:
 Tachycardia or atrial arrhythmia
 Systolic hypertension with wide pulse pressure
 Warm, moist, smooth skin
 Lid lag
 Stare
 Hand tremor
 Muscle weakness
 Weight loss despite increased appetite (although a few patients may gain weight, if
excessive intake outstrips weight loss)
 Reduction in menstrual flow or oligomenorrhea
Presentation of thyrotoxicosis varies, as follows :
 Younger patients tend to exhibit symptoms of sympathetic activation (eg, anxiety,
hyperactivity, tremor)
 Older patients have more cardiovascular symptoms (eg, dyspnea, atrial fibrillation)
and unexplained weight loss
 Patients with Graves disease often have more marked symptoms than patients with
thyrotoxicosis from other causes
 Ophthalmopathy (eg, periorbital edema, diplopia, or proptosis) and pretibial
myxedema dermopathy specifically occur with Graves disease
 Elevated thyroid hormone levels associated with subacute thyroiditis may occur as
part of a postviral syndrome (subacute granulomatous thyroiditis) or within a year of
the end of a pregnancy (postpartum subacute thyroiditis)
Diagnosis

Thyroid function tests for hyperthyroidism and thyrotoxicosis are as follows:


 Thyroid-stimulating hormone (TSH)
 Free thyroxine (FT4) or free thyroxine index (FTI—total T4 multiplied by the
correction for thyroid hormone binding)
 Total triiodothyronine (T3)
Thyroid function study results in hyperthyroidism and thyrotoxicosis are as follows:
 Hyperthyroidism and thyrotoxicosis are marked by suppressed TSH levels and
elevated T3 and T4 levels
 Patients with milder hyperthyroidism may have elevation of T3 levels only with
a suppressed TSH level
 Subclinical hyperthyroidism features decreased TSH and normal T3 and T4 levels
Autoantibody tests for hyperthyroidism are as follows:
 Anti–thyroid peroxidase (anti-TPO) antibody - Elevation with autoimmune thyroid
disease found in 85% of Graves patients
 Thyroid-stimulating antibody (TSab) - Also known as thyroid-stimulating
immunoglobulin (TSI), long-acting thyroid stimulator (LATS), or TSH-receptor
antibody (TRab); found in 63-81% of Graves disease; a positive test is diagnostic and
specific for Graves disease
Autoantibody titers in hyperthyroidism and thyrotoxicosis are as follows:
 Graves disease - Significantly elevated anti-TPO, elevated TSI ab
 Toxic multinodular goiter - Low or absent anti-TPO and negative TSI ab
 Toxic adenoma - Low or absent anti-TPO and negative TSI ab
 Patients without active thyroid disease may have mildly positive anti-TPO and TSI ab
 Subacute thyroiditis - Low or absent anti-TPO and negative TSI ab
If the etiology of elevated thyroid hormone levels is not clear after physical examination and
other laboratory tests, it can be confirmed by scintigraphy: the degree and pattern of isotope
uptake indicate the type of thyroid disorder. Findings are as follows:
 Graves disease – Diffuse enlargement of both thyroid lobes, with uniform uptake of
isotope and elevated radioactive iodine uptake
 Toxic multinodular goiter -- Irregular areas of relatively diminished and occasionally
increased uptake; overall radioactive iodine uptake is mildly to moderately increased
 Subacute thyroiditis – Very low radioactive iodine uptake, either with a painful thyroid
(subacute granulomatous thyroiditis) or occurring within a year of pregnancy
(postpartum subacute thyroiditis)

Management:
Treatment of hyperthyroidism and thyrotoxicosis includes symptom relief, while
hyperthyroidism also requires therapy with antithyroid medications, radioactive iodine-131
(131I), or thyroidectomy. Symptomatic treatment is as follows:
 Oral rehydration for dehydrated patients
 Beta-blockers for relief of neurologic and cardiovascular symptoms
 For mild ophthalmopathy, saline eye drops as needed and tight-fitting sunglasses for
outdoors
 For vision-threatening ophthalmopathy, high-dose glucocorticoids, with
consideration of orbital decompression surgery, ocular radiation therapy, or a recently
approved treatment from the US Food and Drug Administration (FDA), teprotumumab-
trbw, a monoclonal antibody that blocks the insulin-like growth factor-1 receptor (IGF-
1R) and ameliorates proptosis by reducing inflammation and preventing muscle and
fat-tissue remodeling in the orbit
Antithyroid drug treatment is as follows:
 Used for long-term control of hyperthyroidism in children, adolescents, and pregnant
women
 In adult men and nonpregnant women, used to control hyperthyroidism before
definitive therapy with radioactive iodine
 Methimazole is more potent and longer-acting than propylthiouracil
 Propylthiouracil is reserved for use in thyroid storm, first trimester of pregnancy, and
methimazole allergy or intolerance
 Antithyroid drug doses are titrated every 4 weeks until thyroid functions normalize
 Patients with Graves disease may experience remission after treatment for 12-18
months, but recurrences are common within the following year
 Toxic multinodular goiter and toxic adenoma will not go into remission
Radioactive iodine treatment is as follows:
 Preferred therapy for hyperthyroidism
 Administered orally as a single dose in capsule or liquid form
 Causes fibrosis and destruction of the thyroid over weeks to many months
 Hypothyroidism is expected
 Pregnancy, breast feeding, and recent lactation are contraindications
 Radioactive iodine should be avoided in children younger than 5 years
 Radioactive iodine is usually not given to patients with severe ophthalmopathy
 Radioactive iodine is usually not given to patients who cannot comply with physician
restrictions for avoidance of radiation exposure to others
Thyroidectomy is reserved for special circumstances, including the following:
 Severe hyperthyroidism in children
 Pregnant women who are noncompliant with or intolerant of antithyroid medication
 Patients with very large goiters or severe ophthalmopathy
 Patients who refuse radioactive iodine therapy
 Refractory amiodarone-induced hyperthyroidism
 Patients who require normalization of thyroid functions quickly, such as pregnant
women, women who desire pregnancy in the next 6 months, or patients with unstable
cardiac conditions.

Group – C

3. a) Introduction: Epidural anesthesia is regional anesthesia that blocks pain in a


particular region of the body. The goal of an epidural is to provide analgesia, or pain
relief, rather than anesthesia which leads to total lack of feeling. Epidurals block the
nerve impulses from the lower spinal segments.

Indications: Injecting medication into the epidural space is primarily performed for
analgesia.

For analgesia alone, where Surgery is not contemplated. An epidural injection or infusion for
pain relief (e.g. in child birth) is less likely to cause loss of muscle power, but has to be
augmented to be sufficient for surgery.

 As an adjunct to general anaesthesia. This may reduce the patient's requirement


for opioid analgesics. This is suitable for a wide variety of surgery, for example
gynaecological surgery (e.g. hysterectomy), orthopaedic surgery (e.g. hip replacement),
general surgery (e.g.laparotomy) and vascular surgery (e.g. open aortic
aneurysm repair).
 As a sole technique for surgical anaesthesia. Some operations, most
frequently Caesarean section, may be performed using an epidural anaesthetic as the
sole technique. This can allow the patient to remain awake during the operation.
 For post-operative analgesia, after an operation where the epidural technique was used
as either the sole anaesthetic, or was used in combination with general anaesthesia.
Analgesics are given into the epidural space typically for a few days after surgery,
provided a catheter has been inserted. Through the use of a patient-controlled epidural
analgesia (PCEA) infusion pump, a person has the ability to give themselves an
occasional dose of pain medication through an epidural catheter.
 Treatment of back pain. Injection of analgesics and steroids into the epidural space may
improve some forms of back pain.
 Treatment of chronic pain or palliation of symptoms in terminal care, usually in the
short- or medium-term.
Epidural analgesia during childbirth:
Advantages Disadvantages

 More use of instruments to assist with


 Better pain relief than other pain the birth
medication  Increased risk of Caesarean section for
 Fewer babies needing naloxone to fetal distress
counter opiate use by the mother  Longer delivery (second stage of labour)
 Decreased  Increased need for oxytocin to
maternal hyperventilation and stimulate uterine contractions
increased oxygen supply to baby  Increased risk of very low blood pressure
 Decreased  Increased risk of muscular weakness for
circulating adrenocorticotropic a period of time after the birth
hormone and decreased fetal distress [  Increased risk of fluid retention
 Increased risk of fever

Epidural analgesia has been demonstrated to have several benefits after surgery, including:

 Effective analgesia without the need for systemic opioids.


 The incidence of postoperative respiratory problems and chest infections is reduced.
 The incidence of postoperative myocardial infarction ("heart attack") is reduced.
 The stress response to surgery is reduced.
 Motility of the intestines is improved by blockade of the sympathetic nervous system.
 Use of epidural analgesia during surgery reduces blood transfusion requirements.

Complications:

 Failure to achieve analgesia or anaesthesia occurs in about 5% of cases, while


another 15% experience only partial analgesia or anaesthesia
 The following factors are associated with failure to achieve epidural
analgesia/anaesthesia
 Obesity
 Multiparity
 History of a previous failure of epidural anaesthesia
 History of regular opiate use
 Cervical dilation of more than 7 cm at insertion
 The use of air to find the epidural space while inserting the epidural instead of
alternatives such as saline or lidocaine
 Accidental dural puncture with headache. This may cause cerebrospinal fluid (CSF) to
leak out into the epidural space, which may in turn cause a post dural puncture
headache (PDPH). Most cases resolve spontaneously with time.
 Delayed onset of breastfeeding and shorter duration of breastfeeding.
 Bloody tap.
 Catheter misplaced into a vein.
 High block, as described above.
 Catheter misplaced into the subarachnoid space. This may result in a high block, or,
more rarely, a total spinal, where anaesthetic is delivered directly to the brainstem,
causing unconsciousness and sometimes seizures.
 Neurological injury lasting less than 1 year.
 Epidural abscess formation.
 Epidural haematoma formation.
 Neurological injury lasting longer than 1 year.
 Paraplegia.
 Arachnoiditis.
 Death (extremely rare, less than 1 in 100,000).

b) Wingspread anatomical classification of Anorectal malformations (1984)


Associated Anomalies:

 Sacrum and Spine


Sacral deformities appear to be the most frequently associated defect.
 Genitourinary Defects
The frequency of associated genitourinary defects varies from 20% to 54%.
 Anal atresia may occur as a part of the VACTERL group of anomalies
V Vertebral body segmentation defect
A Anal atresia
C Cardiovascular (PDA, VSD)
TE Tracheo esophagial fistula
R unilateral Renal agenesis
L Limb anomaly (radial ray hypoplasia)
 CVS: Tetrology of Fallot or VSD
 GI: Tracheo-esophageal abnormality , Duodenal atresia , Hirschsprung’s .

Algorithm for management of ARM in newborn male


Posterior Sagittal Ano-RectoPlasty – PSARP.

PSARP involves:
 Stimulation of muscles to demonstrate the midline and sphincter
 Posterior sagittal incision - length depends on severity of abnormality and required
extent of dissection
 Rectum identified. Abdominal approach may be required in addition in 10% of males
and 40% of cloacae
Rectum dissected.
 Separation from genitourinary tract - often the most difficult part
 Repositioning the neoanus within the sphincteric mechanism

Algorithm for management of ARM in newborn female

c) Breast abscess:

A. Lactational mastitis: Lactational mastitis may occur either sporadically or in epidemics.

 The most common causative organism is Staphylococcus aureus.


 It presents as a swollen, erythematous, and tender breast; purulent discharge from
the nipple is uncommon.
 In the early cellulitic phase, the treatment is antibiotics. The frequency of nursing or
pumping should be increased. Approximately 25% progress to abscess formation.
 Breast abscesses occur in the later stages and are often not fluctuant.
 The diagnosis is made by failure to improve on antibiotics, abscess cavity seen on
ultrasound, or aspiration of pus.
 Treatment is cessation of nursing and surgical drainage.

B. Nonpuerperal abscesses
Nonpuerperal abscesses result from duct ectasia with periductal mastitis, infected cysts,
infected hematoma, or hematogenous spread from another source.

 They usually are located in the peri/retroareolar area.


 Anaerobes are the most common causative agent, although antibiotics should cover
both anaerobic and aerobic organisms.
 Treatment is surgical drainage.
 Unresolved or recurring infection requires biopsy to exclude cancer. These patients
often have a chronic relapsing course with multiple infections requiring surgical
drainage.
 Repeated infections can result in a chronically draining periareolar lesion or a
mammary fistula lined with squamous epithelium. Treatment is excision of the
central duct along with the fistula once the acute infection resolves. The fistula can
recur even after surgery.

d) Basal cell carcinoma.

Epidemiology and aetiology:

 >400,000 new patients annually

 Age usually over age 40

 Sex Males >Females

 Race rare in brown and black skinned patient.

Clinical features:

 Pearly papules or nodules


 Rolled border
 Telangiectases
 Central ulcer
 Crusting
 Bleed easily
 Rodent ulcer
 Locally malignant

Sites: Tear drop cancer:

 Head, neck
 Forehead
 Cheeks
 Nose
 Ears
 Upper trunk

Biopsy:

 Biopsy: Shave, Punch,Excision


 Specimen to reliable dermatopathologist or pathologist

Treatment:

 Electrodesiccation and curettage


 Excision
 Cryosurgery
 Moh’s Surgery
 Radiation
 5-Fluorouracil
 Aldara (Imiquimod)

e) Spina bifida:

Definition: It is a congenital abnormality with developmental defect in the spinal column


with incomplete closure of vertebral canal due to failure in fusion of vertebral arches ±
protrusion and dysplasia of the spinal cord or its membranes.

 It arises in the first few weeks of gestation, with unknown cause

 It arises when one vertebrae does not close normally and leave the spinal cord
exposed

 It is a midline defect of the: Skin, bone spinal column and spinal cord.

 3 in 1,000 births

Types:

Occulta :

 Mildest form of spina bifida

 The outer part of some of the vertebrae are not completely

 The split in the vertebrae is so small that the spinal cord does not protrude

 The condition is asymptomatic

 Bony abnormality seen by X – ray

 Nerves may be involved when associated with hairy patch or other skin changes
Cystica:

1- Meningeocele:

• Vertebral arches are unfused & with herniation of the meninges part of the cord or
nerves root may present in the sac but they conduct impulses normally.

• Meningeocele is covered by the outer layer of skin and the inner layer of meninges
which communicate with meninges lining the spinal subarachnoid space2-
Myelomeningeocele:

• Sac contains neural elements that protrude through the spinal defects

• The overlying skin is thin & leaks of spinal fluid

• Secondary infection is common , Neurological and Orthopaedic problem are present.

• Hydrocephalus(arnold- chiari malformation)

• Clinical Picture: It will differ according to the level of lesion (most common site is
lumbosacral region)

• Flaccid paralysis, muscle weakness, wasting, decreased or absent tendon reflexes

• Decreased or absent extroceptive & proprioceptive sensation

• Rectal & bladder incontinence, Hydrocephalus, Sever vasomotor changes

• Paralytic or congenital deformities as in club foot , pes cavus.

• Pressure ulceration due to poor sensation, Osteoporosis , soft tissue contracture

• Physical emotion& mental delay

• Prognosis:

• With successful closure of simple meningeocele prognosis is good

• Myelomeningeocele - die from infection, if survive after proper closure - stationary


disability

Proposed Aetiologies for Spina Bifida:

Multi-factorial Vitamin Zinc Viral Mineral deficiency.


inheritance. deficiencies/folate. deficiency. infection.
Potato blight. Maternal fever High sound Alcohol. Medication –
intensity. Phenytoin, Epilim, etc.

Screening and diagnosis:


Prenatal Tests Evaluation Imaging Gait Analysis
Studies

AFP Analysis of individual X-rays


medial history
Ultrasound Physical examination Ultrasound
Testing of Evaluation of critical CT scan
Amniotic fluid body systems MRI

Treatment:

 No cure

 Regular check-ups with physician

 Surgery (24 hours after birth)

 Medication

 Physiotherapy

 Bowel , bladder training.

Surgery:

 Usually performed within 24 hours after birth.

 They remove the infected area and replace it with muscle tissue and skin.

 Helps protect against hydrocephalus.

Group – D

4. a) There are many possible causes of gingival bleeding. The main cause of gingival
bleeding is the formation and accumulation of plaque at the gum line due to improper
brushing and flossing of teeth. The hardened form of plaque is called tartar. An advanced
form of gingivitis as a result of formation of plaque is periodontitis.

Other causes that can exacerbate gingival bleeding include:

 Placement of new dentures


 Tooth or gum infection
 Diabetes mellitus
 Idiopathic thrombocytopenic purpura
 Leukemia
 Malnutrition
 Use of aspirin and anticoagulants(blood thinners) such as warfarin and heparin [5]
 Hormonal imbalances during puberty and pregnancy
 Iron overload

Other less common causes are:

 Vitamin C deficiency (scurvy) and vitamin K deficiency


 Dengue fever

Diagnosis:

 An oral examination to rule out the issues such as malnutrition and puberty.
 Additional corresponding diagnosis tests to certain potential disease may be required.
This includes oral glucose tolerance test for diabetes mellitus, blood studies, human
gonadotrophin levels for pregnancy, and X-rays for teeth and jaw bones.
 In order to determine the periodontal health of a patient, the dentist or dental
hygienist records the sulcular depths of the gingiva and observes any bleeding on
probing. This is often accomplished with the use of a periodontal probe. Alternatively,
dental floss may also be used to assess the Gingival bleeding index. It is used as an
initial evaluation on patient's periodontal health especially to measure gingivitis. The
number of bleeding sites is used to calculate the gingival bleeding score.
 Bleeding on probing is a poor positive predictor of periodontal disease, but conversely
lack of bleeding is a very strong negative predictor. The clinical interpretation of this
research is that while BOP presence may not indicate periodontal disease, continued
absence of BOP is a strong predictor (approximately 98%) of continued periodontal
health.

Treatment

 Corresponding treatments for diagnosed diseases should be taken as first priority.


 Dentist or hygienists should be visited once every three months for plaque removal.
 Soft-bristle toothbrush is recommended for brushing your teeth. Hard-bristled
toothbrushes may be softened by leaving under hot running water (very hot) before
brushing every time, followed by gentle brushing.
 Flossing twice a day can prevent the building up of plaques.
 Tobacco should be avoided as tobacco can aggravate the bleeding gums.
 A balanced healthy diet should also be taken into account.
 Physiotherapy programme using over-the-counter toothpaste with triclosan should be
used with home care.
 If there is persistent continuation of inflammation and bleeding, a prescription of
antiplaque rinse would be useful.

b) Introduction: Hydrocephalus, also known as "water on the brain", is a medical condition


in which there is an abnormal accumulation of cerebrospinal fluid (CSF) in the ventricles, or
cavities, of the brain. This may cause increased intracranial pressure inside the skull and
progressive enlargement of the head, convulsion, and mental disability. Hydrocephalus can
also cause death.
Signs and Symptoms:
Symptoms of increased intracranial pressure:
 Headaches, vomiting, nausea, papilledema, sleepiness, or coma.
 Elevated intracranial pressure may result in uncal and/or cerebellar tonsill herniation,
with resulting life threatening brain stem compression.
 The triad (Hakim triad) of gait instability, urinary incontinence and dementia is a
relatively typical manifestation of the distinct entity normal pressure
hydrocephalus (NPH). Focal neurological deficits may also occur, such as abducens
nerve palsy and vertical gaze palsy (Parinaud syndrome due to compression of
the quadrigeminal plate, where the neural centers coordinating the conjugated vertical
eye movement are located).
Normal pressure hydrocephalus: The symptoms depend on the cause of the blockage, the
person's age, and how much brain tissue has been damaged by the swelling.

In infants with hydrocephalus, CSF fluid builds up in the central nervous system, causing the
fontanelle (soft spot) to bulge and the head to be larger than expected.

Early symptoms Symptoms that may occur in older children can include:
may also include:

 Eyes that appear to  Brief, shrill, high-pitched cry  Loss of bladder control
gaze downward  Changes in personality, memory, or (urinary incontinence)
 Irritability the ability to reason or think  Loss of coordination and
 Seizures  Changes in facial appearance and trouble walking
 Separated sutures eye spacing  Muscle spasticity (spasm)
 Sleepiness  Crossed eyes or uncontrolled eye  Slow growth (child 0-5 years)
 Vomiting movements  Slow or restricted movement
 Difficulty feeding  Vomiting.
 Excessive sleepiness
 Headache
 Irritability, poor temper control

Pathology:
Hydrocephalus is usually due to blockage of cerebrospinal fluid (CSF) outflow in
the ventricles or in the subarachnoid space over the brain. In a person without
hydrocephalus, CSF continuously circulates through the brain, its ventricles and the spinal
cord and is continuously drained away into the circulatory system. Alternatively, the
condition may result from an overproduction of the CSF fluid, from a congenital
malformation blocking normal drainage of the fluid, or from complications of head
injuries or infections.
Clsssification: Hydrocephalus can be caused by impaired cerebrospinal fluid (CSF) flow,
reabsorption, or excessive CSF production.

The most common cause of hydrocephalus is CSF flow obstruction, hindering the free
passage of cerebrospinal fluid through the ventricular system and subarachnoid
space (e.g.,stenosis of the cerebral aqueduct or obstruction of the interventricular foramina -
foramina of Monro secondary
to tumors, hemorrhages, infections or congenital malformations).

Hydrocephalus can also be caused by overproduction of cerebrospinal fluid (relative


obstruction) (e.g., papilloma of choroid plexus).
Based on its underlying mechanisms, hydrocephalus can be classified
into communicating and non-communicating (obstructive). Both forms can be
either congenital or acquired.
Communicating:
Communicating hydrocephalus, also known as non-obstructive hydrocephalus, is caused by
impaired cerebrospinal fluid resorption in the absence of any CSF-flow obstruction between
the ventricles and subarachnoid space.
Various neurologic conditions may result in communicating hydrocephalus, including
subarachnoid/intraventricular hemorrhage, meningitis, Chiari malformation, and
congenital absence of arachnoidal granulations (Pacchioni's granulations).
Scarring and fibrosis of thesubarachnoid space following infectious, inflammatory, or
hemorrhagic events can also prevent resorption of CSF, causing diffuse ventricular
dilatation.

 Normal pressure hydrocephalus (NPH) is a particular form of communicating


hydrocephalus, characterized by enlarged cerebral ventricles, with only intermittently
elevated cerebrospinal fluid pressure.
 Hydrocephalus ex vacuo also refers to an enlargement of cerebral ventricles and
subarachnoid spaces, and is usually due to brain atrophy (as it occurs in dementias),
post-traumatic brain injuries and even in some psychiatric disorders, such
as schizophrenia. As opposed to hydrocephalus, this is a compensatory enlargement of
the CSF-spaces in response to brain parenchyma loss - it is not the result of increased CSF
pressure.

Non-communicating: Non-communicating hydrocephalus, or obstructive hydrocephalus, is


caused by a CSF-flow obstruction ultimately preventing CSF from flowing into the
subarachnoid space (either due to external compression or intraventricular mass lesions).

 Foramen of Monro obstruction may lead to dilation of one or, if large enough (e.g.,
in Colloid cyst), both lateral ventricles.
 The aqueduct of Sylvius, normally narrow to begin with, may be obstructed by a number
of genetically or acquired lesions (e.g., atresia, ependymitis, hemorrhage, tumor) and
lead to dilation of both lateral ventricles as well as the third ventricle.
 Fourth ventricle obstruction will lead to dilatation of the aqueduct as well as the lateral
and third ventricles.
 The foramina of Luschka and foramen of Magendie may be obstructed due to congenital
failure of opening (e.g., Dandy-Walker malformation).

Congenital
The cranial bones fuse by the end of the third year of life. For head enlargement to occur,
hydrocephalus must occur before then. The causes are usually genetic but can also be
acquired and usually occur within the first few months of life, which include
 Intraventricular matrix hemorrhages in premature infants,
 Infections,
 Type ii arnold-chiari malformation,
 Aqueduct atresia and stenosis, and
 Dandy-walker malformation.
In newborns and toddlers with hydrocephalus, the head circumference is enlarged rapidly
and soon surpasses the 97th percentile. Since the skull bones have not yet firmly joined
together, bulging, firm anterior and posterior fontanelles may be present even when the
patient is in an upright position.
The infant exhibits fretfulness, poor feeding, and frequent vomiting.
As the hydrocephalus progresses, torpor sets in, and the infant shows lack of interest in his
surroundings. Later on, the upper eyelids become retracted and the eyes are turned
downwards (due to hydrocephalic pressure on the mesencephalic tegmentum and paralysis
of upward gaze). Movements become weak and the arms may become tremulous.
Papilledema is absent but there may be reduction of vision. The head becomes so enlarged
that the child may eventually be bedridden.
About 80-90% of fetuses or newborn infants with spina bifida—often associated
with meningocele or myelomeningocele—develop hydrocephalus.

Acquired: This condition is acquired as a consequence of CNS infections, meningitis, brain


tumors, head trauma, intracranial hemorrhage (subarachnoid or intraparenchymal) and is
usually extremely painful.
Treatment:
Hydrocephalus treatment is surgical. It involves the placement of a ventricular catheter (a
tube made of silastic), into the cerebral ventricles to bypass the flow
obstruction/malfunctioning arachnoidal granulations and drain the excess fluid into other
body cavities, from where it can be resorbed.
Most shunts drain the fluid into the peritoneal cavity (ventriculo-peritoneal shunt), but
alternative sites include the right atrium (ventriculo-atrial shunt), pleural
cavity (ventriculo-pleural shunt), and gallbladder
An alternative treatment for obstructive hydrocephalus in selected patients is
theendoscopic third ventriculostomy (ETV), whereby a surgically created opening in the
floor of the third ventricle allows the CSF to flow directly to the basal cisterns, thereby
shortcutting any obstruction, as in aqueductal stenosis.
c) The Glasgow Coma Scale or GCS is a neurological scale that aims to give a reliable,
objective way of recording the conscious state of a person for initial as well as subsequent
assessment. A patient is assessed against the criteria of the scale, and the resulting points
give a patient score between 3 (indicating deep unconsciousness) and either 14 (original
scale) or 15 (the more widely used modified or revised scale).
GCS was initially used to assess level of consciousness after head injury, and the scale is
now used by first aid, EMS, nurses and doctors as being applicable to all acute medical
and trauma patients.
In hospitals it is also used in monitoring chronic patients in intensive care.

Glasgow coma scale (GCS)


Elements of the scale
 Eye response (E)
 Verbal response (V)
 Motor response (M)

Minimum score, 3; maximum score, 15.

Generally, brain injury is classified as:


 Severe, with GCS < 8–9
 Moderate, GCS 8 or 9–12 (controversial)
 Minor, GCS ≥ 13.
 Generally when a patient is in a decline of their GCS score, the nurse or medical staff
should assess the cranial nerves and determine which of the twelve have been affected.
 Tracheal intubation and severe facial/eye swelling or damage make it impossible to test
the verbal and eye responses. In these circumstances, the score is given as 1 with a
modifier attached e.g. "E1c" where "c" = closed, or "V1t" where t = tube. A composite
might be "GCS 5tc". This would mean, for example, eyes closed because of swelling = 1,
intubated = 1, leaving a motor score of 3 for "abnormal flexion". Often the 1 is left out, so
the scale reads Ec or Vt.
 The GCS has limited applicability to children, especially below the age of 36 months
(where the verbal performance of even a healthy child would be expected to be poor).
Consequently the Pediatric Glasgow Coma Scale, a separate yet closely related scale, was
developed for assessing younger children.
d) Positron emission tomography (PET) is a nuclear medicine imaging technique which
produces a three-dimensional image or picture of functional processes in the body. The system
detects pairs of gamma rays emitted indirectly by a positron-emitting radionuclide (tracer),
which is introduced into the body on a biologically active molecule. Images of tracer
concentration in 3- dimensional or 4-dimensional space (the 4th dimension being time) within
the body are then reconstructed by computer analysis. In modern scanners, this reconstruction
is often accomplished with the aid of a CT X-ray scan performed on the patient during the same
session, in the same machine.

If the biologically active molecule chosen for PET is FDG, an analogue of glucose, the
concentrations of tracer imaged then give tissue metabolic activity, in terms of regional glucose
uptake. Although use of this tracer results in the most common type of PET scan, other tracer
molecules are used in PET to image the tissue concentration of many other types of molecules of
interest.

Combination of PET with CT and MRI:

PET scans are increasingly read alongside CT or magnetic resonance imaging (MRI) scans,
the combination ("co-registration") giving both anatomic and metabolic information (i.e.,
what the structure is, and what it is doing biochemically). Because PET imaging is most
useful in combination with anatomical imaging, such as CT, modern PET scanners are now
available with integrated high-end multi-detector-row CT scanners. Because the two scans
can be performed in immediate sequence during the same session, with the patient not
changing position between the two types of scans, the two sets of images are more-precisely
registered, so that areas of abnormality on the PET imaging can be more perfectly correlated
with anatomy on the CT images. This is very useful in showing detailed views of moving
organs or structures with higher anatomical variation, which is more common outside the
brain.

PET-MRI: Presently, only the head and brain can be imaged at these high magnetic field
strengths.

Radionuclides: Radionuclides used in PET scanning are typically isotopes with short half
lives such as carbon-11 (~20 min), nitrogen-13 (~10 min), oxygen-15 (~2 min), and fluorine-
18 (~110 min).

It is important torecognize that PET technology can be used to trace the biologic pathway of
any compound inliving humans (and many other species as well), provided it can be
radiolabeled with a PETisotope. Thus the specific processes that can be probed with PET are
virtually limitless, andradiotracers for new target molecules and processes are being
synthesized all the time; as of thiswriting there are already dozens in clinical use and
hundreds applied in research. Presently, however, by far the most commonly used nuclide in
clinical PET scanning is fluorine-18 in theform of FDG. The half life of fluorine-18 is long
enough such that fluorine-18 labeled radiotracers can be manufactured commercially at an
offsite location.

Limitations:

o The minimization of radiation dose to the subject is an attractive feature of the use of
short-livedradionuclides. Besides its established role as a diagnostic technique, PET has
an expanding role as a method to assess the response to therapy, in particular, cancer
therapy, where the risk tothe patient from lack of knowledge about disease progress is
much greater than the risk from thetest radiation.
o Limitations to the widespread use of PET arise from the high costs of cyclotrons needed
toproduce the short-lived radionuclides for PET scanning and the need for specially
adapted on-sitechemical synthesis apparatus to produce the radiopharmaceuticals. This
limitationrestricts clinical PET primarily to the use of tracers labelled with fluorine-18,
which has a half lifeof 110 minutes and can be transported a reasonable distance before
use, or to rubidium-82,which can be created in a portable generator and is used for
myocardial perfusion studies.
o The presence of the small on-site cyclotron promises to expand in the future as the
cyclotrons shrink in response to the high cost of isotope transportation to remote PET
machines Because the half-life of fluorine-18 is about two hours, the prepared dose of
aradiopharmaceutical bearing this radionuclide will undergo multiple half-lives of decay
during theworking day. This necessitates frequent recalibration of the remaining dose
(determination of activity per unit volume) and careful planning with respect to patient
scheduling.

PET is a valuable technique for some diseases and disorders, because it is possible to target
the radio-chemicals used for particular bodily functions.

Oncology:

 PET scanning with the tracer fluorine-18 (F-18) fluorodeoxyglucose (FDG), called FDG-
PET, is widely used in clinical oncology. This tracer is a glucose analog that is taken up by
glucose-using cells and phosphorylated by hexokinase (whose mitochondrial form is
greatly elevated in rapidly growing malignant tumours).
 A typical dose of FDG used in an oncological scan is 200- 400 mBq for an adult human.
Because the oxygen atom which is replaced by F-18 to generate FDG is required for the
next step in glucose metabolism in all cells, no further reactions occur in FDG.
 FDG-PET can be used for diagnosis, staging, and monitoring treatment of cancers,
particularly in Hodgkin's lymphoma, non-Hodgkin lymphoma, and lung cancer.
 Many other types of solid tumors will be found to be very highly labeled on a case-by case
basis—a fact which becomes especially useful in searching for tumor metastasis, or for
recurrence after a known highly active primary tumor is removed.
 Because individual PET scans are more expensive than "conventional" imaging with
computed tomography (CT) and magnetic resonance imaging (MRI), expansion of FDG-
PET in cost-constrained health services will depend on proper health technology
assessment; this problem is a difficult one because structural and functional imaging
often cannot be directly compared, as they provide different information. Oncology scans
using FDG make up over 90% of all PET scans in current practice.
Neurology:

 PET neuroimaging is based on an assumption that areas of high radioactivityare


associated with brain activity. Early diagnosis of Alzheimer's disease can be done.
 Cardiology, atherosclerosis and vascular disease study: In clinical cardiology, FDG-PET
can identify so-called "hibernating myocardium", but its cost-effectiveness in this role
versus SPECT is unclear.
 Neuropsychology / Cognitive neuroscience: To examine links between specific
psychological processes or disorders and brain activity.

Psychiatry:

 Numerous compounds that bind selectively to neuroreceptors of interest in biological


psychiatry have been radiolabeled with C-11 or F-18. Radioligands that bind todopamine
receptors (D1,D2, reuptake transporter), serotonin receptors (5HT1A, 5HT2A, reuptake
transporter) opioid receptors (mu) and other sites have been used successfully in
studies with human subjects.
 Studies have been performed examining the state of these receptors in patients
compared to healthy controls in schizophrenia, substance abuse,mood disorders and
other psychiatric conditions.
Pharmacology: In pre-clinical trials, it is possible to radiolabel a new drug and inject it into
animals.

e) Cleft lip: Clefts of the lip (CL) and alveolus with or without cleft of palate (CL/P) are the
most common congenital craniofacial defects and the fourth most frequent birth defects in
the world.

Clefts of the lip may be


 Unilateral or bilateral,
 Complete or incomplete,
 Right or left sided and
 May occur with or without cleft palate.

They frequently occur as isolated anomalies but may be associated with syndromes such as
 Treacher Collins syndrome,  Goldenhar syndrome,
 Down syndrome,  Pierre Robin sequence,
 Apert syndrome,  Van der Woude syndrome

Risk factors:

 Family history: Parents with a family history of cleft lip or cleft palate face a higher risk of
having a baby with a cleft.
 Race: Common in Native Americans and least common in African-Americans.
 Sex: Males are twice as likely to have a cleft lip with or without cleft palate. Cleft palate
without cleft lip is more common in females.
 Exposure to certain substances during pregnancy: Cleft lip and cleft palate may be more
likely to occur in pregnant women who smoke cigarettes, drink alcohol or take certain
medications.
 Having diabetes: Women diagnosed with diabetes before pregnancy may have an
increased risk of having a baby with a cleft lip with or without a cleft palate.
 Being obese during pregnancy. There is some evidence that babies born to obese women
may have increased risk of cleft lip and palate.

Classification:

Group I: Cleft lip only (Pre-alveolar clefts)


Group Ia: Cleft of lip and alveolus
Group II: Cleft palate only (Post alveolar clefts)
Group III: Cleft of lip, alveolus and palate (Alveolar clefts)
Group I and III clefts are subdivided into unilateral / bilateral / median, and Group II is
subdivided into clefts of hard and soft palates.

Complications:

 Difficulty with feedings


 Difficulty swallowing, with potential for liquids or foods to come out the nose
 Nasal speaking voice

Diagnosis:

Traditionally, the diagnosis is made at the time of birth by physical examination. Recent
advances in prenatal diagnosis have allowed obstetricians to diagnose facial clefts in utero.

Treatment:

 Within the first 2–3 months after birth, surgery is performed to close the cleft lip.
 While surgery to repair a cleft lip can be performed soon after birth, often the
preferred age is at approximately 10 weeks of age, following the "rule of 10s" coined
by surgeons
 If the cleft is bilateral and extensive, two surgeries may be required to close the cleft,
one side first, and the second side a few weeks later. The most common procedure to
repair a cleft lip is the Millard procedure.
The West Bengal University of Health Sciences

M.B.B.S. 3rd Professional Part – II Examination,

Paper – II, Supplementary Examination, 2019

Subject: Surgery Time: 21/2 hrs.

Paper: II Marks: 60

Group –A

1. What are the different forms of renal calculus? How does a renal calculus present?
How would you investigate to confirm diagnosis? 5+5+5

Group – B
2. a) What are the principal symptoms of peripheral arterial occlusive disease? How
would you proceed to investigate such a case? What are the conservative
management you advise in for a lower leg distal smaller vessel disease? 5+5+5
Or
b) A 45 year old lady presents with rapidly developing lump in the upper outer
quadrant of the right breast of the size 4cm x 5cm with a palpable mobile enlarged
central group lymph node in the same axilla. How would you confirm your diagnosis?
How would you stage and prognosticate? How would you manage? 5+5+5

Group – C
3. Answer in brief on any three of the following: 3x5
a) Endoscopic Retrograde Cholangio - pancreatography (ERCP)
b) Spinal Anaesthesia
c) Full thickness skin graft
d) Hypertrophic pyloric stenosis of infancy
e) Neo-adjuvant chemotherapy
Group – D
4. Write short notes (any three): 3x5
a) Dental cyst.
b) Post traumatic pneumothorax
c) Intercostals chest tube drainage
d) Extradural haemorrhage
e) Venous ulcer

1. Types of renal stone:

Radio-opaque:
 Calcium phosphate stones are the most radiodense stones, being almost as dense as
bone. A phosphate calculus (calcium phosphate often with ammonium magnesium
phosphate (struvite)) is smooth and dirty white. Magnesium ammonium phosphate
(struvite) stones are less radiodense than calcium containing stones. It grows in alkaline
urine, especially when urea-splitting Proteus organisms are present.The calculus may
enlarge to fill mostof the collecting system, forming a stag-horn calculus.
 Calcium oxalate stones are slightly less radiodense. Oxalate stones are irregular with
sharp projections. A calcium oxalate monohydrate stone is hard and radiodense.

Relatively radiolucent:
 Cystine stones are relatively radiodense because they contain sulphur.
An uncommon congenital error of metabolism leads to cystinuria.
Cystine stones are often multiple and may grow to form a cast of the collecting system.
They are resistant to ESWL.

Completely radiolucent:

 Uric acid: These are hard, smooth and often multiple and multifaceted. Pure uric acid
stones are radiolucent.
 Triamterene:
 Xanthine: Occurs in patients with xanthine oxidase deficiency.
 Indinavir: found in AIDS patients.

Oxalate stone.

 Oxalate is a normal waste product of metabolism and is relatively insoluble.


Normally, approximately 10–15% of oxalate found in the urine originates from the
diet; the vast majority is a metabolic by-product.
 Most of the oxalate that enters the large bowel is consumed by bacterial
decomposition.
 Diet, however, can have an impact on the amount of oxalate found in the urine. Once
absorbed from the small bowel, oxalate is not metabolized and is excreted almost
exclusively by the proximal tubule.
 The presence of calcium within the bowel lumen is an important factor influencing
the amount of oxalate that is absorbed. The control of oxalate in the urine plays a
pivotal role in the formation of calcium oxalate calculi.
 Small changes in oxalate levels in the urine can have a dramatic impact on the
supersaturation of calcium oxalate.
 The principal precursors of oxalate are glycine and ascorbic acid; however, the
impact of ingested vitamin C (<2 g/day) is negligible.
 Hyperoxaluria may develop in patients with bowel disorders, particularly
inflammatory bowel disease, small bowel resection, and bowel bypass. Renal calculi
develop in 5–10% of patients with these conditions.
 Chronic diarrhea with fatty stools results in a saponification process. Intraluminal
calcium binds to the fat, thereby becoming unavailable to bind to oxalate. The
unbound oxalate is readily absorbed.
 Excessive oxalate may occur secondary to the accidental or deliberate ingestion of
ethylene glycol (partial oxidation to oxalate).
 This may result in diffuse and massive deposition of calcium oxalate crystals and may
occasionally lead to renal failure.
 Formed in acidic urine.
 Calcium oxalate crystals in urine appear as 'envelopes' microscopically. They may
also form 'dumbbells.
Clinical features:
 Silent calculus: Renal failure may be the first indication of bilateral silent calculi,
although secondary infection usually produces symptoms first.
 Pain: Pain occurs in 75 per cent of people with urinary stones. Fixed renal pain occurs in
the renal angle,the hypochondrium, or in both. It may be worse on movement.
 Pain resulting from renal stones rarely lasts more than 8 hours in the absence of
infection.
 There is no pyrexia, although the pulse rate rises because of the severe pain.
 Haematuria (microscopic or occasionally macroscopic).
 Struvite staghorn calculi classically present with recurrent UTIs.
 Malaise, weakness, and loss of appetite can also occur.
 Less commonly, struvite stones present with infective complications (pyonephrosis,
perinephric abscess, septicaemia, xanthogranulomatous pyelonephritis).

Diagnostic tests:

 Plain abdominal radiography: calculi that contain calcium are radiodense. Sulphur-
containing stones (cystine) are relatively radiolucent on plain radiography.
 Completely radiolucent stones (e.g. uric acid, triamterene, indinavir) are usually
suspected on the basis of the patient's history and/or urine pH (pH <6 gout; drug history-
triamterene, indinavir), and the diagnosis may be confirmed by ultrasound, CTU, or MRU.
 Renal ultrasound: its sensitivity for detecting renal calculi is ~95%. A combination of
plain abdominal radiography and renal ultrasonography is a useful screeing test for
renal calculi.
 IVU: increasingly being replaced by CTU. Useful for patients with suspected indinavir
stones (which are not visible on CT).
 CTU: a very accurate method of diagnosing all but indinavir stones. Allows accurate
determination of stone size and location and good definition of pelvicalyceal anatomy.
 MRU: cannot visualize stones, but is able to demonstrate the presence of hydronephrosis.

2. a) Answers

 Acute limb ischaemia


Causes
Acute thrombosis in a vessel with pre-existing atherosclerosis (60%)

 Features suggestive of thrombosis are:


o previous history of intermittent claudication;
o slow onset or incomplete occlusion;
o no obvious source of emboli (see below);
o reduced or absent pulses in the contralateral limb.

Emboli (30%)

 Features suggestive of embolism are:


o no previous history of claudication;
o Rapid onset and complete occlusion;
o Cardiac dysrhythmias (especially AF).

Symptoms and signs (any cause)


Six Ps: pain, pallor, pulselessness, paraesthesia, perishingly cold.

 Chronic upper limb ischaemia


Clinical features

 Weakness, cramp, or exercise-related pain and digital ischaemia/gangrene.


 Examine bilateral upper limb pulses, BP in both arms (elevated/at sides), wrist
Doppler pressures.
 Roos test. Arm abducted to 90°, hands up with elbows braced backward, chin
elevated, hands serially clenched/opened for 1-2min. Test positive if pain or
weakness in hand or forearm.
 Adson's test. Pulse diminishes or absent on elevation/abduction of arm with head
turned to contralateral side. Reliability improved by using in conjunction with
arterial duplex.
 Allen's test. Assesses integrity of the palmar arch and dominant vessel (radial or
ulnar).
 Tinel's test. To exclude carpal tunnel syndrome.

 Critical limb ischaemia


May be defined as ischaemia that is likely to progress to limb loss or progressive tissue loss
if it remains untreated.
Clinical features

 European consensus statement defines critical ischaemia if there is:


o rest pain for > 2 weeks not relieved by simple analgesia; or
o Doppler ankle pressure < 50mmHg (toe pressures < 30mmHg if diabetic); or
o tissue necrosis, i.e. the presence of gangrene or ulceration.
 Rest pain typically worsens at night and during elevation of the limb and is relieved
by hanging the limb dependent.
 Arterial ulceration is typically painful, shallow, non-bleeding with few signs of
healing.

Investigations:

General:

 Blood :
o Routine examination of blood including a hemoglobin percent (low Hb% can
decrease claudication distances and aggravate rest pain),
o Blood sugar examination as diabetics have worse prognosis, are essential.
o Erythrocyte sedimentation rate (ESR) is usually raised in Buerger‘s disease.
o In patients with high suspicion of underlying connective tissue disorders, specific test
like RA factor, LE cell phenomenon etc. May be carried out.
o Lipid profile is mandatory in elderly patients with atherosclerosis.
 Urine examination for sugar.
 Plain X-ray of the abdomen will show the presence of arterial calcification and flecks of
calcium may outline an aneurysm.

 ECG: an abnormality in ECG may influence the decision for surgery, in patients with lower
limb disease.

Tests of global Vascular Status:

 Hand Held Doppler ultrasound: blood flow detection uses a continuous wave ultrasound
Signal, beamed at an artery and the reflected beam is picked up by a receiver. The changes of
Frequency in the reflected beam, as compared with the transmitted beam, are due to the
―Doppler shift, resulting from passage of beam through moving blood.These frequency
Changes are converted to audio signals. This investigation may be used effectively in cases
where a differential diagnosis of atherosclerosis is entertained showing the site of block and
Extent of distal run-off.
 Ankle Brachial systolic blood pressure index (ABPI)

This measurement gives the quantitative assessment of the global limb arterial perfusion.
Ankle Brachial Pressure Index

 Segmental pressures: i.e. Differences in arterial blood pressure between segments of


limb can be detected to give indication of the sites of stenosis, specially as Buerger‘s is
said to be a segmental disease.
 Toe Pressures Using Photoplethysmograph: These are used when the arterial disease is
suspected between the ankle and the toes.
 Pole Test: This is used to determine the adequacy of lower limb bolld flow in patients
with incompressible vessels or who are unable to tolerate an ankle pressure cuff.
 Transcutaneous Oximtery (tcpo2): It is based on the principle that the partial pressure of
the oxygen which diffuses through to the surface of the skin reflects the oxygen tension of
the underlying tissues. It is time consuming and is best used in the selection of
amputation sites since it correlates well to subsequent stump healing.
 Walk Test: The basis of this test is that measurement of ABPI before and after a patient
has walked can expose less severe or compensated peripheral vascular disease.

.
Tests for Disease Localisation:

 Duplex imaging: gives accurate information on the size of artery, the flow rate,
turbulence
And the presence of stenosis. The combination of Doppler and color mapping allows easy
Recognition of stenotic sites. This has been achieved by the use of pulsed or continuous wave
Doppler and the two- dimensional images produced by the B- scan made either singly or in
Combination.
 Intravascular Ultrasound: Gives details of arterial walls, luminal contents and
dimensions. This is not a routine investigation for peripheral arterial disease and as yet
is not cost effective.
 Arteriography: This is an invasive technique which though has become much safer in the
Recent years due to fine 3-4 F catheters, and remains the gold standard to provide a road
map
Required for vascualr surgeons expecially before surgery is planned.
 CT Angiography: The introduction of the helical (spiral) CT scanning and multidetector
CT which uses 2 or 4 helicals to scan the patient, CT imaging has been revolutionized for
vascular imaging wherein a single breathhold time is sufficient to generate the scans
from the aortic arch to the groins with imaging quality as good as conventional
angiography.
 MR Angiography: MRI and Phase Contrast MRI were used to visualize moving blood as a
white image but the definition and clarity of the vessels was found to be inferior to
angiograms. More recently, Gadolinium Enhanced MRI (Gd-MRI) has significantly
improved this quality of image and made it comparable to conventional angiography.

Up to two- thirds of patients first presenting in vascular clinic with intermittent claudication,
can be treated by conservative methods.
 Abstinence from tobacco:
 The only proven treatment guideline to prevent disease progression andnavoiding an
amputation is complete cessation of smoking or other forms of tobacco. Smoking
should be stopped early in disease.
 Repeated education and counseling is required for these patients. Raynaud‘s
phenomenon or claudication may continue even after complete discontinuation of
tobacco.
 Explanation and advice: Many patients are worried by the presence of pain while
walking. Once told about the nature of disease and advice regarding methods to
improve their claudication distance,e.g. by walking slowly or by improving underlying
systemic disorder like, anemia, congestive failure, the claudication distance can be
increased.
 Adjustment of lifestyle: Adjustments to everyday habits of transport can increase
mobility within the claudication distance, e.g use of a bicycle etc.
 Exercise & Diet: Taking regular exercise within limits of pain and control of weight in
case of obesity.
 Care of feet, avoiding socks with holes and amateur chiropody, which can spark off
gangrene in the toes and heels, particularly in diabetic patients.
 Heel raise: claudication distance may be improved by raising the heels of shoes by 1
cm. The work of the calf muscles is reduced thereby.
 Analgesics and position: rest pain can be relieved to some extent in some patients by
use of analgesics, elevation of the head end of the bed (Buerger‘s position) and
Buerger‘s exercises (repeated 2 minute elevation and dependency of limb).

Drugs:
o Despite the clear presence of inflammation in this disorder, anti-inflammatory agents
such as steroids have not been shown to be beneficial.
o Similarly, strategies of anticoagulation (thinning of the blood with aspirin or other
agents to prevent clots) have not proven effective.
o Vasodilator drugs are usually started in these patients but their role is equivocal.
o Some of the drugs used are :
 Prostaglandins : Prostacylin or PGI2 (Iloprost) has forty times antiplatelet and
vasodilator activity as compared to PGE1. They are effective in both cutaneous and
muscular vessels. Intravenous infusion of prostacyclin (Iloprost) has been
demonstrated in some studies to relieve rest pain forup to a month and in some upto
6 months.
 Low molecular weight dextrans: dextrans of molecular weight 50000 are used during
acute attack of thromboangitis. They cause hemodilution, decrease viscosity of blood
and improve micro circulation. Intra-arterial infusion is said to be more effective
than intravenous.
 Intra-arterial Thrombolytic therapy: Selective low dose intrarterial streptokinase
(Bolus 10,000 Units followed by 5,000 units per hour) have been used in a very small
group of patients with alteration of level of amputation or it‘s avoidance in 58%
patientsxxii.
 Praxiline : (niftidrofuryl oxalate) may alter tissue metabolism, increasing the
claudication distance by allowing a greater oxygen debt to be incurred. No proven
benefit.
 Trental : (oxypentifylline) has some effect on whole blood viscosity by reducing
rouleaux formation.
 Aspirin in dispersible form may be prescribed for its anti-adhesive effect on platelets.

2. b) Answer.

Confirmation of diagnosis:

Diagnosis

Tissue Diagnosis:
 Establishing a tissue diagnosis is the initial priority on presentation of LABC. In many
patients, core biopsy of the tumor is diagnostic.
 Core needle is preferred over fine needle aspiration, as cytology is insufficient to
confirm lymphovascular invasion.
 Additionally, multiple cores should be extracted both to confirm invasive cancer and to
evaluate hormone receptor status and HER2/neu expression.

Bilateral mammography:
 Prompt bilateral mammography in this setting is essential (except in known
contraindications of BCT e.g. inflammatory breast cancer, ulcerative lesions).
 Diffuse, suspicious microcalcifications or multiple lesions in different quadrants indicate
multicentric disease, and are a contraindication to breast conservation therapy (BCT).

Breast and axillary ultrasound:


 Breast and axillary ultrasound frequently yield valuable information regarding the
extent of disease.
 In particular, axillary ultrasound can be used for image-guided FNA ultrasound detection
nodal metastases has been shown to provide important prognostic information.

TNM classification of breast cancer:

Primary tumor (T)

TX Primary tumor cannot be assessed

T0 No evidence of primary tumor

Tis Carcinoma in situ

Tis
Ductal carcinoma in situ
(DCIS)

Paget disease of the nipple NOT associated with invasive carcinoma and/or carcinoma in
Tis situ (DCIS) in the underlying breast parenchyma. Carcinomas in the breast parenchyma
(Paget) associated with Paget disease are categorized on the basis of the size and characteristics of
the parenchymal disease, although the presence of Paget disease should still be noted

T1 Tumor ≤ 20 mm in greatest dimension

T1mi Tumor ≤ 1 mm in greatest dimension

Tumor > 1 mm but ≤ 5 mm in greatest dimension (round any measurement >1.0-1.9 mm to 2


T1a
mm)

T1b Tumor > 5 mm but ≤ 10 mm in greatest dimension

T1c Tumor > 10 mm but ≤ 20 mm in greatest dimension

T2 Tumor > 20 mm but ≤ 50 mm in greatest dimension

T3 Tumor > 50 mm in greatest dimension

Tumor of any size with direct extension to the chest wall and/or to the skin (ulceration or
T4
skin nodules), not including invasion of dermis alone
T4a Extension to chest wall, not including only pectoralis muscle adherence/invasion

Ulceration and/or ipsilateral satellite nodules and/or edema (including peau d’orange) of
T4b
the skin, which do not meet the criteria for inflammatory carcinoma

T4c Both T4a and T4b

T4d Inflammatory carcinoma

Regional lymph nodes (N)

Clinical

NX Regional lymph nodes cannot be assessed (eg, previously removed)

N0 No regional lymph node metastasis (on imaging or clinical examination)

N1 Metastasis to movable ipsilateral level I, II axillary lymph node(s)

Micrometastases (approximately 200 cells, larger than 0.2 mm, but none larger than 2.0
N1mi
mm)

Metastases in ipsilateral level I, II axillary lymph nodes that are clinically fixed or
N2 matted; or in ipsilateral internal mammary nodes in the absence of clinically evident axillary
lymph node metastases

Metastases in ipsilateral level I, II axillary lymph nodes fixed to one another (matted) or to
N2a
other structures

Metastases only in ipsilateral internal mammary nodes and in the absence of axillary lymph
N2b
node metastases

Metastases in ipsilateral infraclavicular (level III axillary) lymph node(s), with or without
level I, II axillary node involvement, or in ipsilateral internal mammary lymph node(s)
N3
with level I, II axillary lymph node metastasis; or metastases in ipsilateral supraclavicular
lymph node(s), with or without axillary or internal mammary lymph node involvement

N3a Metastasis in ipsilateral infraclavicular lymph node(s)


N3b Metastasis in ipsilateral internal mammary lymph node(s) and axillary lymph node(s)

N3c Metastasis in ipsilateral supraclavicular lymph node(s)

Distant metastasis (M)

M0 No clinical or radiographic evidence of distant metastasis

No clinical or radiographic evidence of distant metastases in the presence of tumor cells or


deposits no larger than 0.2 mm detected microscopically or by molecular techniques in
M0(i+)
circulating blood, bone marrow, or other nonregional nodal tissue in a patient without
symptoms or signs of metastases.

M1 Distant metastases detected by clinical and radiographic means

The following are prognostic and predictive factors for breast cancer when it is first found
and diagnosed.

o Stage: The stage is the main prognostic factor for breast cancer. There is less risk that
early stage breast cancer will come back (recur) so it has a more favourable prognosis.
o If cancer has spread to lymph nodes: Whether or not cancer has spread to lymph nodes is
the most important prognostic factor for breast cancer. Breast cancer that has spread to
lymph nodes has a higher risk of recurrence.

o The number of lymph nodes that contain cancer (called positive lymph nodes) is also
important. The more positive lymph nodes there are, the higher the risk that breast
cancer will come back. Breast cancer that has spread to 4 or more lymph nodes has the
highest risk for recurrence.

o The size of the tumour: The size of the tumour is the 2nd most important prognostic
factor for breast cancer. The tumour size will affect prognosis no matter how many
lymph nodes have cancer in them. Breast tumours that are 5 cm or larger are more likely
to recur.
o Tumour grade: The grade of the breast cancer can affect prognosis. Low-grade tumours
have a better prognosis because they grow slower and are less likely to spread than high-
grade tumours.

o Hormone receptor status: Hormone receptor–positive tumours usually have a good


prognosis. They often are less aggressive, are lower grade and have a lower risk of
spreading than hormone receptor–negative tumours. They usually respond well to
hormonal therapy.

o HER2 status: HER2-positive breast cancer is more aggressive and more likely to spread
than HER2-negative breast cancer.

o Age at diagnosis: Women younger than 35 years of age tend to be diagnosed with more
aggressive, higher grade tumours.

Anatomic stage/prognostic groups:


Staging: T1N1M0 – Early breast cancer

Treatment:

Surgical options:

o Mastectomy with or without reconstruction.


 Modified radical mastectomy (MRM) involves total (simple) mastectomy and axillary
lymph node dissection.
 Follow-up after mastectomy: physical examination every 3 to 6 months for 3 years,
then every 6 to 12 months for the next 2 years, and then annually. Mammography of
the contralateral breast should continue yearly. Regular gynecologic follow-up is
recommended for all women (tamoxifen increases risk of endometrial cancer).
o Breast conservation therapy (BCT): partial mastectomy and ALND(axillary lymph
node dissection) followed by breast irradiation.
o ALND involves the following:
 Removal of level I and level II nodes and, if grossly involved, possibly level III
nodes. Motor and sensory nerves are preserved unless there is direct tumor
involvement.
 An ALND should remove 10 or more nodes. The number of nodes identified is often
pathologist dependent.
 Patients with 4 or more positive lymph nodes should undergo adjuvant radiation
to the axilla. Selective patients with 1 to 3 positive nodes may also benefit from
radiation therapy to the axilla.
 Intraoperative complications: potential injury to the axillary vessels and
neuropathy secondary to injury to the motor nerves of the axilla (the long thoracic,
thoracodorsal, and medial pectoral nerves).

Adjuvant chemotherapy after completion of surgery.

o All node-positive patients should receive adjuvant chemotherapy.


 Regimens are guided by the tumor biomarkers. Typical regimens comprise four to
eight cycles of a combination of cyclophosphamide and an anthracycline, followed
by a taxane administered every 2 to 3 weeks.
 Patients with ER-positive tumors receive adjuvant hormonal therapy for 5 years.
Tamoxifen is given to premenopausal women, and aromatase inhibitors are given
to postmenopausal women (aromatase inhibitors are not used in premenopausal
women).
 (In postmenopausal women older than 70 years, chemotherapy is performed less
frequently. In postmenopausal women with tumors with ER or PR positivity,
tamoxifen or an aromatase inhibitor is frequently the sole adjuvant medical
therapy.)
 In patients with Her2/neu-positive tumors, polychemotherapy is combined with
biological therapy targeting the Her2/neu protein: Trastuzumab is a recombinant
monoclonal antibody that binds to Her2/neu receptor to prevent cell proliferation.
Adding trastuzumab to a chemotherapy regiment of doxorubicin,
cyclophosphamide, and paclitaxel was associated with an increase in the disease-
free survival by 12% and a 33% reduction in the risk of death at 3 year.

3. a) ERCP:

Answer.

 ERCP is a diagnostic procedure designed to examine diseases of the liver, bile ducts
and pancreas.
 ERCP is performed under intravenous sedation, usually without general anesthesia.
 ERCP is an uncomfortable but not painful procedure. There is a low incidence of
complications.
 ERCP can provide important information that cannot be obtained by other diagnostic
examinations, for example, abdominal ultrasound, CT scan, or MRI.
 Frequently, therapeutic measures can be performed at the time of ERCP to remove
stones in the bile ducts or to relieve obstruction of the bile ducts.

Endoscopic retrograde cholangio-pancreatography (ERCP) is a diagnostic test to examine:


 the duodenum (the first portion of the small intestine),
 the papilla of Vater (a small nipple-like structure with openings leading to the bile
ducts and the pancreatic duct),
 the bile ducts, and
 the gallbladder and the pancreatic duct.

The procedure is performed by using a long, flexible, viewing instrument (a duodenoscope)


about the diameter of a pen. The duodenoscope can be directed and moved around the many
bends of the stomach and duodenum. The modern duodenoscope uses a thin fiber-optic
bundle to transmit light to the tip of the endoscope, and a thin wire with a chip also at the tip
of the endoscope to transmit digital video images to a TV screen. The duodenoscope is
inserted through the mouth, through the back of the throat, down the food pipe (esophagus),
through the stomach and into the duodenum. Once the papilla of Vater is identified, a small
plastic catheter (cannula) is passed through an open channel of the endoscope into the
opening of the papilla, and into the bile ducts and/or the pancreatic duct. Contrast material
(dye) is then injected and X-rays are taken of the bile ducts and the pancreatic duct. Another
open channel in the endoscope also allows other instruments to be passed through it in
order to perform biopsies, to insert plastic or metal stents or tubing to relieve obstruction of
the bile ducts or pancreatic duct caused by cancer or scarring, and to perform incisions by
using electrocautery (electric heat).

The liver is a large solid organ located beneath the right diaphragm. The liver produces bile,
which is stored in the gallbladder (a small sac located beneath the liver). After meals, the
gallbladder contracts and empties the bile through the cystic duct, into the bile ducts,
through the papilla of Vater, and into the intestine to help with digestion. The pancreas is
located behind the stomach. It produces a digestive juice that drains through the pancreatic
duct--which usually joins the bile duct within the papilla,--and then enters the intestine.

An important procedure related to ERCP is endoscopic ultrasonography which uses a similar


endoscope that, in addition to the camera, has an ultrasound probe on its tip to examine the
bile ducts, gallbladder, pancreatic duct, and pancreas ultrasonographically.
Ultrasonographically-directed needle biopsies of the pancreas can be taken through a
channel in the endoscope.

A second, newer procedure related to ERCP is the use of miniature endoscopes that are
passed through the operating channel of a duodenoscope and can be inserted directly into
the bile and pancreatic ducts. The inside of the ducts can be visualized, and directed biopsies
can be taken. Other therapeutic interventions also are possible.

The liver, bile ducts, gallbladder, pancreas and the papilla of Vater can be involved in
numerous diseases, causing myriad of symptoms. ERCP is used in diagnosing and treating
the following conditions:

 Blockage of the bile duct by gallstones, cancer, strictures (scarring) or compression


from adjacent organs or tumors
 Jaundice (yellow coloring of the skin) due to obstruction of the bile duct, also causing
darkening of the urine and light colored stool.
 Persistent or recurrent upper abdominal pain which cannot be diagnosed by other
tests
 Unexplained loss of appetite and weight
 Confirming the diagnosis of cancer of the pancreas or the bile duct, so that surgery or
other treatment can be tailored to the disease
 When there is suspicion that the Sphincter of Oddi within the Papilla of Vater, that
controls the flow of bile and pancreatic juice, is not working normally (Sphincter of
Oddi dysfunction)

ERCP is a highly specialized procedure which requires a lot of experience and skill. The
procedure is quite safe and is associated with a very low risk when it is performed by
experienced physicians. The success rate in performing this procedure varies from 70% to
95% depending on the experience of the physician. Complications can occur in
approximately one to five percent depending on the skill of the physician and the underlying
disorder. The most common complication is pancreatitis which is due to irritation of the
pancreas from the dye used to take pictures and can occur even with very experienced
physicians. This "injection" pancreatitis usually is treated in the hospital for one to two days.
Another possible complication is infection. Other serious risks including perforation of the
intestine, drug reactions, bleeding, depressed breathing. Irregular heart beat or heart attack
are extremely rare and is mainly due to the sedation. In case of complications, patients
usually need to be hospitalized, but surgery rarely is required.

3. b) spinal anaesthesia :
Introduction: Spinal anaesthesia (or spinal anesthesia), also called spinal
analgesia,spinal block or subarachnoid block (SAB), is a form of regional
anaesthesia involving injection of a local anaesthetic into thes ubarachnoid space,
generally through a fine needle.
Injected substances:
 Bupivacaine (Marcaine) is the local anaesthetic most commonly used.
 Lidocaine (lignocaine), tetracaine, procaine,ropivacaine, levobupivicaine, prilocaine an
d cinchocaine may also be used.
 Commonly opioids are added to improve the block and provide post-operative pain
relief.
Examples: include morphine, fentanyl, diamorphine or buprenorphine.
 Non-opioids like clonidine may also be added to prolong the duration of analgesia.
Mechanism:
 Regardless of the anaesthetic agent (drug) used, the desired effect is to block the
transmission of afferent nerve signals from peripheral nociceptors. Sensory signals
from the site are blocked, thereby eliminating pain.
 The degree of neuronal blockade depends on the amount and concentration of local
anaesthetic used and the properties of the axon.
 Thin unmyelinated C-fibres associated with pain are blocked first, while thick, heavily
myelinated A-alpha motor neurons are blocked moderately.
 Heavily myelinated, small preganglionic sympathetic fibers are blocked first. The
desired result is total numbness of the area.
 A pressure sensation is permissible and often occurs due to incomplete blockade of the
thicker A-beta mechanoreceptors. This allows surgical procedures to be performed with
no painful sensation to the person undergoing the procedure.
Limitations:
 Spinal anaesthetics are typically limited to procedures involving most structures below
the upper abdomen.
 To administer a spinal anaesthetic to higher levels may affect the ability to breathe by
paralysing the intercostal respiratory muscles, or even the diaphragm in extreme cases
(called a "high spinal", or a "total spinal", with which consciousness is lost), as well as
the body's ability to control the heart rate via the cardiac accelerator fibres.
 Also, injection of spinal anaesthesia higher than the level of L1 can cause damage to the
spinal cord, and is therefore usually not done.

3. c) Full thickness skin graft.

Full thickness skin grafts (Wolfe grafts)

 Contain epidermis plus the entire thickness of dermis.


 Adnexal structures, e.g. Hair, are included.
 Harvested by elliptical excision from sites of skin laxity, e.g. Postauricular skin crease,
supraclavicular, pre-auricular, groin, or medial upper arm skin.
 Graft secured with a tie-over dressing, e.g. Proflavine-soaked cotton wool, and inspected
after a week.
 Donor site sutured closed.

3. d) Hypertrophic pyloric stenosis of infancy.

Answer.

o This is the most common surgical cause of non-bilious vomiting in infants.


o It occurs in 1 of 400 live births.
o The male-to-female ratio is 4:1.

Diagnosis:

 History and physical examination


 Occurs generally in neonates who are 2 to 5 weeks of age.
 Vomiting
 Characteristically forceful or projectile and occurs 30 to 60 minutes after
feeding.
 Formula intolerance initially suspected but does not resolve with change of
feeds.
 Dehydration
 Lethargy.
 Absence of tears.
 Sunken anterior fontanelle.
 Dry mucous membranes.
 Decreased urine output.
 The “olive” mass
 Mass palpated to the right and above the umbilicus.
 Approximately 2 cm in diameter, firm, and mobile.
 Abdominal ultrasonography:

 Pyloric diameter greater than 14 mm,


 muscular thickness greater than 4 mm,
 and pyloric length greater than 16 mm
are diagnostic of pyloric stenosis .

Upper GI contrast study:

 Enlarged stomach.
 Poor gastric emptying.
 Elongated, narrow pyloric channel or “string sign.”
o Management:
 Preoperative fluid resuscitation

 20-mL/kg bolus.
 5% dextrose in normal saline to achieve urine output of 2 mL/kg/hour.
 Addition of potassium and changing to 5% dextrose in 0.45% normal saline
occurs when urine output is adequate.

 Correction of the hypochloremic hypokalemic metabolic alkalosis.


 Operative intervention
 Indicated only after adequate resuscitation and correction of metabolic alkalosis.
 Pyloromyotomy
 Division of the hypertrophied pyloric muscle, leaving the mucosa intact.
 Open or laparoscopic technique.
 Postoperative feeding
 Begin electrolyte solution by mouth 6 hours after pyloromyotomy.
 Over the next 12 hours, formula or pumped breast milk can be started and
should reach goal within 24 hours.
 Parents should be advised that vomiting may occur postoperatively as a result
of swelling at the pyloromyotomy, but this problem is self-limited.
 If the pyloric mucosa is perforated and repaired during surgery, nasogastric
drainage is recommended for 24 hours.

3. e) Neoadjuvant chemotherapy

Answer. Neoadjuvant therapy is the administration of therapeutic agents before a main


treatment.
One example is neoadjuvant hormone therapy prior to radical radiotherapy for
adenocarcinoma of the prostate.
Neoadjuvant therapy aims to reduce the size or extent of the cancer before using radical
treatment intervention, thus both making procedures easier and more likely to succeed and
reducing the consequences of a more extensive treatment technique, which would be
required if the tumor were not reduced in size or extent.
Another related concept is that neoadjuvant therapy acts on micrometastatic disease. The
downstaging is then a surrogate marker of efficacy on undetected dissemination, resulting in
improved longtime survival compared to the surgery-alone strategy.
This systemic therapy (chemotherapy, immunotherapy or hormone therapy) or radiation
therapy is commonly used in cancers that are locally advanced, and clinicians plan an
operation at a later stage. The use of such therapy can effectively reduce the difficulty
and morbidity of more extensive procedures.
The use of therapy can turn a tumor from untreatable to treatable by shrinking the volume.
Often, it is unclear which surrounding structures are directly involved in the disease and
which are just showing signs of inflammation. By administering therapy, a distinction can
often be made. Some doctors give the therapy in the hope that a response is seen, and they
can then decide what is the best course of action.
However, not everyone is suitable for therapy in this way because it can be extremely toxic.
Some patients react so severely that further treatments, especially surgery, are precluded,
and the patient is rendered unfit for anesthesia.

4. a) Dental cyst:
Answer.
o The periapical cyst (also termed radicular cyst, and to a lesser extentdental cyst) is the
most common odontogenic cyst.
o It is caused by pulpal necrosis secondary to dental caries or trauma.
o It arises from epithelial residues in periodontal ligament as a result of inflammation. The
inflammation usually follows death of dental pulp. Radicular cysts are found at root
apices of involved teeth. These cysts may persists even after extraction of offending
tooth; such cysts are called residual cysts.
o The cyst lining is derived from the cell rests of Malassez.
o Usually, the periapical cyst is asymptomatic, but a secondary infection can cause pain.
o On radiographs, it appears a radiolucency (dark area) around the apex of a tooth's root.

Signs and symptoms:


o Dental cysts can cause several problems but some remain undetected for months or
even years. Many cause problems when they become infected (causing pain and/or
swelling).
o They slowly replace some bony tissue (they can weaken the jaws),
o They press against other teeth and structures.
o They prevent the normal function of the teeth and mouth tissues.
Treatment:
o The type of treatment patients will receive usually depends on the type of cyst they have.
Usually, an emergency dentist drains the pus from an infected cyst to relieve pain and
pressure for those patients with an abscess.
o In emergency dentist will clean the space where the abscess was previously located to
remove any remnants of infection or other debris. ‘
o The treatment for a periapical abscess – which is formed due to an infection inside the
pulp of the tooth – is even more complex than the other forms, with an initial X-ray
required to identify its exact location. A root canal procedure will then be carried out
after the cyst is identified.

4. b) Post traumatic pneumothorax

Answer. A traumatic pneumothorax is when part of your lung collapses. A traumatic


pneumothorax is caused by an injury that tears your lung and allows air to enter the pleural
space. The air trapped in your pleural space prevents your lung from filling with air, which
causes it to collapse.

Causes of traumatic pneumothorax include the following:


 Trauma - Penetrating and nonpenetrating injury
 Rib fracture
 High-risk occupation (eg, diving, flying)
Traumatic pneumothorax can result from both penetrating and nonpenetrating lung
injuries. Complications include hemopneumothorax and bronchopleural fistula. Traumatic
pneumothoraces often can create a one-way valve in the pleural space (only letting in air
without escape) and can lead to a tension pneumothorax.

Investigations:

 Chest X-ray
 CT thorax
 ABG

Management depends not only on the clinical setting, the site where we treat the patient
(site of trauma or in the hospital), any procedure which is causing pneumothorax, but also
on the size of pneumothorax, associated co-morbid condition, whether it is open/closed and
simple/tension pneumothorax.

Treatment:

Tube thoracostomy
This procedure is recommended if simple aspiration proves ineffective and thoracoscopy is
not readily available. The site for the insertion is same as for simple aspiration. It rapidly
results in the re-expansion of the underlying lung and does not require prolonged
hospitalization. Risk of re-expansion pulmonary edema is greater when the lung is re-
expanded rapidly, it is probably better to use water seal and to avoid suction for the first 24
h of tube thoracostomy.
If the lung remains unexpanded or if there is a persistent air leak 72 h after tube
thoracostomy, consideration should be given to performing thoracoscopy or thoracotomy.
4. c) Intercostals chest tube drainage

Answer:

Introduction:

 Also known as chest drain.


 A chest drain is a tube inserted through the chest wall between the ribs and into the
pleural cavity to allow drainage of air (pneumothorax), blood (haemothorax), fluid
(pleural effusion) or pus (empyema) out of the chest.

Characteristics:

Chest tubes are commonly made from clear plastics like PVC and soft silicone. Chest tubes
are made in a range of sizes measured by their external diameter from 6 Fr to 40 Fr. Chest
tubes, like most catheters, a measured in French catheter scale. For adults, 20 Fr to 40 Fr
(6.7 to 13.3mm external diameter) are commonly used, and 6 Fr to 26 Fr for children.
Conventional chest tubes feature multiple drainage fenestrations in the section of the tube
which resides inside the patient, as well as distance markers along the length of the tube,
and a radiopaque stripe which outlines the first drainage hole. Chest tubes are also provided
in right angle, trocar, flared, and tapered configurations for different drainage needs. As
well, some chest tubes are coated with heparin to help prevent thrombus formation, though
the effect of this is disputed.
Channel style chest drains, also called Blake drains, are so-called silastic drains made of
silicone and feature open flutes that reside inside the patient. Drainage is thought to be
achieved by capillary action, allowing the fluids to travel through the open grooves into a
closed cross section, which contains the fluid and allows it to be suctioned through the tube.
Though these chest tubes are more expensive than conventional ones, they are theoretically
less painful, and drainage with these chest tubes has been proven to be clinically adequate
for cardiac surgery drainage.
Indications:

 Pneumothorax: accumulation of air or gas in the pleural space.


 Pleural effusion: accumulation of fluid in the pleural space

 Chylothorax: a collection of lymphatic fluid in the pleural space


 Empyema: a pyogenic infection of the pleural space
 Hemothorax: accumulation of blood in the pleural space
 Hydrothorax: accumulation of serous fluid in the pleural space
 Postoperative: for example, thoracotomy, oesophagectomy, cardiac surgery.

Contraindications:

 Contraindications to chest tube placement include refractory coagulopathy


 Presence of a diaphragmatic hernia.
 Hepatic hydrothorax.
 Additional contraindications include scarring in the pleural space (adhesions).
Complications:
 Major insertion complications include hemorrhage, infection, and
reexpansion pulmonary edema.
 Injury to the liver, spleen or diaphragm is possible if the tube is placed inferior to the
pleural cavity. Injuries to the thoracic aorta and heart can also occur.
 Minor complications include a subcutaneous hematoma or seroma, anxiety,
shortness of breath (dyspnea), and cough (after removing large volume of fluid).
 Chronic pain related to chest tube induced scarring of the intercostal space is not
uncommon.
 Subcutaneous emphysema indicates backpressure created by a clogged drain or
insufficient negative pressure.

4. d) Extradural haemorrhage

Answer.

Epidural or extradural hematoma (haematoma) is a type of traumatic brain injury (TBI) in


which a buildup of blood occurs between the dura mater (the tough outer membrane of
the central nervous system) and the skull.
The dura mater also covers the spine, so epidural bleeds may also occur in the spinal
column. Often due to trauma, the condition is potentially deadly because the buildup of
blood may increase pressure in the intracranial space and compress delicate brain tissue.
The condition is present in one to three percent of head injuries.
Between 15 and 20% of patients with epidural hematomas die of the injury.

Clinical features:
o Epidural bleeds are extra-axial bleeds, occurring outside of the brain tissue.
o Epidural bleeding is rapid because it is usually from arteries, which are high pressure.
Epidural bleeds from arteries can grow until they reach their peak size at six to eight
hours post injury, spilling from 25 to 75 cubic centimeters of blood into
the intracranial space. As the hematoma expands, it strips the dura from the inside of
the skull, causing an intense headache.
o Epidural bleeds can become large and raise intracranial pressure, causing the brain to
shift, lose blood supply, or be crushed against the skull. Larger hematomas cause more
damage. Epidural bleeds can quickly expand and compress the brain stem, causing
unconsciousness, abnormal posturing, and abnormal pupil responses to light.
In the hallmark of epidural hematoma, patients may regain consciousness during
what is called a lucid interval, only to descend suddenly and rapidly into
unconsciousness later.
The lucid interval, which depends on the extent of the injury, is a key to diagnosing epidural
hemorrhage. If the patient is not treated with prompt surgical intervention, death is likely to
follow.

Imaging:
On images produced by CT scans and MRIs, epidural hematomas usually appear convex in
shape because their expansion stops at skull's sutures, where the dura mater is tightly
attached to the skull.
Thus they expand inward toward the brain rather than along the inside of the skull, as
occurs in subdural hematoma. The lens like shape of the hematoma leads the appearance of
these bleeds to be called "lentiform".
Epidural hematomas may occur in combination with subdural hematomas, or either may
occur alone. CT scans reveal subdural or epidural hematomas in 20% of unconscious
patients.
Causes:
o The interior of the skull has sharp ridges by which a moving brain can be injured.
o The main cause of epidural hematoma is usually traumatic, although spontaneous
hemorrhage is known to occur. Hemorrhages commonly result from acceleration-
deceleration trauma and transverse forces. 10% of epidural bleeds may be venous, due
to shearing injury fromrotational forces. Epidural hematoma commonly results from a
blow to the side of the head. The pterion region which overlies the middle meningeal
artery is relatively weak and prone to injury. Thus only 20 to 30% of epidural
hematomas occur outside the region of the temporal bone.
o The brain may be injured by prominences on the inside of the skull as it scrapes past
them.
o Epidural hematoma is usually found on the same side of the brain that was impacted by
the blow, but on very rare occasions it can be due to a contrecoup injury.
Treatment:

 Minimally invasive surgical procedures, including the use of burr holes and negative
pressure drainage, may be used in selected cases.

Novel therapeutic approaches include the following:

 Endovascular embolization to minimize bleeding during the acute stage


 Thrombolytic evacuation using closed suction drain
 Minimally invasive evacuation

As with other types of intracranial hematomas, the blood may be aspirated surgically to
remove the mass and reduce the pressure it puts on the brain.
 The hematoma is evacuated through a burr hole or craniotomy. If transfer to a facility
with neurosurgery is prolonged trephination may be performed in the emergency
department.

Consultations with any of the following may prove useful:

 Neurosurgeon (for potential emergent evacuation of the hematoma)


 Neurologist
 Rehabilitation specialist.

4. e) Venous ulcer
Answer.

Introduction:
 Chronic venous disease, including chronic venous insufficiency and chronic venous
ulceration, is a common and important medical problem that causes significant
morbidity. Venous ulcers are expensive to treat and adversely impact patient’s quality
of life.
 Venous ulcers occur more commonly in the elderly, the peak prevalence occurring
between ages 60 and 80 years.
 A venous leg ulcer can develop after a minor injury if there is a problem with the
circulation of blood in your leg veins. If this happens, the pressure inside the veins
increases.

Risk factors:
 Obesity – this increases the risk of high pressure in the leg veins
 Not being able to move for a long period of time – this can weaken the calf muscles,
which can affect circulation in the leg veins
 Having previously had deep vein thrombosis (DVT)– blood clots that develop in the
leg, which can damage valves
 Varicose veins – swollen and enlarged veins caused by malfunctioning valves
 Previous injury to the leg, such as a broken or fractured bone, which may cause DVT
 Previous surgery to the leg, such as a hip replacement or knee replacement, which
can prevent you from moving about
 Increasing age – as people generally find it harder to move about as they get older

Clinical features:
Venous leg ulcers are open, often painful, sores in the skin that take more than four to six
weeks to heal. They most often develop on the inside of the leg, just above the ankle.
Symptoms:
 Swollen ankles (oedema)
 Discolouration and darkening of the skin around the ulcer
 Hardened skin around the ulcer, which may make your leg feel hard and resemble
the shape of an upside-down champagne bottle
 A heavy feeling in your legs
 Aching or swelling in your legs
 Red, flaky, scaly and itchy skin on your legs (varicose eczema)
 Swollen and enlarged veins on your legs (varicose veins)
 An unpleasant and foul-smelling discharge from the ulcer

Signs of an infection:
A venous leg ulcer can be susceptible to bacterial infection. Symptoms of an infected leg
ulcer can include:
 Worsening pain
 A green or unpleasant discharge coming from the ulcer
 Redness and swelling of the skin around the ulcer
 A high temperature (fever)
Nonsurgical Treatment

 Infected ulcers

 Necessitate treatment of the infection first.


 Staphylococcus aureus, Streptococcus pyogenes, and Pseudomonas species are
responsible for most infections.
 Usually treated with local wound care, wet-to-dry dressings, and oral antibiotics.
 Topical antiseptics should be avoided.
 Severe infections require intravenous antibiotics.

 Leg elevation

Leg elevation can temporarily decrease edema and should be instituted when swelling
occurs. This should be done before a patient is fitted for stockings or boots.

 Compression therapy
 Compression therapy is the primary treatment for CVI.

 Elastic compression stockings


o Fitted to provide a compression gradient from 30 to 40 mm Hg, with the greatest
compression at the ankle.
o Donned on arising from bed and removed at bedtime.
o Effective in healing ulcers but can take months to obtain good results.
o Stockings do not correct the abnormal venous hemodynamics and must be worn
after the ulcer has healed to prevent recurrence.
o Principal drawback is patient compliance.
 Unna boots
o Paste gauze compression dressings that contain zinc oxide, calamine, and
glycerin.
o Used to help prevent further skin breakdown.
o Provide nonelastic compression therapy.
o Changed once or twice a week.
o Healing time for ulcers is less than that of elastic compression alone.
 Pneumatic compression devices
o Provide dynamic sequential compression.
o Used primarily in the prevention of deep vein thrombi in hospitalized patients.
o Also used successfully to treat venous insufficiency.

 Topical medications

 Largely ineffective as a stand-alone therapy for venous stasis ulcers.


 Topical therapy is directed at absorbing wound drainage and avoiding desiccation of
the wound.
 Antiseptics can be counterproductive. Hydrogen peroxide, povidone-iodine, acetic
acid, and sodium hypochlorite are toxic to cultured fibroblasts and should be used for
the shortest duration necessary to control ulcer infection.

 Surgical Therapy: Skin grafting.


The West Bengal University of Health Sciences

M.B.B.S. 3rd Professional Part – II, Supplementary Examination, 2018

Subject: Surgery Time: 21/2 hrs.

Paper: II Marks: 60

Group – A

1. Discuss the presenting symptoms of Benign Hyperplasia of Prostate. How will you
manage a 65 year old male patient with acute retention of urine in emergency and
subsequently? 5+5+5
Group – B
2. a) Outline the etiopathogenesis of multinodular goiter. Describe its management. 5+10
Or
b) Discuss the etiopathology of extradural haematoma. Describe its management.
5+10

Group – C
3. Answer in brief on any three of the following: 3x5
a) Classification of nerve injury
b) Management of pneumothorax
c) Criteria of brain death
d) Types of skin grafting
e) Chemotherapy of testicular cancer.
Group – D
4. Write short notes (any three): 3x5
a) Uses of LASERs in surgery
b) Cleft lip
c) Glasgow coma scale
d) Preoperative assessment of pulmonary function
e) Dentigerous cyst.

Answers.

1. Clinical presentations of brnign hyperplasia of prostate:


BPH is the most common cause of lower urinary tract symptoms (LUTS), which are
divided into storage, voiding, and symptoms which occur after urination.
Storage symptoms include the need to urinate frequently, waking at night to urinate,
urgency (compelling need to void that cannot be deferred), involuntary urination,
including involuntary urination at night, or urge incontinence (urine leak following a
strong sudden need to urinate).
Voiding symptoms include urinary hesitancy (a delay between trying to urinate and the
flow actually beginning), intermittency (not continuous), involuntary interruption of
voiding, weak urinary stream, straining to void, a sensation of incomplete emptying, and
terminal dribbling (uncontrollable leaking after the end of urination, also called post-
micturition dribbling).These symptoms may be accompanied by bladder pain or pain
while urinating, called dysuria.
Bladder outlet obstruction (BOO) can be caused by BPH. Symptoms are abdominal pain, a
continuous feeling of a full bladder, frequent urination, acute urinary retention (inability
to urinate), pain during urination (dysuria), problems starting urination (urinary
hesitancy), slow urine flow, starting and stopping (urinary intermittency), and nocturia.
BPH can be a progressive disease, especially if left untreated. Incomplete voiding results
in residual urine or urinary stasis, which can lead to an increased risk of urinary tract
infection.

Causes of acute retention of urine in this patient:

Obstructive:
 Benign prostatic hyperplasia
 Urethral Strictures
 Bladder calculi
 Faecal Impactation
 Phimosis / paraphimosis
 Benign/malignant pelvic masses
 Meatal Stenosis
 Stone in the urethral meatus

Infectious and Inflammatory Causes:

 Prostatitis
 Prostatic abscess
 Balantitis
 Cystitis
 Bilharziasis
 Herpes simplex virus

Pharmacologic Causes:

 Drugs with anticholingeric properties eg: tricylic antidepressants


(amitriptyline)
 Opioids
 Sympathomimetic drugs eg: oral decongestants containing Ephedrine.
 NSAIDs
 Antiparkinsonian agents (levodopa)
 Antipsychotics (chlopromazine)
 Muscle relaxants (Baclofen)

Neurologic Causes:

 Autonomic or peripheral nerve : Diabetes mellitus, Guillain-Barre syndrome,


Pernicious anaemia, radical pelvic surgery.
 Brain: CVA, MS, Tumour, Parkinson’s disease, concussion.
 Spinal cord : Haematoma / abscess / tumour, Cauda equine, spina bifida
occulta.

Diagnosis:

 Proper history.
 Clinical examination: including digital rectal examination.
 Investigations:

Radiological investigations:

 KUB - to diagnose stone disese


 Abdominal US - to diagnose stone disese–, prostatic hypertrophy.
 IVP
 Helical CTU

Advantages over IVP:


»
greater specificity (95%) and sensitivity (97%) for diagnosing ureteric stones
»
Can identify other, non-stone causes of flank pain.
»
No need for contrast administration.
»
Faster, taking just a few minutes
»
the cost of CTU is equivalent to that of IVU
 MRI
 Very accurate way of determining whether or not a stone is present in the ureter
 Very high cost
 Urethrography: for stricture.

Initial management:

o Check the blood pressure, pulse rate and hydration of the patient.
o Palpate the bladder – whether full or not.
o Drain the bladder – by Foley’s catheterisation - if failed – Suprapubic catheterisation.
o Antibiotics.
o Analgesics.
o Try to find out the actual cause.

Definitive treatment: treat the cause.

 BPH develops in the transition zone.


 It is truly a hyperplastic process resulting from an increase\in cell number.
 Microscopic evaluation reveals a nodular,growth pattern that is composed of varying
amounts of stroma and epithelium. Stroma is composed of varying amounts of
collagen and smooth muscle.
 The differential representation of the histologic components of BPH explains, in part,
the potential responsiveness to medical therapy.
 As BPH nodules in the transition zone enlarge, they compress the outer zones of the
prostate, resulting in the formation of a so-called surgical capsule. This boundary
separates the transition zone from the peripheral zone and serves as a cleavage plane
for open enucleation of the prostate during open simple prostatectomies performed
for BPH.

2. a) Factors that may be involved in the evolution of Muitinodular Goiter.


 Primary factors
 Functional heterogeneity of normal follicular cells, cause unknown, possibly genetic
and acquisition of new inheritable qualities by replicating epithelial cells
 Subsequent functional and structural abnormalities in growing goiters
 Secondary factors (Stimuli to New Follicle Generation)
 TSH (induced by, e.g., iodine deficiency, goitrogens, inborn errors of thyroid hormone
synthesis)
 Other thyroid-stimulating factors

Pathogenesis:

 Reflects impaired synthesis of thyroid hormone most often caused by iodine deficiency

 Impairment leads to compensatory ↑ in TSH levels → hypertrophy and hyperplasia of


follicular cells → gross enlargement of gland

 Euthyroid metabolic state

 Degree of enlargement is proportional to level and duration.

 The pathogenesis of MNG encompasses processes of diffuse follicular hyperplasia, focal


nodular proliferation and eventual acquisition of functional automaticity. The
development of MNG is a result of long-term exposure of the thyroid gland to
proliferative stimuli, such as iodine deficiency, goitrogens and inborn error of thyroid
hormone synthesis. All of the above results in insufficient thyroid hormone production
and stimulate pituitary secretion of thyroid stimulating hormone (TSH).
{The pathogenesis of MNG encompasses processes of diffuse follicular hyperplasia, focal
nodular proliferation and eventual acquisition of functional automaticity.
The development of MNG is a result of long-term exposure of the thyroid gland to
proliferative stimuli, such as iodine deficiency, goitrogens and inborn error of thyroid
hormone synthesis.
All of the above results in insufficient thyroid hormone production and stimulate pituitary
secretion of thyroid.}

Management of multinodular goiter:


Investigations:
The investigative workup of a patient with MNG includes
 T4 and TSH levels to evaluate thyroid function.
 High resolution USG and FNA should be used for suspicious or dominant nodules where
malignancy is suspected. Predominantly solid nodules especially when they are
hypoechoic or display a sonoluscent rim surrounding the lesion(―halo sign‖) should be
evaluated by FNAC.
 X ray Neck AP and Lateral views to assess tracheal position, retro tracheal extension or
incipient compression.
 Chest X ray to evaluate for any features of retrosternal extension.
 CT/MRI are useful in select cases where a retrosternal extension is suspected. It provides
accurate delineation of the depth of goiter extension into the chest and its relation to the
trachea, esophagus and great veins.
 Thyroid scintigraphy is another option to confirm the extent and functional status of the
gland but it is not needed routinely.

 The indications for treatment are as following


Strong
o Compressive symptoms
o Hyperthyroidism
o Suspected malignancy
Relative
o Cosmesis
o Potential for Tracheo-esophageal compression

The various therapeutic options include


· Suppressive levothyroxine therapy
· Surgical
· Radioactive iodine therapy.

 Suppressive Levothyroxine Therapy:


 The results of T4 suppressive therapy are inconsistent and marginal. This therapy
has a limited role in management of patients with MNG. The aim of such therapy is to
consistently suppress TSH levels to <0.5 mu/l. The problem of this therapy is that a
significant number of patients become hyperthyroid with time. Only small goiter
would respond and that too partially. Goitres that respond do so within a period of 6
months
 Radioactive Iodine Therapy:
 It is of limited value and is useful in only two classes of patients:
 Those with small goiters may benefit.
 For patients with substantially increased perioperative risk and reasonable
thyroid gland function.
 RAI therapy is of no value in large multinodular goiters with poorly functioning
nodules as the efficacy of RAI therapy depends on the presence of reasonable gland
activity all over the thyroid.
 Surgery:
 Surgical treatment is the modality of choice in the management of patients with
multinodular goitre.
 The results of surgery are immediate and tissue is available for histological
confirmation of the diagnosis and evaluation for any malignant change.
 Surgery is the only treatment option in those with compressive symptoms or those
with suspected malignancy.
 Surgical treatment options are between Subtotal and Total thyroidectomy.

2. b) Aetiopathology of extradural haematoma:

Extraduural / epidural haematoma is when bleeding occurs between the tough outer
membrane covering the brain (dura mater), and the skull.

o Often there is loss of consciousness following a head injury, a brief regaining of


consciousness, and then loss of consciousness again.
o Other symptoms may include headache, confusion, vomiting, and an inability to move
parts of the body.[1]
o Complications may include seizures.
o The cause is typically head injury that results in a break of the temporal bone and
bleeding from the middle meningeal artery.
o Occasionally it can occur as a result of a bleeding disorder or blood vessel malformation.
o Diagnosis is typically by a CT scan or MRI.
o When this condition occurs in the spine it is known as a spinal epidural hematoma
Causes:

 The most common cause of intracranial epidural hematoma is traumatic, although


spontaneous hemorrhage is known to occur. Hemorrhages commonly result from
acceleration-deceleration trauma and transverse forces.
 The majority of bleeds originate from meningeal arteries, particularly in the temporal
region.
 10% of epidural bleeds may be venous, due to shearing injury from rotational forces.
Epidural hematoma commonly results from a blow to the side of the head.
 The pterion region which overlies the middle meningeal artery is relatively weak and
prone to injury.
 Thus only 20 to 30% of epidural hematomas occur outside the region of the temporal
bone.
 The brain may be injured by prominences on the inside of the skull as it scrapes past
them. Epidural hematoma is usually found on the same side of the brain that was
impacted by the blow, but on very rare occasions it can be due to a contrecoup injury.

Imaging:
 On images produced by CT scans and MRIs, epidural hematomas usually appear convex
in shape because their expansion stops at skull's sutures, where the dura mater is
tightly attached to the skull.
 Thus they expand inward toward the brain rather than along the inside of the skull, as
occurs in subdural hematoma. The lens like shape of the hematoma leads the
appearance of these bleeds to be called "lentiform".
 Epidural hematomas may occur in combination with subdural hematomas, or either
may occur alone. CT scans reveal subdural or epidural hematomas in 20% of
unconscious patients.
Treatment:

 Minimally invasive surgical procedures, including the use of burr holes and negative
pressure drainage, may be used in selected cases.

Novel therapeutic approaches include the following:

 Endovascular embolization to minimize bleeding during the acute stage


 Thrombolytic evacuation using closed suction drain
 Minimally invasive evacuation

As with other types of intracranial hematomas, the blood may be aspirated surgically to
remove the mass and reduce the pressure it puts on the brain.
 The hematoma is evacuated through a burr hole or craniotomy. If transfer to a facility
with neurosurgery is prolonged trephination may be performed in the emergency
department.

Consultations with any of the following may prove useful:

 Neurosurgeon (for potential emergent evacuation of the hematoma)


 Neurologist
 Rehabilitation specialist

3. a) Types of nerve injuries:


 Neuropraxia- physiologic block of nerve conduction within an axon without any anatomical
interruption.

Many infants with birth brachial plexus injury have neuropraxia and recover spontaneously
because neuropraxia tends to disappear within 4-6 weeks.

 Axonotmesis - anatomical interruption of the axon with no or only partial interruption of the
connective tissue framework.

This type of nerve injury requires regrowth of the axon to the target muscle, which takes a
considerable amount of time. This regrowth can be inhibited by scar formation. Whether
patients with axonotmesis will require surgical treatment depends on the number of disrupted
axons and the extent of scar formation at the site of nerve injury.

When an axon is disrupted, there is a 2-4 week delay before the axon starts to regenerate. Axons
grow in adults at about 1 inch per month, which means many months will be required for the
axon to grow down to the muscles in the arms. In infants, however, the axon may regenerate
more rapidly, and the distance to be covered is much less. When a muscle loses its innervation,
the nerve receptors will disappear over a period of 12 to 18 months. This affects the timing of
neurosurgical intervention, because a repair done too late will not have receptors in the
muscles for the growing nerves.

 Neurotmesis - complete anatomical disruption of the both the axon and all of the
surrounding connective tissue (rupture of the nerve).

Birth brachial plexus injury is sometimes associated with neurotmesis. This is the most severe
type of nerve injury and has no chance of spontaneous recovery. Early surgical treatment is
necessary.

3. b) Management of pneumothorax:

Chest radiograph

A pneumothorax is when looked for, usually relatively easily appreciated. Typically they
demonstrate:

 visible visceral pleural edge is seen as a very thin, sharp white line
 no lung markings are seen peripheral to this line
 peripheral space is radiolucent compared to the adjacent lung
 lung may completely collapse
 mediastinum should not shift away from the pneumothorax unless a tension
pneumothorax is present (discussed separately).
 subcutaneous emphysema and pneumomediastinum may also be present

In cases where these features are not clearly present a number of techniques can be
employed:
 lateral decubitus radiograph:
o should be done with the suspected side up
o the lung will then 'fall' away from the chest wall
 expiratory chest radiograph:
o lung becomes smaller and denser
o pneumothorax remains the same size and is thus more conspicuous: although
some authors suggest that there is no difference in detection rate 6

CT scan

When imaged supine detection can be difficult: see pneumothorax in a supine patient, and
pneumothorax is one cause of a transradiant hemithorax.

Provided lung windows are examined, a pneumothorax is very easily identified on CT, and
should pose essentially no diagnostic difficulty. When a bullous disease is present, a
loculated pneumothorax may appear similar.

Treatment and prognosis

Treatment depends on a number of factors:

 Size of the pneumothorax


 Symptoms
 Background lung disease/respiratory reserve

These can be used together to determine the best course of action. The following guidelines
are based on the British Thoracic Society guidelines for the treatment of pneumothorax;
local protocol may differ:

 Asymptomatic small rim pneumothorax (<2 cm): no treatment with follow up


radiology to confirm resolution
 Pneumothorax with mild symptoms (no underlying lung condition): needle
aspiration in the first instance
 Pneumothorax in a patient with background chronic lung disease or significant
symptoms: intercostal drain insertion (small drain using the Seldinger technique)

In patients with recurrent pneumothoraces or who are at very high risk of having recurrent
events and have a poor respiratory reserve, a pleurodesis can be performed. This can either
be medical (e.g. talc poudrage) or surgical (e.g. VATS pleurectomy, pleural abrasion,
sclerosing agent).

3. c) Criteria of brain death.

Clinical testing for brainstem death:


.
Absence of motor response Absence of spontaneous respiration

 Pupillary reflex The absence of a motor response to After preventilation with 100% O2 for at
 Corneal reflex painful stimuli applied to the head/face least 5 minutes, the patient is disconnected
 Pharyngeal (gag) and and the absence of a motor response from the ventilator for 10 minutes to
tracheal (cough) within the cranial nerve distribution to confirm absence of respiratory effort,
reflex adequate stimulation of any somatic area during which time the arterial PCO2 level
 Oculovestibular is an indicator of brainstem death. The should be >8 kPa (60 mmHg) to ensure
(caloric) reflex presence of spinal reflexes does not adequate respiratory stimulation. To
preclude brainstem death prevent hypoxia during the apnoeic period,
O2 (6 L/min) is delivered via an
endotracheal catheter.

3. d) A skin graft is a piece of dermis and epidermis that is completely removed from its
original bodily attachment (the donor site). It is fixed to a recipient site and develops a
new blood supply from the underlying tissue.

 Autograft: transfer from one part of a person's body to another part


 Isograft: transfer between genetically identical individuals
 Allograft: transfer between individuals of the same species
 Xenograft: transfer between individuals of different species

Full thickness skin grafts (Wolfe grafts)

 Contain epidermis plus the entire thickness of dermis.


 Adnexal structures, e.g. Hair, are included.
 Harvested by elliptical excision from sites of skin laxity, e.g. Postauricular skin crease,
supraclavicular, pre-auricular, groin, or medial upper arm skin.
 Graft secured with a tie-over dressing, e.g. Proflavine-soaked cotton wool, and inspected
after a week.
 Donor site sutured closed.

Split thickness skin grafts (Thiersch grafts)

 Consist of epidermis plus a variable thickness of dermis.


 Harvested by shaving off a layer of skin with a skin graft knife or dermatome. Can be
taken from any area of the body (thigh skin most often used—plentiful and easy to
access).
 Graft is often fenestrated (to stop blood or serous fluid collecting under it) or meshed (to
expand the graft).
 Graft secured with glue, sutures, or staples, then a non-adherent, compressive dressing.
Inspected after 5 days.
 Defect heals by re-epithlialization from skin appendages.

3. e) Chemotherapy in testicular tumour:

Stage I seminomas:

Chemotherapy: An option that works as well as radiation is 1 or 2 cycles of chemotherapy


with the drug carboplatin after surgery. Many experts prefer chemo over radiation because
it seems to be easier to tolerate. .
Stage IIA seminomas:

Chemotherapy: Another option is chemo, with either 4 cycles of EP (etoposide and cisplatin)
or 3 cycles of BEP (bleomycin, etoposide, and cisplatin). You doctor will watch you closely
(every 3 to 6 months) to look for signs that the cancer has come back.

Stage IIB seminomas:

Chemotherapy: This is the preferred treatment. You can get either 4 cycles of EP (etoposide
and cisplatin) or 3 cycles of BEP (bleomycin, etoposide, and cisplatin).

Stage IIC seminomas:

Chemotherapy with 4 cycles of EP or 3 or 4 cycles of BEP. Radiation therapy is generally not


used for stage IIC seminoma.

Stage IA non-seminomas:

 Chemotherapy: Instead of surgery, your doctor may suggest you get 1 cycle of the BEP
regimen (bleomycin, etoposide, and cisplatin). This helps reduce your risk of relapse.

Stage IB (T2, T3, or T4)

 Chemotherapy: Instead of surgery, your doctor may recommend 1 cycle of the BEP
regimen (bleomycin, etoposide, and cisplatin). If cancer was found in lymph nodes after
surgery, you may get 2 to 4 cycles of BEP or EP (etoposide, and cisplatin). Stage IS non-
seminoma.

If tumor marker levels (like AFP or HCG) are still high even after the cancer has been
removed, but the CT scan doesn't show a tumor, chemo is recommended. Patient may get
either 3 cycles of BEP (bleomycin, etoposide, and cisplatin) or 4 cycles of EP (etoposide and
cisplatin).

Stage IIA non-seminomas

If your tumor marker levels are normal

 Chemotherapy: If cancer was found in many lymph nodes, give either 4 cycles of EP
(etoposide and cisplatin) or 3 cycles of BEP (bleomycin, etoposide, and cisplatin). Stage
IIB non-seminomas

If tumor marker levels are normal,

 Chemotherapy: Give either 4 cycles of EP (etoposide and cisplatin) or 3 cycles of BEP


(bleomycin, etoposide, and cisplatin) may be used.

Stage III seminomas and non-seminomas:


Both stage III seminomas and non-seminomas are treated with radical inguinal orchiectomy
followed by chemo with either 4 cycles of EP (etoposide and cisplatin) or 3 or 4 cycles of BEP
(bleomycin, etoposide, and cisplatin).

Patient may get 4 cycles of BEP if you have an intermediate or poor risk non-seminoma.
(This depends on the spread to distant areas and tumor marker levels.) If there is medical
reasons that make treatment with bleomycin unsafe, then one may get VIP (vinblastine,
ifosfamide, and cisplatin) instead.

If the cancer is seminoma that has spread to bones, liver, or brain, it's intermediate risk then
give VIP (etoposide, mesna, ifosfamide, and cisplatin).

If the patient has high levels of the tumor marker HCG, distant spread of cancer is seen on
scans, and there's a high suspicion that he might have a testicular choriocarcinoma, chemo
may be started without a biopsy or surgery to remove the testicle.

4. a) Uses of laser in surgery:

 Laser surgery is a type of surgery that uses a laser (in contrast to using a scalpel) to
cut tissue.
 Examples include the use of a laser scalpel in otherwise conventional surgery, and
soft-tissue laser surgery, in which the laser beam vaporizes soft tissue with high
water content.
 Laser surgery is commonly used on the eye. Techniques used include LASIK, which is
used to correct near and far-sightedness in vision, and photorefractive keratectomy,
a procedure which permanently reshapes the cornea using an excimer laser to
remove a small amount of the human tissue.
 Types of surgical lasers include carbon dioxide, argon, Nd:YAG laser, and potassium
titanyl phosphate, from among others.

Applications:

o Soft tissue: Soft-tissue laser surgery is used in a variety of applications in human


(general surgery, neurosurgery, ENT, dentistry, orthodontics, and oral and
maxillofacial surgery)

o Dermatology and plastic surgery:


A range of lasers such as erbium, dye, Q switch lasers and CO2 are used to treat
various skin conditions including scars, vascular and pigmented lesions, and for
photorejuvenation. The laser surgery for dermatology often bypass the skin surface.
The principle of laser surgery for dermatologic problem is based on SPTL(selective
photothermolysis). The laser beam penetrates the skin until it encounters
chromophore which absorbs the laser beam. After absorption of the laser beam, heat
is generated to induce coagulation, necrosis of the targeted tissue, this results in
removal of unwanted tissue by laser surgery.

Lasers are also used for laser-assisted lipectomy.

o Eye surgery: Various types of laser surgery are used to treat refractive error:
 LASIK, in which a knife is used to cut a flap in the cornea, and a laser is used to
reshape the layers underneath, to treat refractive error

 Intra LASIK, a variant in which the flap is also cut with a laser

 Photorefractive keratectomy (PRK, LASEK), in which the cornea is reshaped without


first cutting a flap
 Laser thermal keratoplasty, in which a ring of concentric burns is made in the cornea,
which cause its surface to steepen, allowing better near vision

Lasers are also used to treat non-refractive conditions, such as:

 Phototherapeutic keratectomy (PTK), in which opacities and surface irregularities


are removed from the cornea
 Laser coagulation, in which a laser is used to cauterize blood vessels in the eye, to
treat various conditions
 Lasers can be used to repair tears in the retina.

o Endovascular surgery: Laser endarterectomy is a technique in which an entire


atheromatous plaque in the artery is excised. Laser recanalization of blocked
arteries. other applications include laser assisted angioplasties and laser assisted
vascular anastomosis.
o Foot and ankle surgery: Lasers are used to treat several disorders in foot and ankle
surgery. They are used to remove benign and malignant tumors,[12] treat bunions,[13]
debride ulcers and burns, excise epidermal nevi, blue rubber bleb nevi, and keloids,
and the removal of hypertrophic scars and tattoos.[14]
o A carbon dioxide laser (CO2) is used in surgery to treat onychocryptosis (ingrown
nails), onychauxis (club nails), onychogryposis (rams horn nail), and onychomycosis
(fungus nail).
o Gastro-intestinal tract:

 Peritoneum-Laser is used for adhesiolysis.


 Peptic ulcer disease and oesophageal varices - Laser photoablation is done.
 Coagulation of vascular malformations of stomach, duodenum and colon.
 Lasers can be effectively used to treat early gastric cancers provided they are less
than 4 cm and without lymph node involvement. Lasers are also used in treating oral
submucous fibrosis.
 Palliative laser therapy is given in advanced oesophageal cancers with obstruction of
lumen. Recanalisation of the lumen is done which allows the patient to resume soft
diet and maintain hydration.
 Ablative laser therapy is used in advanced colorectal cancers to relieve obstruction
and to control bleeding.
 Laser surgery used in hemorrhoidectomy, and is a relatively popular and non-
invasive method of hemorrhoid removal.
 Laser-assisted liver resections have been done using carbon dioxide and Nd:YAG
lasers.
 Ablation of liver tumors can be achieved by selective photovaporization of the tumor.
 Endoscopic laser lithotripsy is a safer modality compared to electrohydraulic
lithotripsy.
o Oral and dental surgery: The CO2 laser is used in oral and dental surgery for virtually
all soft-tissue procedures, such as gingivecomies, vestibuloplasties, frenectomies and
operculectomies.

The CO2 10,600 nm wavelength is safe around implants as it is reflected by titanium,


and thus has been gaining popularity in the field of periodontology. The laser may
also be effective in treating peri-implantitis.

o Spine surgery: Laser spine surgery first began seeing clinical use in the 1980s and
was primarily used within discectomy to treat lumbar disc disease under the notion
that heating a bulging disc vaporized enough tissue to relieve pressure on the nerves
and help alleviate pain.
o Other surgery:The CO2 laser is also used in gynecology, genitourinary, general and
thoracic surgery, otorhinolaryngology, orthopedic, and neurosurgery.
o Hard tissues:Lasers are used to cut or ablate bones and teeth in dentistry.

4. b) Cleft lip:
Cleft lip: Clefts of the lip (CL) and alveolus with or without cleft of palate (CL/P) are the most
common congenital craniofacial defects and the fourth most frequent birth defects in the
world.

Clefts of the lip may be


 Unilateral or bilateral,
 Complete or incomplete,
 Right or left sided and
 May occur with or without cleft palate.

They frequently occur as isolated anomalies but may be associated with syndromes such as
 Treacher Collins syndrome,  Goldenhar syndrome,
 Down syndrome,  Pierre Robin sequence,
 Apert syndrome,  Van der Woude syndrome

Risk factors:

 Family history: Parents with a family history of cleft lip or cleft palate face a higher risk of
having a baby with a cleft.
 Race: Common in Native Americans and least common in African-Americans.
 Sex: Males are twice as likely to have a cleft lip with or without cleft palate. Cleft palate
without cleft lip is more common in females.
 Exposure to certain substances during pregnancy: Cleft lip and cleft palate may be more
likely to occur in pregnant women who smoke cigarettes, drink alcohol or take certain
medications.
 Having diabetes: Women diagnosed with diabetes before pregnancy may have an
increased risk of having a baby with a cleft lip with or without a cleft palate.
 Being obese during pregnancy. There is some evidence that babies born to obese women
may have increased risk of cleft lip and palate.

Classification:
Group I: Cleft lip only (Pre-alveolar clefts)
Group Ia: Cleft of lip and alveolus
Group II: Cleft palate only (Post alveolar clefts)
Group III: Cleft of lip, alveolus and palate (Alveolar clefts)
Group I and III clefts are subdivided into unilateral / bilateral / median, and Group II is
subdivided into clefts of hard and soft palates.

Complications:

 Difficulty with feedings


 Difficulty swallowing, with potential for liquids or foods to come out the nose
 Nasal speaking voice

Diagnosis:

Traditionally, the diagnosis is made at the time of birth by physical examination. Recent
advances in prenatal diagnosis have allowed obstetricians to diagnose facial clefts in utero.

Treatment:

 Within the first 2–3 months after birth, surgery is performed to close the cleft lip.
 While surgery to repair a cleft lip can be performed soon after birth, often the preferred
age is at approximately 10 weeks of age, following the "rule of 10s" coined by surgeons
 If the cleft is bilateral and extensive, two surgeries may be required to close the cleft, one
side first, and the second side a few weeks later. The most common procedure to repair a
cleft lip is the Millard procedure.

4. c) Glasgow Coma Scale: The Glasgow Coma Scale or GCS is a neurological scale that aims
to give a reliable, objective way of recording the conscious state of a person for initial as
well as subsequent assessment. A patient is assessed against the criteria of the scale, and
the resulting points give a patient score between 3 (indicating deep unconsciousness)
and either 14 (original scale) or 15 (the more widely used modified or revised scale).
GCS was initially used to assess level of consciousness after head injury, and the scale is now
used by first aid, EMS, nurses and doctors as being applicable to all acute medical and
trauma patients.
In hospitals it is also used in monitoring chronic patients in intensive care.

Glasgow coma scale (GCS)


Elements of the scale
 Eye response (E)
 Verbal response (V)
 Motor response (M)
Minimum score, 3; maximum score, 15.

Generally, brain injury is classified as:


 Severe, with GCS < 8–9
 Moderate, GCS 8 or 9–12 (controversial)
 Minor, GCS ≥ 13.

 Generally when a patient is in a decline of their GCS score, the nurse or medical staff
should assess the cranial nerves and determine which of the twelve have been affected.
 Tracheal intubation and severe facial/eye swelling or damage make it impossible to test
the verbal and eye responses. In these circumstances, the score is given as 1 with a
modifier attached e.g. "E1c" where "c" = closed, or "V1t" where t = tube. A composite
might be "GCS 5tc". This would mean, for example, eyes closed because of swelling = 1,
intubated = 1, leaving a motor score of 3 for "abnormal flexion". Often the 1 is left out, so
the scale reads Ec or Vt.
 The GCS has limited applicability to children, especially below the age of 36 months
(where the verbal performance of even a healthy child would be expected to be poor).
Consequently the Pediatric Glasgow Coma Scale, a separate yet closely related scale, was
developed for assessing younger children.

4. d) Preoperative assessment of pulmonary function:


Preoperative Pulmonary Function Assessment

The objective is to establish that after surgical resection of the lung for a tumor, there will be
sufficient pulmonary reserve to keep the patient comfortable, and will not become a
respiratory cripple.

You should always evaluate the patient to determine whether he could withstand
pneumonectomy even if radiologically only a lobectomy or limited resection is
contemplated. On thoracotomy, a surgeon may be forced to do pneumonectomy because of
an unexpected node over the pulmonary artery. If you have decided the patient cannot
withstand pneumonectomy, this should be addressed with the surgeon ahead of
thoracotomy.

Step 1: Routine PFTs. If the patient meets the following criteria, no further workup is
necessary:

FEV1 > 2 liters

FEV1/FVC > 50%

MVV > 50% of predicted

RV/TLC <50%

If these criteria were met and the patient were to have pneumonectomy, he would be left
with at least 1 liter of FEV1 in the residual lung.

Step 2: If the patient does not meet the above criteria on routine PFT, and if the FEV1 volume
is less than 2 liter, we need to perform split lung function testing. Lungs with tumor may not
be contributing to total FEV1 volume and thus removal of it may not significantly affect
pulmonary function. On the other hand, in some patients the diseased lung is the best lung.
The best and most current method of estimating split lung function is to perform
quantitative V/Q scan. Perfusion scans correlate better with pulmonary function. One can
calculate the FEV1 volume of left over lung by knowing percentage of perfusion to left and
right lung. For example:

Preoperative FEV1 1.5 liters

Right Lung Perfusion 30%

Left Lung Perfusion 70%

The tumor is in the right lung. Following resection of the right lung, we can estimate 1.5 x .7
= 1.05 liters of the left lung to remain. The minimum acceptable predicted postoperative
FEV1 is 800 ml. If the predicted postoperative FEV1 volume is less than 800 milliliters the
patient is not a candidate for pneumonectomy.
Step 3: If the patient has predicted post-operative FEV1 value is less than 800 ml, and if the
surgeon still feels that he has a resectable lesion with a good prognosis, the next evaluation
would be to occlude the pulmonary artery and measure the pulmonary artery pressure at
rest and with exercise. If the pulmonary artery pressure is elevated at rest or with exercise,
the patient is not a candidate for pneumonectomy. The patient obviously has no capillary
bed reserve and is not able to tolerate the loss of vascular bed. He will develop cor
pulmonale and the expected 5 year survival will be less than 50%. This can also be done on
the operating table by clamping the pulmonary artery and measuring PA pressures.

4. e) Introduction: A dentigerous cyst or follicular cyst is an odontogenic cyst - thought to


be of developmental origin - associated with the crown of an unerupted (or partially
erupted) tooth.

The cyst cavity is lined by epithelial cells derived from the reduced enamel epithelium of the
tooth forming organ. Regarding its pathogenesis, it has been suggested that the pressure
exerted by an erupting tooth on the follicle may obstruct venous flow inducing accumulation
of exudate between the reduced enamel epithelium and the tooth crown.

Histology:
 Histologically a normal dental follicle is lined by enamel epithelium, whereas a
dentigerous cyst is lined by non-keratinized stratified squamous epithelium.
 Since the dentigerous cyst develops from follicular epithelium it has more potential for
growth, differentiation and degeneration than a radicular cyst.
 Occasionally the wall of a dentigerous cyst may give rise to a more ominous
mucoepidermoid carcinoma.
 Due to the tendency for dentigerous cysts to expand rapidly, they may cause pathological
fractures of jaw bones.
 On Fine needle aspiration, thin straw colored fluid is seen.
Radiology:
 The usual radiographic appearance is that of a well-demarcated radiolucent lesion
attached at an acute angle to the cervical area of an unerupted tooth.
 The border of the lesion may be radiopaque.
 Radiographic differentiation between a dentigerous cyst and a normal dental follicle is
based merely on size.
 Radiographically, a dentigerous cyst should always be differentiated from a normal
dental follicle.
 Dentigerous cysts are the most common cysts with this radiographic appearance.
 Radiographically the cyst appears unilocular with well defined margins and often
sclerotic borders but sometimes it may be multilocular in appearance and may also have
a continuous cystic membrane.
 Infected cysts show ill-defined margins.Follicular space more than 5mm is to be
considered a dentigerous cyst.
 Radiographically there are three types of dentigerous cyst, namely the Central type,
Lateral type and the Circumferential type.
Location: The most common location of dentigerous cysts are the Mandibular 3rd Molars
and the Maxillary Canines, and they rarely involve deciduous teeth and are occasionally
associated with odontomes.
Treatment:
 A dentigerous cyst is often treated by excision of the cyst along with the extraction of
the assosciated tooth.
 In case of a large cyst marsupialization is done.
The West Bengal University of Health Sciences

M.B.B.S. 3rd Professional Part – II (Supplementary) Examination, 2017

Subject: Surgery Time: 21/2 hrs.

Paper: II Marks: 60

Group –A

1. Discuss the pathology of salivary gland tumours and management of pleomorphic salivary
adenoma. 5+10
Answer. Surgical pathology of salivary gland tumours.

Benign Malignant
 Pleomorphic adenoma  Acinic cell carcinoma  Salivary duct carcinoma
 Warthin's tumor  Adenocarcinoma
 Mucoepidermoid carcinoma
 Capillary hemangioma  Myoepithelial carcinoma
 Adenoid cystic carcinoma
 Oncocytoma  Malignant mixed tumor
 Polymorphous low-grade
 Basal cell adenoma adenocarcinoma  Squamous cell carcinoma
 Canalicular adenoma  Epithelial-myoepithelial carcinoma  Small cell carcinoma
 Myoepithelioma  Basal cell adenocarcinoma  Lymphoma
 Sialadenoma  Sebaceous carcinoma  Metastatic carcinoma
papilliferum
 Papillary cystadenocarcinoma  Carcinoma ex pleomorphic
 Intraductal papilloma adenoma
 Mucinous adenocarcinoma
 Inverted ductal
papilloma  Oncocytic carcinoma

Pleomorphic adenoma:

Morphology: Most pleomorphic adenomas present as rounded, well-demarcated masses


rarely exceeding 6 cm in greatest dimension.Although they are encapsulated, in some
locations (particularly the palate) the capsule is not fully developed, and expansile growth
produces tonguelike protrusions.

Gross pathology Histology

 Smooth  Mixture of epithelial, myopeithelial


 Well-demarcated and stromal components
 Solid  Epithelial cells: nests, sheets, ducts,
 Cystic changes trabeculae
 Stroma: myxoid, chrondroid, fibroid,
osteoid
 No true capsule
 Tumor pseudopods
Warthin’s Tumor:

Gross pathology Histology

 Encapsulated  Papillary projections into cystic spaces


 Smooth/lobulated surface surrounded by lymphoid stroma
 Cystic spaces of variable size, with  Epithelium: double cell layer
viscous fluid, shaggy epithelium  Luminal cells
 Solid areas with white nodules  Basal cells
representing lymphoid follicles  Stroma: mature lymphoid follicles with
germinal centers

Oncocytoma:

Gross pathology Histology

 Encapsulated  Cords of uniform cells and thin fibrous stroma


 Homogeneous, smooth  Large polyhedral cells
 Orange/rust color  Distinct cell membrane
 Granular, eosinophilic cytoplasm
 Central, round, vesicular nucleus

Mucoepidermoid Carcinoma

Gross pathology Histology

 Well-circumscribed to partially encapsulated  Low-grade


to unencapsulated  Mucus cell > epidermoid cells
 Solid tumor with cystic spaces  Prominent cysts
 Mature cellular elements

Management of pleomorphic adenoma:

Investigations:

FNAC – Important and diagnostic

Open biopsy – contraindicated Due to -

 Chance of injury to facial nerve,


 Seedling & high chance of recurrence,
 Parotid fistula formation
CT SCAN – Best imaging modality.

 Smoothly marginated or lobulated homogeneous small spherical mass is the most


common appearance. When larger they can be heterogeneous with foci of necrosis. Small
regions of calcification are common.
 When the tumour is small, the enhancement tends to be prominent. In larger tumours
enhancement is less marked but can demonstrate delayed enhancement

MRI: They are commonly seen as well-circumscribed and homogeneous when small. Larger
tumours may be heterogeneous.

 T1: usually of low intensity


 T2
o usually of very high intensity (especially myxoid type)
o often have a rim of decreased signal intensity on T2-weighted images
representing the surrounding fibrous capsule
 T1 C+ (Gd): usually demonstrates homogeneous enhancement

Treatment:

• Tumor is radio resistant

o Surgery :
 Enucleation –avoided. High recurrence.
 Treatment of choice: Superficial parotidectomy –Patey’s operation( if superficial lobe
alone is involved)
 Total conservative parotidectomy (If both lobes involved)
 Facial nerve is preserved.

Group – B

2. Answer any one of the following:


a) 20 year old male presenting with right testicular mass-how will you proceed to investigate
and manage the case? 7+8

Answer. Most probably the patient is suffering from nonseminomatous germ cell tumour.

Risk factors include a history of testicular cancer in the contralateral testis, cryptorchidism
(undescended testis), gonadal dysgenesis, prenatal exposure to high estradiol levels, exposure to
chemical carcinogens, trauma, and orchitis. Other risk factors may include childhood inguinal
hernias and any cause of testicular atrophy.

They tend to be more aggressive than seminomas, and frequently metastasise.

Serological markers:

Clinical examination:
 Testicular Cancer presents as a painless, unilateral testicular scrotal mass, as a casual US
finding or revealed by a scrotal trauma. Scrotal pain may be the first symptom in 20% of
cases.
 Gynaecomastia appears in 7% of cases (more common in non-seminomatous tumours).
 Back and flank pain due to metastasis is present in about 11% of cases.
 Ultrasound (US) must be performed in any doubtful case. A correct diagnosis must be
established in all patients with an intrascrotal mass.
 US serves to confirm the presence of a testicular mass and to explore the contralateral
testis. Its sensitivity is almost 100%, and it has an important role in determining whether
a mass is intra- or extratesticular.
 US of the testis should be performed in young men with retroperitoneal or visceral
masses and/or elevated serum human chorionic gonadotrophin (hCG) or alpha-
fetoprotein (AFP) and/or consulting for fertility problems and without a palpable
testicular mass.
 CECT Abdomen and thorax to know the origin and extension can be done.
 Magnetic resonance imaging (MRI) of scrotum offers higher sensitivity and specificity
than US in the diagnosis of Testicular Cancer, but its high cost does not justify its routine
use for diagnosis.

Management of non-seminomatous germ cell tumours (NSGCT):


Following radical orchidectomy and formal staging, the patient is normally managed by
retroperitoneal lymph node dissection (RPLND) in selected cases.
In the presence of elevated AFP, a seminoma would be managed as for teratoma. Combination
chemotherapy, introduced in the 1980s, revolutionized the treatment of metastatic testicular
teratoma, which was hitherto virtually untreatable.
Treatment and follow-up:
Non-metastatic disease
T1-4N0M0S0: surveillance or chemotherapy (bleomycin, low-dose etoposide, cisplatin — 2
cycles) depending on risk factors for relapse (lymphatic or vascular invasion, T2-4);
surveillance in presence of risk factors results in 25% relapse rate, most <1 year post
orchidectomy.
Metastatic disease
Good prognosis: chemotherapy (bleomycin, etoposide, cisplatin × 3 cycles); RPLND for
residual or recurrent mass; salvage chemotherapy if histology confirms tumour.
Intermediate and poor prognosis: chemotherapy (bleomycin, etoposide, cisplatin A— 4
cycles); RPLND for residual or recurrent mass; salvage chemotherapy if histology confirms
tumour.

Approximately 70% of NSGCTs produce hormonal markers (tumour markers):

 alpha-fetoprotein (AFP): typically elevated in yolk sac tumour


 beta-human chorionic gonadotropin (B-hCG): typically elevated in choriocarcinoma
 lactate dehydrogenase (LDH)

Pathology: Non-seminomatous germ cell tumours can be divided histologically into:

 embryonal cell carcinoma: rare


 choriocarcinoma: rare (carries worst prognosis)
 yolk sac tumour
 teratoma: accounts for 5-10% of germ cell tumours
o mature
o immature
o teratoma with malignant transformation
 mixed germ cell tumour
o is the most common type of NSGCT
o accounts for 40% of all germ cell tumours
o the most common combination is teratoma and embryonal cell carcinoma

Treatment and prognosis: The prognosis of non-seminomatous germ cell tumours is variable
and depends on the biological behaviour of individual tumours. Prognosis can be inferred
from tumour markers, the location of the primary tumour and of course presence of
metastases.

 Good prognosis (all of the following must be true)


o tumour markers
 alpha-fetoprotein <1000 ng/mL
 beta HCG <5000 IU/L
 LDH <1.5x upper limit of normal
o non-mediastinal location of the primary tumour
o no nonpulmonary metastases

These patients are usually treated with orchiectomy followed by chemotherapy


(Bleomycin, Etoposide and Cisplatin - BEP regimen).

 Intermediate prognosis (all of the following must be true)


o tumour markers
 alpha-fetoprotein: 1000-10,000 ng/mL
 beta-hCG: 5000-50,000 IU/L
 LDH: 1.5-10x upper limit of normal
o non-mediastinal location of the primary tumour
o no nonpulmonary metastases

These patients are started with chemotherapy cycles and sometimes followed by
adjuvant surgery.

 Poor prognosis (any of the following):


o tumour markers
 alpha-fetoprotein: >10,000 ng/mL
 beta-hCG: >50,000 IU/L
 LDH: >10x upper limit of normal
o mediastinal primary tumour
o nonpulmonary metastases present

b) A 30 year old adult complaining of colicky pain from right loin to groin and vomiting – how
will you proceed to investigate and manage this case? 7+8

Answer. The patient is suffering from right ureteric colic.

USG whole abdomen should be done to rule out other diseases like acute appendicitis.

Urine for R/E and C/S examination.

Blood for complete hemogram, urea and creatinine.

Ureteric stones: diagnostic radiological imaging

The intravenous urogram (IVU), for many years the mainstay of imaging in patients with flank
pain, has been replaced by CT urography (CTU .

Compared with IVU, CTU:


 Has greater specificity (95%) and sensitivity (97%) for diagnosing ureteric stones it
can identify other, non-stone causes of flank pain.
 Requires no contrast administration so avoiding the chance of a contrast reaction (risk
of fatal anaphylaxis following the administration of low-osmolality contrast media for
IVU is in the order of 1 in 100,000).
 Is faster, taking just a few minutes to image the kidneys and ureters. An IVU,
particularly where delayed films are required to identify a stone causing high-grade
obstruction, may take hours to identify the precise location of the obstructing stone.
 Is equivalent in cost to IVU, in hospitals where high volumes of CT scans are done.

If you only have access to IVU, remember that it is contraindicated in patients with a history
of previous contrast reactions and should be avoided in those with hay fever, a strong history
of allergies, or asthma who have not been pre-treated with high-dose steroids 24h before the
IVU. Patients taking metformin for diabetes should stop this for 48h prior to an IVU. Clearly,
being able to perform an alternative test, such as CTU in such patients, is very useful.

Plain abdominal X-ray and renal ultrasound are not sufficiently sensitive or specific for their
routine use for diagnosing ureteric stones.

MR urography: This is a very accurate way of determining whether a stone is present in the
ureter or not.

Ureteric stones: acute management


While appropriate imaging studies are being organized, pain relief should be given.

 A non-steroidal anti-inflammatory (e.g. diclofenac) by intramuscular or intravenous


injection, by mouth or per rectum. Provides rapid and effective pain control. Where
NSAIDS are inadequate, opiate analgesics such as pethidine or morphine are added
 Calcium channel antagonists (e.g. nifedipine) may reduce the pain of ureteric colic by
reducing the frequency of ureteric contractions.

Renal blood flow and urine output from the affected kidney falls during an episode of acute,
partial obstruction due to a stone.
Excess urine output will tend to cause a greater degree of hydronephrosis in the affected
kidney which will make ureteric peristalsis even less efficient than it already is.

Watchful waiting:
 In many instances, small ureteric stones will pass spontaneously within days or a few
weeks, with analgesic supplements for exacerbations of pain.
 Chances of spontaneous stone passage depend principally on stone size. Between 90 -
98% of stones measuring <4mm will pass spontaneously.
 Average time for spontaneous stone passage for stones 4 - 6mm in diameter is 3 weeks.
Stones that have not passed in 2 months are unlikely to do so. Therefore, accurate
determination of stone size (on plain abdominal X-ray or by CTU) helps predict chances of
spontaneous stone passage.
 Nifedipine and tamsulosin (an alpha adrenergic adrenoceptor blocking drug) may assist
spontaneous stone passage and reduce frequency of ureteric colic.

Ureteric stones: indications for intervention to relieve obstruction and/or remove the stone

 Pain which fails to respond to analgesics or recurs and cannot be controlled with
additional pain relief.
 Fever. Have a low threshold for draining the kidney (usually done by percutaneous
nephrostomy).
 Impaired renal function (solitary kidney obstructed by a stone, bilateral ureteric
stones, or pre-existing renal impairment which gets worse as a consequence of a
ureteric stone). Threshold for intervention is lower.
 Prolonged unrelieved obstruction. This can result in long-term loss of renal function .
 Social reasons. Young, active patients may be very keen to opt for surgical treatment
because they need to get back to work or their childcare duties, whereas some patients
will be happy to sit things out. Airline pilots and some other professions are unable to
work until they are stone free.

Emergency temporizing and definitive treatment of the stone:


Where the pain of a ureteric stone fails to respond to analgesics or where renal function is
impaired because of the stone, then temporary relief of the obstruction can be obtained by
insertion of a JJ stent or percutaneous nephrostomy tube. (Percutaneous nephrostomy tube
can restore efficient peristalsis by restoring the ability of the ureteric wall to coapt.)

The patient may elect to proceed to definitive stone treatment by immediate ureteroscopy
(for stones at any location in the ureter) or ESWL (if the stone is in the upper and lower
ureter). ESWL cannot be used for stones in the mid ureter because this region is surrounded
by bone, which prevents penetration of the shock waves).

Treatment options for ureteric stones:

 ESWL: in situ; after push-back into the kidney (i.e. into the renal pelvis or calyces); or
after JJ stent insertion
 Ureteroscopy
 PCNL
 Open ureterolithotomy
 Laparoscopic ureterolithotomy

Basketing of stones (blind or under radiographic control) are historical treatments (the
potential for serious ureteric injury is significant).
The ureter can be divided into two halves (proximal and distal to the iliac vessels) or in thirds
(upper third from the PUJ to the upper edge of the sacrum; middle third from the upper to the
lower edge of the sacrum; lower third from the lower edge of the sacrum to the VUJ).

Recommendations
Proximal ureteric stones

 <1cm diameter: ESWL (in situ, push-back)


 >1cm diameter: ESWL, ureteroscopy, PCNL

JJ stent insertion does not increase stone free rates and is therefore not required in routine
cases. Indicated for pain relief, relief of obstruction, and in those with solitary kidneys.
Distal ureteric stones

 Both ESWL and ureteroscopy are acceptable options.


 Stone free rate <1cm: 80 - 90% for both ESWL and ureteroscopy; >1cm: 75% for both
ESWL and ureteroscopy.

Group – C
3. Answer in brief on any three of the following: 3x5
a) Thyroglossal cyst.
b) MRCP.
c) Complications of Radiotherapy.
d) Imperforate anus.
e) Flail chest.

Answer.

a) Definition: Thyroglossal cyst is a fluid-filled sac resulting from incomplete closure of the
thyroglossal duct.
Anatomy: The thyroglossal duct arises embryologically between the first and second
pharyngeal pouches. It runs as a hollow tube from the foramen caecum on the dorsal surface
of the tongue, becoming a solid cord of cells migrating through the tongue and into the
midline of the neck. The tract usually passes in front of the hyoid bone and then loops up
behind it before descending in the midline of the neck where the cells divide to form the two
lobes of the thyroid gland either side of the midline. The duct normally atrophies in the sixth
week of gestation.
Clinical features:
 Usually presents in children or young adults.
 75% appear in front of the hyoid bone and the majority of the rest at the level of the
thyroid or cricoid cartilage of the larynx.
 5% become infected presenting as a painful, red neck swelling.
 90% present as a painless midline cyst.
 The cyst is mobile and moves up on swallowing and protrusion of tongue.
 15% have a fistula to the skin (due to infection or incomplete excision).
 10% appear on one side of the midline, usually the left.
 If large enough it will transilluminate.
 Papillary carcinoma of the thyroglossal ductal cells is rare. Treatment is by excision.
Diagnosis and investigations:
CT scan - often reveal a well circumscribed cyst related to the midline of the hyoid bone.
Fine-needle aspiration may reveal a cloudy infected fluid or a straw-coloured fluid.
Treatment:
Infected thyroglossal cyst:
 Majority respond to antibiotics.
 Surgical drainage if abscess formed or failure to respond to antibiotics.
 Elective excision of the cyst once acute infection has resolved.
Surgery
 Excision is recommended for most cysts.
 Remove through a transverse midline incision in a skin crease.
 Divide the platysma muscle and dissect the cyst out bluntly.
 On the deep surface it will be found to be attached to the hyoid bone: excise
approximately 1cm of the bone, removing any underlying thyroglossal duct
epithelium.
 Close the wound in layers with a suction drain.
 If there is a fistula or sinus in the neck excise it through a transverse elliptical incision.
Again use blunt dissection and remove the middle part of the hyoid bone (Sistruck
procedure).

b) Magnetic resonance cholangiopancreatography (MRCP) is a non-invasive imaging


technique to visualize intra and extrahepatic biliary tree and pancreatic ductal system.

It can provide the diagnostic range equivalent to the ERCP and so it can replace the ERCP in
high risk patient to avoid significant morbidity.
Indications: MRCP can be used to evaluate various conditions of pancreatobiliary ductal
system, some of them are:

 Identification of congenital anomalies of the cystic and hepatic ducts


 Post-surgical biliary anatomy and complications
 Pancreas divisum
 Anomalous pancreaticobiliary junction
 Choledocholithiasis
 Biliary strictures
 Chronic pancreatitis
 Pancreatic cystic lesions
 Trauma to biliary system

Physics

The technique exploits the fluid which is present in the biliary and pancreatic ducts as a
contrast agent by acquiring the images using heavily T2 weighted sequences. Since the fluid-
filled structures in the abdomen have a long T2 relaxation time as compared to the
surrounding soft tissue, these structures appear hyperintense than the surrounding on a
heavily T2 weighted sequence and can easily be distinguished.

Technique and protocols

No contrast is administered within the body.

Fasting for 4 hours prior to the examination is required to reduce gastroduodenal secretions,
reduce motility to eliminate motion artifacts and to promote distension of gall bladder. MRCP
is performed on a 1.5-T or superior MRI system, using a phased-array body coil.

All protocols obtain heavily T2 weighted sequences. Most commonly obtained sequences are:

 RARE: rapid acquisition and relaxation enhancement


 FRFSE: fast-recovery fast spin-echo coronal oblique 3D respiratory triggered
 HASTE: half-Fourier acquisition single shot turbo spin echo- Axial 2D breath hold
sequence which provide superior images and can be performed in single breath hold
(<20 sec) and a fat suppressed sequence
 additional sequence which can be acquired to evaluate duct wall is fat suppressed T1
GRE sequence

For optimum visualization of ducts, acquired images are reformatted in different planes using
multiplanar reconstruction (MPR) and maximum intensity projection (MIP).

The advantage of FRFSE, as a 3D technique, is the ability to perform multiplanar


reconstructions. However, despite respiratory triggering, this sequence is often prone to
motion artifact.

Technical modifications: With the evolution of MRCP, modified techniques came into
existence. Commonly applied modified MRCP techniques are:

 secretin stimulated MRCP


 functional MRCP
 negative oral contrast to 'null' the duodenum
o commercially available agents
o natural products such as pineapple juice which is rich in manganese and shortens T2
relaxation time

Practical points: Artifacts related to technique and reconstruction, motion or susceptibility


artifacts due to metal clips and gas may give rise to poor spatial resolution as a result of that
misinterpretation.

c) Complications of radiotherapy:

Acute side effects


 Nausea and vomiting
 Damage to the epithelial surfaces

Epithelial surfaces may sustain damage from radiation therapy. Depending on the area
being treated, this may include the skin, oral mucosa, pharyngeal, bowel mucosa and
ureter. Typically the skin starts to become pink and sore several weeks into treatment.
The reaction may become more severe during the treatment and for up to about one
week following the end of radiation therapy, and the skin may break down. Skin
reactions tend to be worse in areas where there are natural folds in the skin, such as
underneath the female breast, behind the ear, and in the groin.

 Mouth, throat and stomach sores


 Intestinal discomfort: The lower bowel may be treated directly with radiation
(treatment of rectal or anal cancer) or be exposed by radiation therapy to other pelvic
structures (prostate, bladder, female genital tract). Typical symptoms are soreness,
diarrhoea, and nausea.
 Swelling: As part of the general inflammation that occurs, swelling of soft tissues may
cause problems during radiation therapy. This is a concern during treatment of brain
tumors and brain metastases, especially where there is pre-existing raised intracranial
pressure or where the tumor is causing near-total obstruction of a lumen (e.g., trachea
or main bronchus). Surgical intervention may be considered prior to treatment with
radiation. If surgery is deemed unnecessary or inappropriate, the patient may receive
steroids during radiation therapy to reduce swelling.
 Infertility: The gonads (ovaries and testicles) are very sensitive to radiation. They may
be unable to produce gametes following direct exposure to most normal treatment
doses of radiation. Treatment planning for all body sites is designed to minimize, if not
completely exclude dose to the gonads if they are not the primary area of treatment.
Infertility can be efficiently avoided by sparing at least one gonad from radiation.

Late side effects

Late side effects occur months to years after treatment and are generally limited to the area
that has been treated. They are often due to damage of blood vessels and connective tissue
cells. Many late effects are reduced by fractionating treatment into smaller parts.

 Fibrosis: Tissues which have been irradiated tend to become less elastic over time due to
a diffuse scarring process.
 Epilation: Epilation (hair loss) may occur on any hair bearing skin with doses above 1 Gy.
It only occurs within the radiation field/s. Hair loss may be permanent with a single dose
of 10 Gy, but if the dose is fractionated permanent hair loss may not occur until dose
exceeds 45 Gy.
 Dryness: The salivary glands and tear glands have a radiation tolerance of about 30 Gy in
2 Gy fractions, a dose which is exceeded by most radical head and neck cancer treatments.
Dry mouth (xerostomia) and dry eyes (xerophthalmia) can become irritating long-term
problems and severely reduce the patient's quality of life. Similarly, sweat glands in
treated skin (such as the armpit) tend to stop working, and the naturally moist vaginal
mucosa is often dry following pelvic irradiation.
 Lymphedema
 Cancer: Radiation is a potential cause of cancer, and secondary malignancies are seen in a
very small minority of patients.
 Heart disease: Radiation has potentially excess risk of death from heart disease seen after
some past breast cancer RT regimens.
 Cognitive decline
 Radiation enteropathy
 Cumulative side effects: Cumulative effects from this process should not be confused with
long-term effects—when short-term effects have disappeared and long-term effects are
subclinical, reirradiation can still be problematic. These doses are calculated by the
radiation oncologist and many factors are taken into account before the subsequent
radiation takes place.

 Effects on reproduction: During the first two weeks after fertilization, radiation therapy is
lethal but not teratogenic. High doses of radiation during pregnancy induce anomalies,
impaired growth and intellectual disability, and there may be an increased risk of
childhood leukemia and other tumours in the offspring.

In males previously having undergone radiotherapy, there appears to be no increase in


genetic defects or congenital malformations in their children conceived after therapy.
However, the use of assisted reproductive technologies and micromanipulation
techniques might increase this risk.

 Effects on pituitary system: Hypopituitarism commonly develops after radiation therapy


for sellar and parasellar neoplasms, extrasellar brain tumours, head and neck tumours,
and following whole body irradiation for systemic malignancies.

d) Wingspread anatomical classification of Anorectal malformations (1984)


Associated Anomalies:

 Sacrum and Spine


Sacral deformities appear to be the most frequently associated defect.
 Genitourinary Defects
The frequency of associated genitourinary defects varies from 20% to 54%.
 Anal atresia may occur as a part of the VACTERL group of anomalies
V Vertebral body segmentation defect
A Anal atresia
C Cardiovascular (PDA, VSD)
TE Tracheo esophagial fistula
R unilateral Renal agenesis
L Limb anomaly (radial ray hypoplasia)
 CVS: Tetrology of Fallot or VSD
 GI: Tracheo-esophageal abnormality , Duodenal atresia , Hirschsprung’s .

Algorithm for management of ARM in newborn male

Posterior Sagittal Ano-RectoPlasty – PSARP.

PSARP involves:
 Stimulation of muscles to demonstrate the midline and sphincter
 Posterior sagittal incision - length depends on severity of abnormality and required
extent of dissection
 Rectum identified. Abdominal approach may be required in addition in 10% of males
and 40% of cloacae
Rectum dissected.
 Separation from genitourinary tract - often the most difficult part
 Repositioning the neoanus within the sphincteric mechanism

Algorithm for management of ARM in newborn female

e) Flail chest:
Introduction: Flail chest is a life-threatening medical condition that occurs when a segment of
the rib cage breaks under extreme stress and becomes detached from the rest of the chest
wall. It occurs when multiple adjacent ribs are broken in multiple places, separating a
segment, so a part of the chest wall moves independently.

Causes:
 The most common reason for flail chest injuries are vehicle accidents.
 Another main cause of flail chest injuries results from falling which is mainly elderly
related.
 In children, the majority of flail chest injuries can be a result of the common blunt
force traumas or metabolic bone diseases, one known as osteogenesis imperfecta.

Clinical features:
 Two of the symptoms of flail chest are chest pain and dyspnea.
 The characteristic paradoxical motion of the flail segment occurs due to pressure
changes associated with respiration that the rib cage normally resists.
 Results in paradoxical motion of the chest wall. Hypoxia is caused by restricted chest
wall movement and underlying lung contusion.

 During normal inspiration, the diaphragm contracts and intercostal muscles pull the rib
cage out. Pressure in the thorax decreases below atmospheric pressure, and air rushes in
through the trachea. The flail segment will be pulled in with the decrease in pressure
while the rest of the rib cage expands.
 During normal expiration, the diaphragm and intercostal muscles relax increasing
internal pressure, allowing the abdominal organs to push air upwards and out of the
thorax. However, a flail segment will also be pushed out while the rest of the rib cage
contracts.

Treatment: Treatment of the flail chest initially follows the principles of advanced trauma life
support. Further treatment includes:

 Good analgesia including intercostal blocks, avoiding narcotic analgesics as much as


possible. This allows much better ventilation, with improved tidal volume, and increased
blood oxygenation.
 If the segment is small and respiration is not compromised, nurse patient in HDU with
adequate analgesia. Encourage early ambulation and vigorous physiotherapy. Regular
blood gas analysis should be done.
 Positive pressure ventilation, meticulously adjusting the ventilator settings to
avoid pulmonary barotrauma.
 Chest tubes as required.
 Adjustment of position to make the patient most comfortable and provide relief of pain.
 Aggressive pulmonary toilet.
 In more severe cases, endotracheal intubation with positive-pressure ventilation is
required.
 Surgical fixation can help in significantly reducing the duration of ventilatory support and
in conserving the pulmonary function.

Group – D

4. Write short notes on any three of the following: 3x5


a) Glasgow coma scale.
b) IVU.
c) Marjolin’s ulcer.
d) Hypospadias.
e) Ingrowing toe nail.

Answer. The Glasgow Coma Scale or GCS is a neurological scale that aims to give a reliable,
objective way of recording the conscious state of a person for initial as well as subsequent
assessment. A patient is assessed against the criteria of the scale, and the resulting points give a
patient score between 3 (indicating deep unconsciousness) and either 14 (original scale) or 15
(the more widely used modified or revised scale).
GCS was initially used to assess level of consciousness after head injury, and the scale is now
used by first aid, EMS, nurses and doctors as being applicable to all acute medical and trauma
patients.
In hospitals it is also used in monitoring chronic patients in intensive care.

Glasgow coma scale (GCS)


Elements of the scale
 Eye response (E)
 Verbal response (V)
 Motor response (M)

Minimum score, 3; maximum score, 15.

Generally, brain injury is classified as:


 Severe, with GCS < 8–9
 Moderate, GCS 8 or 9–12 (controversial)
 Minor, GCS ≥ 13.

o Generally when a patient is in a decline of their GCS score, the nurse or medical staff
should assess the cranial nerves and determine which of the twelve have been affected.
o Tracheal intubation and severe facial/eye swelling or damage make it impossible to test
the verbal and eye responses. In these circumstances, the score is given as 1 with a
modifier attached e.g. "E1c" where "c" = closed, or "V1t" where t = tube. A composite might
be "GCS 5tc". This would mean, for example, eyes closed because of swelling = 1, intubated
= 1, leaving a motor score of 3 for "abnormal flexion". Often the 1 is left out, so the scale
reads Ec or Vt.
o The GCS has limited applicability to children, especially below the age of 36 months
(where the verbal performance of even a healthy child would be expected to be poor).
Consequently the Pediatric Glasgow Coma Scale, a separate yet closely related scale, was
developed for assessing younger children.

b) Intravenous urography (IVU), also referred as intravenous pyelography (IVP) or excretory


urography (EU), is a radiographic study of the renal parenchyma, pelvicalyceal system,
ureters and the urinary bladder. This exam has been largely replaced by CT urography.

Terminology

Some prefer the term "urogram" to refer to visualization of the kidney parenchyma, calyces,
and pelvis after intravenous injection of contrast, and reserve the term "pyelogram" to
retrograde studies involving the collecting system. In practice, both terms are often used
interchangeably.

Procedure

Indications

 check for normal function of kidneys


 check for anatomical variants or congenital anomalies (e.g.horse-shoe kidney)
 check the course of the ureters
 detect and localise a ureteric obstruction (urolithiasis)
 assess for synchronous upper tract disease in those with bladder transitional cell
carcinoma (TCC)

Patient preparation

 overnight fasting prior to the date of examination; a laxative would help to achieve a
good preparation
 on the day of procedure take a scout/pilot film to check patient preparation and also
for radio-opaque calculi
 check serum creatinine level to be within normal range (as per hospital guidelines)
 take a history of the patient for any known drug allergies followed by a written
informed consent for the procedure

Technique

Exposures are generally in the 65-75 kV range, mA of 600-1000, with exposure of <0.1 sec.
Higher kV ranges reduce contrast of the renal parenchyma.

 IV access is required for administration of a water soluble contrast


o nonionic contrast is preferred
 dose will vary as per the weight of the patient; generally up to 1.5 ml/kg body weight is
well tolerated by patient
 the contrast dose is usually instilled at a fast (bolus) rate
 the calyces are usually visualized in <2 minutes following contrast administration -
this is the nephrogram
 serial images are taken at 5-20 minutes for visualisation of the pelvicalyceal systems
and ureters when required and with operator preference
 additional views taken are prone and obliques for ureters
 the full length 10-15 minute film is performed with a compression band applied to the
patient
o compression should not be applied if ureteral calculi, ureteral obstruction,
recent surgery, nephrostomy, or abdominal aortic aneurysm is suspected
 lastly take a full bladder and post void film

There is a wide variation in protocols. One protocol is suggested below, but additional images
should usually be obtained to answer the clinical question:

1. Scout images
2. (1-2 minutes) Nephrogram
3. (>3 minutes) Early and late images of the upper collecting system (abdominal
compression then applied)
4. Tomography may be obtained, if desired
5. (10-15 minutes) Supine, after release of compression, images of the upper collecting
system and proximal ureters
6. (20 minute) Supine image
7. (20 minute) Prone image

Emergency medications and emergency equipment must always be available in case the
patient has a reaction to contrast.

c) Introduction: Marjolin's ulcer refers to an aggressive ulcerating squamous cell


carcinomapresenting in an area of previously traumatized, chronically inflamed, or scarred
skin. They are commonly present in the context of chronic wounds including burn
injuries, venous ulcers, ulcers from osteomyelitis, and post radiotherapy scars.

Appearance: Slow growth, painlessness (as the ulcer is usually not associated
with nerve tissue), and absence of lymphatic spread due to local destruction of lymphatic
channels.

Characteristic: Histologically, the tumour is a well-differentiated squamous cell carcinoma.


This carcinoma is aggressive in nature, spreads locally and is associated with a poor
prognosis. 40% occur on the lower limb and the malignant change is usually painless.
This malignant change of the wound happens a long time after initial trauma, usually 10–25
years later. Its edge is everted and not always raised.

Diagnosis: Wedge biopsy is the favored method of diagnosis. Tissue specimens obtained
should be taken from both the centre and margin of lesion, as the central ulcerated deposits
may be necrotic.
Treatment: Treatment is usually surgical, with a wide excision of the lesion; typically a 1 cm
margin all around is required. Radiation therapy is also a good alternative in most cases.

d) Definition:
Hypospadias is a congenital deformity where the opening of the urethra (the meatus) occurs
on the underside (ventral) part of the penis, anywhere from the glans to the perineum.
o It is often associated with hoodedforeskin and chordee (ventral curvature of the penile
shaft).
o It occurs in 1 in 250 live male births.
Classification:
Hypospadias can be classified according to the anatomical location of the urethral meatus
 Anterior (or distal): glandular, coronal, and subcoronal (~50%)
 Middle: distal penile, midshaft, and proximal penile (~30%)
 Posterior (or proximal): penoscrotal, scrotal, and perineal (~20%)
Aetiology:

Hypospadias results from incomplete closure of urethral folds on the underside of the penis
during embryological development. This is related to a defect in production or metabolism of
fetal androgens, or the number and sensitivity of androgen receptors in the tissues. Chordee
are caused by abnormal urethral plate development, and the hoodedforeskin is due to failed
formation of the glandular urethra and fusion of the preputial folds (resulting in a lack of
ventral foreskin but an excess of dorsal tissue).

Diagnosis:

o A full clinical examination will make the diagnosis. However, it is also important to seek
out associated abnormalities which will need treatment (undescended testes, inguinal
hernias, and hydroceles).
o Patients with absent testes and severe hypospadias should undergo chromosomal and
endocrine investigation to exclude intersex conditions.

Treatment:

o Surgery is indicated where deformity is severe, interferes with voiding, or is predicted to


interfere with sexual function.
o Surgery is now performed between 6-12 months of age.
o Local application of testosterone for 1 month pre-operatively can help increase tissue
size.
o Surgery aims to correct penile curvature (orthoplasty), reconstruct a new urethra, and
bring the new meatus to the tip of the glans using urethroplasy, glanuloplasty, and
meatoplasty techniques. Severe cases may require staged procedures.
o Common operations for anterior hypospadias include meatal advancement and
glanuloplasty (MAGPI), meatal-based flaps (Mathieu procedure), and tubularization of the
urethral plate. Posterior defects require free grafts (buccal mucosa), onlay grafts, and
preputial transfer flaps.
Complications:

1.Bleeding, 4.Meatal 7.Urethral


2.Infection, Diverticulum,
3.Urethral 5.Stenosis,
Strictures And
6.Urethrocutaneous
Fistula, 8. Failed procedures
requiring re-operation.

e) Ingrown toenails (unguis incarnatus) are a common toenail problem of uncertain etiology.
Various causes include poorly fit (tight) footwear, infection, improperly trimmed toenails,
trauma, and heredity.

The great toe is the most commonly involved. The lateral side is involved more commonly
than the medial side. The ingrown nail is often diagnosed in school children, adolescents,
young adults, and pregnant women.

Packing, taping, gutter treatment, and nail braces are options for relatively mild cases of
ingrown toenails, whereas surgery is exclusively done by physicians, and phenolization of
the lateral matrix horn is now the safest, simplest, and most commonly performed method
with the lowest recurrence rate.

Nail phenolization is indicated when partial and definitive removal of the nail plate is
necessary. Chemical matricectomy using 10% sodium hydroxide has been shown to be as
efficacious as phenolization.

Prevention: The most common place for ingrown nails is in the big toe, but ingrowth can
occur on any nail. Ingrown nails can be avoided by cutting nails straight across; not along
a curve, not too short, and no shorter than the flesh around it. Footwear that is too small
or too narrow, or with too shallow of a 'toe box', will exacerbate any underlying problem
with a toenail. Sharp square corners may be uncomfortable and cause snagging on socks.
Proper cutting leaves the leading edge of the nail free of the flesh, precluding it from
growing into the toe. Filing of the corner is reasonable. Some nails require cutting of the
corners far back to remove edges that dig into the flesh, this is often done as a partial
wedge resection at a podiatrist's office. Ingrown toe nails can be caused by injury,
commonly blunt trauma where the flesh is pressed against the nail causing a small cut
that swells. Also, injury to the nail can cause it to grow abnormally, making it wider or
thicker than normal or even bulged or crooked.
The West Bengal University of Health Sciences

M.B.B.S. 3rd Professional Part – II (Supplementary) Examination, 2016


Subject: Surgery Time: 21/2 hrs.

Paper: II Marks: 60

Group –A

1. Describe the clinical features and management of primary thyrotoxicosis.


5+10

Answer.
Primary thyrotoxicosis refers to excessive thyroxine (T4) and/or triiodothyronine (T3)
secretion secondary to abnormal thyroid stimulation, thyroid hyperfunctioning nodules,
or ectopic malignant thyroxine secretion. Secondary hyperthyroidism is the result of
abnormal, excessive thyroid stimulating hormone (TSH) release and stimulation of the
thyroid resulting in excessive T4 release. Excessive TSH production may be secondary
to pituitary pathology or hypothalamic pathology resulting in excess thyrotropin
releasing hormone (TRH) production, which will stimulate the pituitary gland to secrete
excessive amounts of TSH. Overtreatment of hypothyroidism, either by physician or
patient abuse, results in iatrogenic disease. The antiarrythmic agent amiodarone may
also cause hyperthyroidism.

Symptoms of hyperthyroidism include heat intolerance, irritability, weight loss,


frequent bowel movements, proximal muscle weakness, tremulousness, anxiety, fever,
and fatigue. Menstruating females often suffer oligomenorrhea.

Physical findings include tachycardia, a widened pulse pressure, eye findings (lid lag,
exophthalmos), goiter, thyroid bruit upon auscultation, a thrill with palpation of the
thyroid, brisk reflexes, pretibial myxedema, and a systolic murmur.

Patients afflicted with this disorder may exhibit electrocardiographic abnormalities to


include sinus tachycardia and atrial fibrillation. In fact, hyperthyroidism should be ruled
out in all patients with new onset atrial fibrillation.

Apathetic hyperthyroidism refers to hyperthyroidism that is asymptomatic except for


cardiac findings and occurs in the elderly. Cardiac symptoms may include congestive
heart failure, which is often refractory to therapy, tachycardia, and a widened pulse
pressure.

Associated lab abnormalities include an extremely depressed TSH level, an elevated free
T4 and/or free T3 which are characteristic of primary hyperthyroidism. Secondary
hyperthyroidism is characterized by elevated levels of TSH, freeT4 and/or free T3
levels. If the free T4 level is normal but symptoms of hyperthyroidism are present along
with a suppressed TSH, T3 toxicosis should be considered and a serum free T3 level
should be determined if not ordered initially. The profile of a low TSH with normal free
T4 and free T3 levels is consistent with subclinical hyperthyroidism. In acutely ill
patients, euthyroid sick syndrome may manifest as a low to low normal TSH, low T3,
and a normal T4 level. Medications associated with low TSH levels include
glucocorticoids, octreotide, dopamine, dobutamine, and amiodarone.
Hypercalcemia may result from hyperthyroidism-associated bone disease, and
osteoporosis is a potential long-term complication if effective therapy is not
administered. Hyperthyroid-induced hepatosteatosis may result in elevation of liver
function tests to include the alkaline phosphatase, ALT and AST values.

Primary thyrotoxicosis may be secondary to one of several different pathologies, which


are listed in order of decreasing frequency:
Grave’s disease: This is the most common form of hyperthyroidism and is
characterized by the triad of exophthalmos, diffuse goiter, and hyperthyroidism. This is
an autoimmune disorder, and diagnosis is confirmed by finding the above triad plus the
presence of serum antibodies (TSH receptor antibodies composed of thyroid-
stimulating immunoglobulin and the TSH receptor-blocking antibodies) or the
characteristic elevated diffuse uptake of iodine 123 on uptake testing. The presence of
pretibial myxedema is also indicative of Grave s disease as the underlying cause of
hyperthyroidism.

Toxic nodular goiter (toxic adenoma and toxic multinodular goiter): This form of
hyperthyroidism may be the result of a single or multiple autonomously
hyperfunctioning nodules. This disorder usually occurs in thyroid glands that have been
enlarged for prolonged periods of time. These nodules are benign. When patients with
this condition are exposed to iodine (radiocontrast, amiodarone), they may suffer
clinical worsening of their disease (jodbasedow effect).
Thyroiditis: These are transient forms of hyperthyroidism (subacute ,painless and
postpartum). Subacute thyroiditis is a painful condition that often follows a respiratory
illness and is secondary to thyroid follicle disruption with release of stored thyroid
hormone. Postpartum disease is generally painless and lasts approximately one to three
months. Because this disorder is transient, antithyroid agents (methimazole or
propylthiouracil) are unnecessary and are ineffective. Aspirin 650 mg PO QID may
relieve the pain, and if aspirin is ineffective, prednisone 20-40 mg PO QD may offer
relief. Symptoms of hyperthyroidism may be controlled with propranolol 20-40 mg PO
QID, but the medication should be discontinued after a few weeks and the patient
monitored for signs of hyperthyroidism.

Thyrotoxicosis Factitia: This is hyperthyroidism that occurs secondary to ingestion of


exogenous thyroxine preparations. In persons ingesting exogenous thyroxine, the
serum thyroglobulin level will be extremely low.
Trophoblastic tumors (rare): Hyperthyroidism results when trophoblastic tumors
secrete excessive amounts of beta hCG greater than 100,00 mIU/mL. Hyperthyroidism
occurs because beta hCG has a similar alpha subunit to that of TSH and binds to TSH
receptors on thyroid follicular cells.
Struma Ovarii (rare): This uncommon disorder occurs when thyroid rests within the
ovary become hyperplastic. This disorder is characterized by a low TSH, an elevated T4,
low thyroid radioisotope uptake on thyroid scanning, and detection of pelvic
radioisotope uptake when the gamma scanner is directed at the pelvis during thyroid
scanning.

Thyroid radioiodine uptake may be helpful in establishing the cause of


hyperthyroidism. Radioiodine uptake will be elevated in Grave s disease (homogeneous
uptake of 35-100%), toxic nodular goiter (heterogeneous uptake 20-60%), toxic
adenoma (increased uptake of 20-60% in area of nodule). Conversely, the uptake of
radioiodine will be low in cases of subacute thyroiditis, postpartum thyroiditis, struma
ovarii, and overdosing of exogenous thyroid hormone (iatrogenic or factitious
hyperthyroidism). Radioiodine uptake testing should not be performed within 8 weeks
of the patient receiving intravenous iodinated contrast medium for other radiographic
testing as the prior iodine-containing contrast will saturate the thyroid and prevent
uptake of the iodine 123 even if the thyroid is hyperactive. Pregnancy is a definite
contraindication to iodine 123 uptake testing.

Thyroid storm is a life-threatening form of hyperthyroidism, and this disorder is


characterized by signs and symptoms or a history of hyperthyroidism plus fever and a
change in mental status. Therapy includes propylthiouracil (150 mg PO STAT followed
by 100-150 mg PO TID) and iodides to block thyroid hormone release (SSKI 10 drops
QD or sodium iodide 0. 5 grams by slow intravenous drip Q 6-8 hours). Iodine should
only be administered 1-2 hours after thyroid-blocking agents (propylthiouracil) have
been given. Decadron (4 mg intravenously Q 6 hours) reduces serum T3 levels. Beta
blockade with propranolol (30-60 mg PO Q 8 hours) or intravenous esmolol will help
control heart rate. Supportive measures include fluids and determining the cause
(infection, trauma). Fever should not be treated with cooling mattresses, and Tylenol is
ineffective. Fever may be controlled with Demerol (25-50 mg intravenously Q 6 hours)
and chlorpromazine (25-50 mg intravenously Q 6 hours).
Therapy for hyperthyroidism depends on the etiology. Commonly used therapies for
Grave s disease include radioablation or medical therapy with propylthiouracil (50-150
mg PO TID/QID) or methimazole (5-20 mg PO BID/TID) plus beta blockade for
symptom control. Therapy may be continued for a period of approximately 6 months to
two years and then discontinued to see if remission has occurred. If hyperthyroidism
persists, therapy may be continued indefinitely or radioablation of the thyroid may be
offered to the patient. Patients should be monitored post ablation for the development
of a hypothyroid state. Thyroidectomy is a uncommon form of therapy in appropriate
cases.

Group – B

2. Answer any one of the following:


a) Classify adrenal tumours. Describe the investigation and treatment of Adrenal
incidentaloma. 5+10
Answer. WHO histological classification of adrenal gland tumours

Adrenal cortical tumours

 Adrenal cortical carcinoma


 Adrenal cortical adenoma
Adrenal medullary tumours

 Malignant phaeochromocytoma
 Benign phaeochromocytoma
 Composite phaeochromocytoma/paraganglioma

Extra-adrenal paraganglioma

 Carotid body
 Jugulotympanic
 Vagal
 Laryngeal
 Aortico-pulmonary
 Gangliocytic
 Cauda equine
 Orbital Nasopharyngeal
Extra-adrenal sympathetic paraganglioma

 Superior and inferior para-aortic paraganglioma


 Urinary bladder
 Intrathoracic and cervical paravertebral
Other adrenal tumours

 Adenomatoid tumour
 Sex-cord stromal tumour
 Soft tissue and germ cell tumours
 Myelolipoma
 Teratoma
 Schwannoma
 Ganglioneuroma
 Angiosarcoma
Secondary tumours
Algorithm for the management of an adrenal incidentaloma. Adrenalectomy is
recommended for all patients with functional tumors. For nonfunctioning tumors, the risk
for malignancy is assessed according to size. Tumors larger than 5 cm on CT carry a >25%
risk for malignancy and need to be removed. Those <3 cm can be safely observed.

Case-specific factors must be considered for intermediate sized tumors. PAC, Plasma
aldosterone concentration, in ng/ dL; PRA, plasma renin activity, in ng/ mL • hr .
b) Enumerate the causes of retention of urine in different age groups. How will you
investigate a case of retention of urine? How will you treat retention of urine?
5+5+5
Answer.

Causes of retention of urine in different age group:


In children:

Lower urinary tract Urinary tract Constipation Urethral foreign body


stones infection
Neurologic problems Ureterocele Imperforated hymen Urethral cyst

Trauma Benign obstructive Prostatic utricle Prune Belly


lesions Syndrome
Local inflammation Iatrogenic Hinman syndrome Rhabdomyosarcoma

See the answer of Question 1.b of Group – A of Supplementary Paper – II of 2012.


Treatment:

A thorough history, physical examination, and selected diagnostic testing should determine
the cause of urinary retention in most cases. Initial management includes bladder
catheterization with prompt and complete decompression. Men with acute urinary
retention from benign prostatic hyperplasia have an increased chance of returning to
normal voiding if alpha blockers are started at the time of catheter insertion. Suprapubic
catheterization may be superior to urethral catheterization for short-term management
and silver alloy-impregnated urethral catheters have been shown to reduce urinary tract
infection. Patients with chronic urinary retention from neurogenic bladder should be able
to manage their condition with clean, intermittent self-catheterization; low-friction
catheters have shown benefit in these patients. Definitive management of urinary retention
will depend on the etiology and may include surgical and medical treatments.

Group – C

3. Write short notes on (any three) of the following: 3x5


a) Different types of nerve injuries.
b) Chest drain after chest injury.
c) Regional anaesthesia.
d) Fournier’s gangrene.
e) Undescended testis.
Answer.

a) Different types of nerve injuries:


Neuropraxia - physiologic block of nerve conduction within an axon without any
anatomical interruption.
Many infants with birth brachial plexus injury have neuropraxia and recover
spontaneously because neuropraxia tends to disappear within 4-6 weeks.
Axonotmesis - anatomical interruption of the axon with no or only partial interruption
of the connective tissue framework.
This type of nerve injury requires regrowth of the axon to the target muscle, which
takes a considerable amount of time. This regrowth can be inhibited by scar formation.
Whether patients with axonotmesis will require surgical treatment depends on the
number of disrupted axons and the extent of scar formation at the site of nerve injury.
When an axon is disrupted, there is a 2-4 week delay before the axon starts to
regenerate. Axons grow in adults at about 1 inch per month, which means many
months will be required for the axon to grow down to the muscles in the arms. In
infants, however, the axon may regenerate more rapidly, and the distance to be
covered is much less. When a muscle loses its innervation, the nerve receptors will
disappear over a period of 12 to 18 months. This affects the timing of neurosurgical
intervention, because a repair done too late will not have receptors in the muscles for
the growing nerves.
Neurotmesis - complete anatomical disruption of the both the axon and all of the
surrounding connective tissue (rupture of the nerve).
Birth brachial plexus injury is sometimes associated with neurotmesis. This is the
most severe type of nerve injury and has no chance of spontaneous recovery. Early
surgical treatment is necessary.
b) Chest drain after chest injury.
Indications:
A chest tube is indicated to drain the contents of the pleural space. Usually this will be
air or blood, but may include other fluids such as chyle or gastric/oesophageal contents.
Chest tube insertion is also appropriate to prevent the development of a pleural
collection, such as after a thoractomy or to prevent a tension pneumothorax in the
ventilated patient with rib fractures.

Absolute Indications

 Pneumothorax (tension, open or simple)


 Haemothorax
 Traumatic Arrest (bilateral)
Relative Indications

 Rib fractures & Positive pressure ventilation


 Profound hypoxia / hypotension & penetrating chest injury
 Profound hypoxia / hypotension and unilateral signs to a hemithorax
Site: The chest tube is placed (on the correct side) in the mid- or anterior- axillary line,
behind pectoralis major (to avoidhaving to dissect through this thick muscle). On
expiration, the diaphragm rises to the 5th rib at the level of the nipple, and thus chest
drains should be placed above this level. Rib spaces are counted down from the 2nd rib
at the sternomanubrial joint. Practically, the highest rib space that can be easily felt in
the axilla (usually the 4th or 5th) is the most appropriate.

Complications
Acute complications (technique)

 Haemothorax, usually from laceration of intercostal vessel (may require


thoracotomy)
 Lung laceration (pleural adhesions not broken down)
 Diaphragm / Abdominal cavity penetration (placed too low)
 Stomach / colon injury (diaphragmatic hernia not recognised)
 Tube placed subcutaneously (not in thoracic cavity)
 Tube placed too far (pain)
 Tube falls out (not secured)

Late complications

 Blocked tube (clot, lung)


 Retained haemothorax
 Empyema
 Pneumothorax after removal (poor technique)

c) See the answer of question 3.d of Group – C of Supplementary Paper – II of 2009.


d) See the answer of question 3.c of Group - C of Paper –I of 2013.
e) See the answer of question 2.b of Group – B of Paper –II of 2015.
Group – D

4. Answer in brief on any three of the following: 3x5


a) Dental cyst.
b) Brachytherapy.
c) Pre malignant condition of oral cavity.
d) Intermittent claudication.
e) P.E.T scan.

Answer.
a) See the answer of question 4.a of Group –D of Paper –II of 2008.
b) See the answer of question 4.c of Group - D of Supplementary Paper – II of 2012.
c) Pre malignant conditions of oral cavity:
The most common precancerous conditions of the oral cavity are:
Leukoplakia.

Erythroplakia.

Erythroleukoplakia.
Proliferative verrucous leukoplakia (PVL)

Oral submucous fibrosis.

Leukoplakia

Leukoplakia is an abnormal white area or spots on the mucous membrane lining the mouth
with no clear cause. There are many conditions that present with white areas, such as
thrush and lichen planus. White spots may also be caused by irritation, such as biting the
cheek or lip.

The presence of leukoplakia does not necessarily mean cancer. About 3%–17.5% of people
develop squamous cell carcinoma in an area of leukoplakia within 15 years of developing
leukoplakia.
The actual risk of developing oral cavity cancer depends on how abnormal the cells lining
the mouth are in shape, size and appearance. This abnormality is called dysplasia. People
who have leukoplakia are closely followed to look for signs of cancer.
Risks factors

The following risk factors may increase a person s chance of developing leukoplakia:
• (eavy smoking and excessive alcohol use

• Chewing tobacco snuff

Signs and symptoms:The signs and symptoms of leukoplakia may include a whitish area or
spots inside the mouth that cannot be easily scraped off.
Diagnosis:If the signs and symptoms of leukoplakia are present, the doctor will do an oral
examination and order tests to make a diagnosis and rule out other conditions that can
cause similar lesions. Tests may include taking a sample of cells from the abnormal area
and examining them under a microscope (biopsy). Doctors may do a biopsy of any
suspicious area, especially in people at high-risk of developing cancer, such as those who
use tobacco products.
Treatment:There are no standard treatments for leukoplakia. It is managed by active
surveillance, which involves closely monitoring a person s condition. Tests and exams are
done on a regular schedule to detect any cancerous change early.
Because leukoplakia has the potential to develop into cancer, people with this condition
should avoid known risk factors, such as tobacco and alcohol.

Erythroplakia: Erythroplakia is an abnormal red area or red spots that form on the
mucous membrane lining the mouth with no clear cause. The presence of erythroplakia
does not necessarily mean cancer, but this precancerous condition has the highest risk of
developing into cancer. About 51% of these lesions develop into squamous cell carcinoma
and 40% develop into carcinoma in situ
Risks factors:

The following risk factors may increase a person s chance of developing erythroplakia:
• (eavy smoking and excessive alcohol use

• Chewing tobacco
Signs and symptoms:

The signs and symptoms of erythroplakia may include a raised, velvety, red area that often
bleeds when scraped.

Diagnosis: If the signs and symptoms of erythroplakia are present, the doctor will do an
oral examination and run tests to make a diagnosis and rule out other conditions that can
cause similar lesions. Tests may include taking a sample of cells from the abnormal area
and examining them under a microscope (biopsy). Doctors may do a biopsy of any
suspicious area, especially in people at high-risk of developing cancer, such as those who
use tobacco products.

Treatment:
Treatment options for erythroplakia may include:

• Careful watching surveillance and frequent follow-up


• Surgical removal

• Cryosurgery and laser surgery – uncommon


Because erythroplakia has the potential to develop into cancer, people with this condition
should avoid known risk factors, such as tobacco and alcohol.
Erythroleukoplakia
Erythroleukoplakia is a mix of white and red areas on the mucous membrane lining the oral
cavity with no clear cause. Other diseases that cause similar lesions are ruled out. This
precancerous condition is also called speckled leukoplakia or speckled erythroplakia. The
risk of oral cavity cancer developing from erythroleukoplakia falls between the risk for
leukoplakia and erythroplakia.

Proliferative verrucous leukoplakia (PVL)


This is an aggressive form of oral leukoplakia. It is seen more often in older women and is
not strongly linked to tobacco use. People with PVL develop multiple lesions that do not go
away and have a high risk of developing into cancer. Up to 87% of cases develop into
cancer. Because it is so aggressive, PVL requires close follow-up. If cancer develops, then
early and aggressive treatment is given. It often recurs or is resistant to treatment.

Oral submucous fibrosis


Oral submucous fibrosis is a condition that causes progressive fibrosis (formation of scar-
like tissue) in the lining of the mouth. It is strongly linked to chewing betel nut (paan). The
chance of developing oral cavity cancers in people with oral submucous fibrosis is 7%–
13%.
d) Intermittent claudication: This is a special character of pain described for arterial
disorders. This is a clinical condition where a cramping, aching or tightness like
severe pain appears in the leg affected during exercise, usually after a fixed level of
exercise and is promptly (within two to three minutes) relieved with rest. It Is due
to the accumulation of Substance P which fails to get washed away due to poor blood
supply.
Boyd s classification of intermittent claudication:

Grade I Pain starts but if the patient continues to walk the metabolites increase the muscle
Blood flow and sweep away the P- substance produced by exercise and pain
disappears.
Grade 2 Pain continues but the patient can still walk with effort.
Grade 3 Pain compels the patient to take rest.
Grade 4 Pain compels the patient to take rest.

Investigations:

General:

 Blood :
o Routine examination of blood including a hemoglobin percent (low Hb% can decrease
claudication distances and aggravate rest pain),
o Blood sugar examination as diabetics have worse prognosis, are essential.
o Erythrocyte sedimentation rate ESR is usually raised in Buerger s disease.
o In patients with high suspicion of underlying connective tissue disorders, specific test
like RA factor, LE cell phenomenon etc. May be carried out.
o Lipid profile is mandatory in elderly patients with atherosclerosis.
 Urine examination for sugar.
 Plain X-ray of the abdomen will show the presence of arterial calcification and flecks of
calcium may outline an aneurysm.

 ECG: an abnormality in ECG may influence the decision for surgery, in patients with
lower limb disease.

Tests of global Vascular Status:

 Hand Held Doppler ultrasound: blood flow detection uses a continuous wave ultrasound
Signal, beamed at an artery and the reflected beam is picked up by a receiver. The
changes of frequency in the reflected beam, as compared with the transmitted beam,
are due to the ―Doppler shift, resulting from passage of beam through moving
blood. These frequency changes are converted to audio signals. This investigation
may be used effectively in cases where a differential diagnosis of atherosclerosis is
entertained showing the site of block and extent of distal run-off.
Tests for Disease Localisation:

 Duplex imaging: gives accurate information on the size of artery, the flow rate,
turbulence and the presence of stenosis. The combination of Doppler and color
mapping allows easy recognition of stenotic sites. This has been achieved by the use
of pulsed or continuous wave Doppler and the two- dimensional images produced
by the B- scan made either singly or in combination.
 Intravascular Ultrasound: Gives details of arterial walls, luminal contents and
dimensions. This is not a routine investigation for peripheral arterial disease and as
yet is not cost effective.

e) Positron emission tomography (PET) is a nuclear medicine imaging technique which


produces a three-dimensional image or picture of functional processes in the body. The
system detects pairs of gamma rays emitted indirectly by a positron-emitting
radionuclide (tracer), which is introduced into the body on a biologically active
molecule. Images of tracer concentration in 3- dimensional or 4-dimensional space (the
4th dimension being time) within the body are then reconstructed by computer
analysis. In modern scanners, this reconstruction is often accomplished with the aid of a
CT X-ray scan performed on the patient during the same session, in the same machine.

If the biologically active molecule chosen for PET is FDG, an analogue of glucose, the
concentrations of tracer imaged then give tissue metabolic activity, in terms of regional
glucose uptake. Although use of this tracer results in the most common type of PET
scan, other tracer molecules are used in PET to image the tissue concentration of many
other types of molecules of interest.

Operation: To conduct the scan, a short-lived radioactive tracer isotope is injected into
the living subject (usually into blood circulation). The tracer is chemically incorporated
into a biologically active molecule. There is a waiting period while the active molecule
becomes concentrated in tissues of interest; then the research subject or patient is
placed in the imaging scanner. The molecule most commonly used for this purpose is
fluorodeoxyglucose (FDG), a sugar, for which the waiting period is typically an hour.
During the scan a record of tissue concentration is made as the tracer decays. As the
radioisotope undergoes positron emission decay (also known as positive beta decay), it
emits a positron, an antiparticle of the electronwith opposite charge. After travelling up
to a few millimeters[quantify] the positron encounters an electron. The encounter
annihilates them both, producing a pair of annihilation (gamma) photons moving in
opposite directions. These are detected when they reach a scintillator in the scanning
device, creating a burst of light which is detected by photomultiplier tubes or silicon
avalanche photodiodes (Si APD). The technique depends on simultaneous or coincident
detection of the pair of photons moving in approximately opposite direction (it would
be exactly opposite in their center of mass frame, but the scanner has no way to know
this, and so has a built-in slight direction-error tolerance). Photons that do not arrive in
temporal "pairs" (i.e. within a timing-window of few nanoseconds) are ignored.

Localization of the positron annihilation event: The most significant fraction of electron-
positron decays result in two 511 keV gamma photons being emitted at almost 180
degrees to each other; hence it is possible to localize their source along a straight line of
coincidence (also called formally the line of response or LOR). In practice the LOR has a
finite width as the emitted photons are not exactly 180 degrees apart. If the resolving
time of the detectors is less than 500 picoseconds rather than about 10 nanoseconds, it
is possible to localize the event to a segment of a chord, whose length is determined by
the detector timing resolution. As the timing resolution improves, the signal-to-noise
ratio (SNR) of the image will improve, requiring fewer events to achieve the same image
quality. This technology is not yet common, but it is available on some new systems.

Image reconstruction using coincidence statistics:

More commonly, a technique much like the reconstruction of computed tomography


(CT) and single photon emission computed tomography (SPECT) data is used, although
the data set collected in PET is much poorer than CT, so reconstruction techniques are
more difficult. Using statistics collected from tens-of-thousands of coincidence events, a
set of simultaneous equations for the total activity of each parcel of tissue along many
LORs can be solved by a number of techniques, and thus a map of radioactivities as a
function of location for parcels or bits of tissue (also called voxels), may be constructed
and plotted. The resulting map shows the tissues in which the molecular probe has
become concentrated, and can be interpreted by a nuclear medicine physician or
radiologist in the context of the patient's diagnosis and treatment plan.

Combination of PET with CT and MRI:

PET scans are increasingly read alongside CT or magnetic resonance imaging (MRI)
scans, the combination ("co-registration") giving both anatomic and metabolic
information (i.e., what the structure is, and what it is doing biochemically). Because PET
imaging is most useful in combination with anatomical imaging, such as CT, modern PET
scanners are now available with integrated high-end multi-detector-row CT scanners.
Because the two scans can be performed in immediate sequence during the same
session, with the patient not changing position between the two types of scans, the two
sets of images are more-precisely registered, so that areas of abnormality on the PET
imaging can be more perfectly correlated with anatomy on the CT images. This is very
useful in showing detailed views of moving organs or structures with higher anatomical
variation, which is more common outside the brain.

PET-MRI: Presently, only the head and brain can be imaged at these high magnetic field
strengths.

Radionuclides: Radionuclides used in PET scanning are typically isotopes with short
half lives such as carbon- 11 (~20 min), nitrogen-13 (~10 min), oxygen-15 (~2 min),
and fluorine-18 (~110 min).
It is important to recognize that PET technology can be used to trace the biologic
pathway of any compound in living humans (and many other species as well), provided
it can be radiolabeled with a PET isotope. Thus the specific processes that can be probed
with PET are virtually limitless, and radiotracers for new target molecules and
processes are being synthesized all the time; as of this writing there are already dozens
in clinical use and hundreds applied in research. Presently, however, by far the most
commonly used nuclide in clinical PET scanning is fluorine-18 in the form of FDG. The
half life of fluorine-18 is long enough such that fluorine-18 labeled radiotracers can be
manufactured commercially at an offsite location.

Limitations:

o The minimization of radiation dose to the subject is an attractive feature of the use
of short-lived radionuclides. Besides its established role as a diagnostic technique,
PET has an expanding role as a method to assess the response to therapy, in
particular, cancer therapy, where the risk to the patient from lack of knowledge
about disease progress is much greater than the risk from the test radiation.
o Limitations to the widespread use of PET arise from the high costs of cyclotrons
needed to produce the short-lived radionuclides for PET scanning and the need for
specially adapted on-site chemical synthesis apparatus to produce the
radiopharmaceuticals. This limitation restricts clinical PET primarily to the use of
tracers labelled with fluorine-18, which has a half life of 110 minutes and can be
transported a reasonable distance before use, or to rubidium-82, which can be
created in a portable generator and is used for myocardial perfusion studies.
o The presence of the small on-site cyclotron promises to expand in the future as the
cyclotrons shrink in response to the high cost of isotope transportation to remote
PET machines Because the half-life of fluorine-18 is about two hours, the prepared
dose of a radiopharmaceutical bearing this radionuclide will undergo multiple half-
lives of decay during the working day. This necessitates frequent recalibration of the
remaining dose (determination of activity per unit volume) and careful planning
with respect to patient scheduling.
PET is a valuable technique for some diseases and disorders, because it is possible to
target the radio-chemicals used for particular bodily functions.

Oncology:
1. PET scanning with the tracer fluorine-18 (F-18) fluorodeoxyglucose (FDG), called
FDG-PET, is widely used in clinical oncology. This tracer is a glucose analog that is
taken up by glucose-using cells and phosphorylated by hexokinase (whose
mitochondrial form is greatly elevated in rapidly growing malignant tumours).
2. A typical dose of FDG used in an oncological scan is 200- 400 mBq for an adult
human. Because the oxygen atom which is replaced by F-18 to generate FDG is
required for the next step in glucose metabolism in all cells, no further reactions
occur in FDG.
3. FDG-PET can be used for diagnosis, staging, and monitoring treatment of cancers,
particularly in Hodgkin's lymphoma, non-Hodgkin lymphoma, and lung cancer.
4. Many other types of solid tumors will be found to be very highly labeled on a case-by
case basis—a fact which becomes especially useful in searching for tumor
metastasis, or for recurrence after a known highly active primary tumor is removed.
5. Because individual PET scans are more expensive than "conventional" imaging with
computed tomography (CT) and magnetic resonance imaging (MRI), expansion of
FDG-PET in cost-constrained health services will depend on proper health
technology assessment; this problem is a difficult one because structural and
functional imaging often cannot be directly compared, as they provide different
information. Oncology scans using FDG make up over 90% of all PET scans in
current practice.
Neurology:

1. PET neuroimaging is based on an assumption that areas of high radioactivityare


associated with brain activity. Early diagnosis of Alzheimer's disease can be done.
2. Cardiology, atherosclerosis and vascular disease study: In clinical cardiology, FDG-
PET can identify so-called "hibernating myocardium", but its cost-effectiveness in
this role versus SPECT is unclear.
3. Neuropsychology / Cognitive neuroscience: To examine links between specific
psychological processes or disorders and brain activity.
Psychiatry:

1. Numerous compounds that bind selectively to neuroreceptors of interest in


biological psychiatry have been radiolabeled with C-11 or F-18. Radioligands that
bind todopamine receptors (D1,D2, reuptake transporter), serotonin receptors
(5HT1A, 5HT2A, reuptake transporter) opioid receptors (mu) and other sites have
been used successfully in studies with human subjects.
2. Studies have been performed examining the state of these receptors in patients
compared to healthy controls in schizophrenia, substance abuse,mood disorders and
other psychiatric conditions.
Pharmacology: In pre-clinical trials, it is possible to radiolabel a new drug and inject it
into animals.

Pulse Shape Discrimination: The pulse Shape Discrimination (PSD) is a technique


used to define which pulse is related to each crystal. Different Techniques were
introduced to discriminate between two-types of pulses according to its shape (indeed
due to the decay time).

 Safety: PET scanning is non-invasive, but it does involve exposure to ionizing


radiation. The total dose of radiation is not insignificant, usually around 11 mSv.
When compared to the classification level for radiation workers in the UK, of 6 mSv
it can be seen that PET scans need proper justification. This can also be compared to
2.2 mSv average annual background radiation in the UK, 0.02 mSv for a chest x-ray
and 6.5 - 8 mSv for a CT scan of the chest, according to the Chest Journal and ICRP. A
policy change suggested by the IFALPA member associations in year 1999
mentioned that an aircrew member is likely to receive a radiation dose of 4–9 mSv
per year.
The West Bengal University of Health Sciences
M.B.B.S. 3rd Professional Part – II (Supplementary) Examination, 2015
Subject: Surgery Time: 21/2 hrs.
Paper: II Marks: 60
Group –A
1) Describe the pathology, investigations and treatment of differentiated Thyroid
Carcinoma. 4+3+8 = 15
Answer. See the Question 1.a of Group –A of Paper – II of 2012
See the Question 1.a of Group – A of Paper –II of 2011.
The initial treatment for cancer of the thyroid is surgical. The exact nature of the surgical
procedure to be performed depends for the most part on the extent of the local disease. A
consensus approach might be to perform a total thyroidectomy if the primary tumor is larger than
1 cm in diameter or if there is extrathyroidal involvement or distant metastases. Clinically
evident lymphadenopathy should be removed with a neck dissection. If the primary tumor is less
than 1 cm in diameter, a unilateral lobectomy might be considered.
Current National Comprehensive Cancer Network (NCCN) guidelines recommend lobectomy
plus isthmusectomy as the initial surgery for patients with follicular neoplasms, with prompt
completion of thyroidectomy if invasive follicular thyroid carcinoma (FTC) is found on the final
histologic section. Therapeutic neck dissection of involved compartments is recommended for
clinically apparent/biopsy-proven disease.

Group – B
2) a) Mention common causes of lump in left upper quadrant of abdomen. Describe the
clinical features and management of Renal Cell Carcinoma. 5+5+5 = 15
Answer.
Common causes of lump in left upper quadrant of abdomen:
Spleen:
 Splenomegaly
 Splenic cyst
 Splenic neoplasm
Pancreas:
 Pseudocyst pancreas
 Neoplasm of pancreatic body and tail
Stomach:
 Carcinoma stomach
Renal:
 Hydronephrosis
 RCC
 Polycystic kidney.

Others: Retroperitoneal tumour.

For Renal cell carcinoma - See the Question 2.b of Group - B of Supplementary Paper- II
of 2013

Or
b) Mention the sites of narrowing of ureter. Describe the clinical features, complications
and treatment of ureteric stones. 3+4+3+5 = 15
Answer. There are three constrictions, which are the most common sites of renal
calculus obstruction:
 At the pelvi-ureteric junction (PUJ) of the renal pelvis and the ureter
 As the ureter enters the pelvis and cross over the common iliac artery bifurcation
 At the vesicoureteric junction (VUJ) as the ureter enters the bladder wall.
Clinical features:
 Ureteric stones usually present with sudden onset of severe flank pain which is colicky
(waves of increasing severity are followed by a reduction in severity, but it seldom goes
away completely).
 Fever.
 Patients with ureteric calculus may present with peristaltic pain (renal colic), haematuria,
nausea and vomiting.
 It may radiate to the groin as the stone passes into the lower ureter. ~50% of patients with
classic symptoms for a ureteric stone do not have a stone confirmed on subsequent imaging
studies, nor do they physically ever pass a stone.
 Ureteric stone pain is colicky the patient moves around, trying to find a comfortable position.
They may be doubled-up with pain.
 The quality and location of pain is dependent on the calculi's location within the ureter.
Calculi within the pelvicoureteric junction may cause deep flank pain without radiation to the
groin due to distension of the renal capsule, whereas pain from upper ureteral calculi radiate
to the flank and lumbar areas. Calculi in the mid ureter results in pain radiating anteriorly
while distal ureteric calculi pain radiates to the groin via referred pain from the genitofemoral
or ilioinguinal nerves.

 Calculi in the versicoureteric junction may also cause irritative voiding symptoms such as
dysuria and urinary frequency.

Complications:
 Urinary tract infection
 Acute urinary retention
 Hydroureter
 Hydronephrosis
 Pyoureter and pyonephrosis

Treatment:
 The most important aspect of examination in a patient with a ureteric stone confirmed on
imaging is to measure their temperature.
 If the patient has a stone and a fever, they may have infection proximal to the stone. A fever
in the presence of an obstructing stone is an indication for urine and blood culture,
intravenous fluids and antibiotics, and nephrostomy drainage if the fever does not resolve
within a matter of hours.
 The intravenous urogram (IVU), is the mainstay of imaging in patients with flank pain, has
been replaced by CT urography.
 Plain abdominal X-ray and renal ultrasound are not sufficiently sensitive or specific for their
routine use for diagnosing ureteric stones.
 MR urography: This is a very accurate way of determining whether a stone is present in the
ureter or not.
Ureteric stones: acute management:
While appropriate imaging studies are being organized, pain relief should be given.

 A non-steroidal anti-inflammatory (e.g. diclofenac) by intramuscular or intravenous


injection, by mouth or per rectum provides rapid and effective pain control
 Where NSAIDS are inadequate, opiate analgesics such as pethidine or morphine are
added
 Calcium channel antagonists (e.g. nifedipine) may reduce the pain of ureteric colic by
reducing the frequency of ureteric contractions.

Watchful waiting:

o In many instances, small ureteric stones will pass spontaneously within days or a few weeks,
with analgesic supplements for exacerbations of pain.
o Chances of spontaneous stone passage depend principally on stone size. Between 90 - 98%
of stones measuring <4mm will pass spontaneously. Average time for spontaneous stone
passage for stones 4 - 6mm in diameter is 3 weeks.
o Stones that have not passed in 2 months are unlikely to do so. Therefore, accurate
determination of stone size (on plain abdominal X-ray or by CTU) helps predict chances of
spontaneous stone passage.
o Nifedipine and tamsulosin (an alpha adrenergic adrenoceptor blocking drug) may assist
spontaneous stone passage and reduce frequency of ureteric colic.

Ureteric stones: indications for intervention to relieve obstruction and/or remove the stone:

 Pain which fails to respond to analgesics or recurs and cannot be controlled with additional
pain relief.
 Fever. Have a low threshold for draining the kidney (usually done by percutaneous
nephrostomy).
 Impaired renal function (solitary kidney obstructed by a stone, bilateral ureteric stones, or
pre-existing renal impairment which gets worse as a consequence of a ureteric stone).
Threshold for intervention is lower.
 Prolonged unrelieved obstruction. Social reasons. Young, active patients may be very keen to
opt for surgical treatment because they need to get back to work or their childcare duties,
whereas some patients will be happy to sit things out. Airline pilots and some other
professions are unable to work until they are stone free.

Emergency temporizing and definitive treatment of the stone:


Where the pain of a ureteric stone fails to respond to analgesics or where renal function is
impaired because of the stone, then temporary relief of the obstruction can be obtained by
insertion of a JJ stent or percutaneous nephrostomy tube.
The patient may elect to proceed to definitive stone treatment by immediate ureteroscopy (for
stones at any location in the ureter) or ESWL (if the stone is in the upper and lower ureter ESWL
cannot be used for stones in the mid ureter because this region is surrounded by bone, which
prevents penetration of the shock waves) .

Treatment options for ureteric stones:

 ESWL: in situ; after push-back into the kidney (i.e. into the renal pelvis or calyces); or
after JJ stent insertion.
 Ureteroscopy
 PCNL
 Open ureterolithotomy
 Laparoscopic ureterolithotomy

The ureter can be divided into two halves (proximal and distal to the iliac vessels) or in thirds
(upper third from the PUJ to the upper edge of the sacrum; middle third from the upper to the
lower edge of the sacrum; lower third from the lower edge of the sacrum to the VUJ).
Proximal ureteric stones:
 <1cm diameter: ESWL (in situ, push-back)
 >1cm diameter: ESWL, ureteroscopy, PCNL

Distal ureteric stones:

 Both ESWL and ureteroscopy are acceptable options.


 Stone free rate <1cm: 80 - 90% for both ESWL and ureteroscopy; >1cm: 75% for both
ESWL and ureteroscopy.

Group – C
3) Write short notes on (any three) 3x5 = 15
a) Testicular torsion
b) Extradural haematoma.
c) Flail chest.
d) Ameloblastoma.
e) Submandibular sialolithiasis.
Answer.

a) Testicular torsion. See the Question 3.d of Group – C of Paper – II of 2012


b) Extradural haematoma. See the Question 2.c of Group – B of Supplementary Paper
–II of 2012.
c) Flail chest. See the Question 4.c of Group – D of Supplementary Paper - II of 2014.
d) Ameloblastoma. See the Question 4.c of Group – D of Paper – II of 2012
e) Submandibular sialolithiasis.

Introduction:

 Salivary duct stones are masses of crystallized minerals that form in the tubes that saliva
passes through after it’s made in the salivary glands. This condition is also known as
sialolithiasis, and the stone may be referred to as salivary duct calculus.
 These stones mainly occur in middle-aged adults.
 They are the most common cause of blockages in the salivary ducts.
 Salivary duct stones occur most often in the ducts connected to the submandibular glands.
Causes:
 Certain substances in the saliva, such as calcium phosphate and calcium carbonate, can
crystalize and form stones that range in size from a few millimeters to more than two
centimeters.
 When these stones block the salivary ducts, saliva builds up in your salivary glands,
which makes them swell.
 The reason why the stones form in the first place isn’t known. A few factors have been
associated with a higher risk of having these stones.
These include:
o Taking medications, such as blood pressure drugs and antihistamines, which reduce the
amount of saliva the glands produce
o Being dehydrated, as this makes your saliva more concentrated
o Not eating enough food, which causes a decrease in saliva production

Clinical features:

 Pain, which is intermittent, and may suddenly get worse before mealtimes, and then
slowly get better (partial obstruction).
 Swelling of the gland, also usually intermittent, often suddenly appearing or increasing
before mealtimes, and then slowly going down (partial obstruction).
 Tenderness of the involved gland.
 Palpable hard lump, if the stone is located near the end of the duct. If the stone is near the
submandibular duct orifice, the lump may be felt under the tongue.
 Lack of saliva coming from the duct (total obstruction).
 Erythema (redness) of the floor of the mouth (infection).
 Pus discharging from the duct (infection).
 Cervical lymphadenitis (infection).

Diagnosis:

 Clinical examination.
 Imaging tests can provide a more accurate X-rays, an ultrasound, or a computed
tomography (CT) scan of the face.
 An ultrasound creates an image of the face using sound waves, and a CT scan combines
multiple X-rays into one cross-sectional, detailed image.

Treatment: Some current treatment options are:

 Non-invasive:

 For small stones, hydration, moist heat therapy, NSAIDs (nonsteroidal anti-inflammatory
drugs) occasionally, and having the patient take any food or beverage that is bitter and/or
sour. Sucking on citrus fruits, such as a lemon or orange, may increasesalivation and
promote spontaneous expulsion of the stone.
 Some stones may be massaged out by a specialist.
 Shock wave therapy (Extracorporeal shock wave lithotripsy).

 Minimally invasive:

 Sialendoscopy
 Surgical:

 An ENT or oral/maxillofacial surgeon may cannulate the duct to remove the stone
(sialectomy).
 A surgeon may make a small incision near the stone to remove it.
 In some cases when stones continually reoccur the offending salivary duct is removed.

 Supporting treatment:

 To prevent infection while the stone is lodged in the duct, antibiotics are sometimes
used.
Group – D
4) Answer briefly on (any three): 3x5 = 15
a) Telecobalt therapy.
b) Extradural haematoma.
c) Pyloric stenosis in infant.
d) P.D.A.
e) Wax bath.
Answer.
a) Telecobalt therapy.
b) Introduction: External beam radiotherapy (EBRT) or teletherapy is the most common form
of radiotherapy. The patient sits or lies on a couch and an external source of radiation is
pointed at a particular part of the body.
o Cobalt therapy or cobalt-60 therapy is the medical use of gamma rays from cobalt-
60 radioisotopes to treat conditions such as cancer.
o Telecobalt machines are still prominently used for the treatment of a variety of cancer
cases in developing countries.
o Cobalt units which produce stable, dichromatic beams of 1.17 and 1.33 MeV, resulting in
an average beam energy of 1.25 MeV. The role of the cobalt unit has partly been replaced
by the linear accelerator, which can generate higher energy radiation. Cobalt treatment
still has a useful role to play in certain applications (for example the - Gamma Knife) and
is still in widespread use worldwide, since the machinery is relatively reliable and simple
to maintain compared to the modern linear accelerator.
Uses:
 In small lesions, T1 and T2, of the laryngopharynx, telecobalt therapy gives as good
results as the combination of surgery and pre and post operative telecobalt therapy.
 Early malignant tumours of vocal cord.
 In oesophageal carcinomas with low-dose rate telecobalt therapy (LDRT) as a boost.
 In bronchial cancer.
 Total body irradiation. Technique using fractionated irradiation by telecobalt-therapy
c) Extradural haematoma. See the Question 2.c of Group – B of Supplementary Paper –II
of 2012.
d) Pyloric stenosis in infant. See the Question 3.d of Group – C of Paper –II of 2010.
e) P.D.A.
Definition: Ductus arteriosus is a vessel that connects the pulmonary artery and aorta. Failure
of closure and continued patency of fetal channel is termed persisent ductus arteriosus (PDA).

 In the fetus, the ductus diverts blood flow from the pulmonary circulation (high resistance)
to the descending aorta.

 An Increase in PaO2 constricts the ductus. Other factors, such as the release of vasoactive
substances, contribute to the closure

 Balance between constricting/relaxing substances

Factors assoc with increased incidence: Factors associated with decreased incidence:

 Prematurity  Antenatal steroid administration

 RDS  IUGR

 Fluid overload  Prolonged rupture of membranes

 Asphyxia

Clinical Signs:

 The clinical features assoc with L to R shunt depend on the magnitude of shunt and the
ability to handle extra volume.

 Shunt: magnitude and direction related to vessel diameter and pressure gradient between A
and PA.

PDA: Clinical Signs:

 Murmur

 Hyperactive precordium

 Bounding peripheral pulses

 Increase in pulse pressure


 Hypotension

 Respiratory deterioration
Management:

 Ventilatory support

 Fluid restriction

 Maintain hematocrit

 Non surgical: Indomethacin (PGE1 inhib)


Ibuprofen
Transcatheter

 Surgical: Ligation

f) Wax bath. See the Question 4.b of Group – D of Paper – II of 2013.


The West Bengal University of Health Sciences

M.B.B.S. 3rd Professional Part – II (Supplementary) Examination, 2014

Subject: Surgery Time: 21/2 hrs.

Paper: II Marks: 60

Group –A

1. Discuss causes, investigation and management of haematuria. 4+3+8 = 15

See the answer of [Question 2.b of Group – B of paper –II of 2013]

Causes of Hematuria
Glomerular hematuria
Primary: IgA nephropathy, GBM disease, FSGS, MPGN
Secondary: postinfectious, SLE, ANCA-GN
Nonglomerular hematuria
Neoplasm- kidney, bladder, prostate
Urolithiasis
Infection
Anatomical abnormalities: ADPKD, reflux, cystocele, obstruction, nutcracker
Other diseases: aneurysm, AV malformation, infarction, trauma, hypercalciuria, hyperuricosuria,
sickle cell disease

Group – B

2. a. What are the different types of renal calculi? Discuss the clinical features and
management of renal calculi. 3+4+8= 15

Answer. Different types of renal calculi – see the answer of question no 3.e of group
C of paper II of 2013

Clinical features:
 Silent calculus: Renal failure may be the first indication of bilateral silent calculi,
although secondary infection usually produces symptoms first.
 Pain: Pain occurs in 75 per cent of people with urinary stones. Fixed renal pain occurs in
the renal angle ,the hypochondrium, or in both. It may be worse on movement.
 Pain resulting from renal stones rarely lasts more than 8 hours in the absence of infection.
 There is no pyrexia, although the pulse rate rises because of the severe pain.
 Haematuria (microscopic or occasionally macroscopic).
 Struvite staghorn calculi classically present with recurrent UTIs. Malaise, weakness, and
loss of appetite can also occur.
 Less commonly, struvite stones present with infective complications (pyonephrosis,
perinephric abscess, septicaemia, xanthogranulomatous pyelonephritis).

Diagnostic tests

 Plain abdominal radiography: calculi that contain calcium are radiodense. Sulphur-
containing stones (cystine) are relatively radiolucent on plain radiography.
 Radiodensity of stones in decreasing order: calcium phosphate > calcium oxalate >
struvite (magnesium ammonium phosphate) >> cystine.
 Completely radiolucent stones (e.g. uric acid, triamterene, indinavir) are usually
suspected on the basis of the patient's history and/or urine pH (pH <6 gout; drug history-
triamterene, indinavir), and the diagnosis may be confirmed by ultrasound, CTU, or
MRU.
 Renal ultrasound: its sensitivity for detecting renal calculi is ~95%. A combination of
plain abdominal radiography and renal ultrasonography is a useful screeing test for renal
calculi.
 IVU: increasingly being replaced by CTU. Useful for patients with suspected indinavir
stones (which are not visible on CT).
 CTU: a very accurate method of diagnosing all but indinavir stones. Allows accurate
determination of stone size and location and good definition of pelvicalyceal anatomy.
 MRU: cannot visualize stones, but is able to demonstrate the presence of hydronephrosis.

Treatment:

Treatment:
Acute presentations (renal colic)

 Analgesia, e.g. diclofenac; IV fluids.


 Small stones (<0.5cm) may be managed expectantly as most will pass spontaneously.
 Emergency treatment with percutaneous nephrostomy and/or ureteric stent insertion is
necessary if either pain or obstruction is persistent.
 The traditional indications for intervention are pain, infection, and obstruction.
Haematuria caused by a stone is only very rarely severe or frequent enough to be the only
reason to warrant treatment.

Elective presentations

 Extra-corporeal shock-wave lithotripsy (ESWL).


o Focused, externally generated electrohydraulic or ultrasonic shock-waves.
o Targeted on to the calculus using ultrasound, X-ray, or a combination.
o Causes stone disintegration and the fragments are then voided.
o Effective for stones upto 1cm.
 Percutaneous nephrolithotomy (PCNL).
o For stones in the renal pelvis or calyces.
o Percutaneous track into the renal pelvis using fluoroscopic guidance.
o Nephroscope is inserted and the calculus visualized.
o Removed either in total or, if large, following fragmentation.
 Open nephrolithotomy/ureterolithotomy.
o For large staghorn calculi or complex stones.

Prevention of recurrence

 Increase oral fluid intake and reduce calcium intake.


 Correct metabolic abnormalities.
 Treat infection promptly.
 Urinary alkalization, e.g. sodium (mainly for cystine and urate stones).
 Thiazide diuretics (for idiopathic hypercalciuria).

Or

b. What are the clinical features of primary hyperparathyroidism? Discuss the


investigation and management of primary hyperparathyroidism. 6+5+4 = 15

Answer.

Introduction:

 Primary hyperparathyroidism (PHPT) is a common endocrine disease.


 Prevalence is highest among postmenopausal women, with 1:500 possibly being affected.
 Most patients are identified by an incidental finding of raised serum calcium during
investigations for another condition.

Clinical features:

 Classical symptoms are described as:


o Moans - psychological/psychiatric symptoms (lethargy, depressed mood);
o Groans - non-specific gastrointestinal symptoms (abdominal pain, constipation);
o Bones - aches/pains localized in large joints;
o Stones - calcium based renal stones.
Symptoms and signs of primary hyperparathyroidism

Causes:

 The most common cause of primary hyperparathyroidism is a sporadic,


single parathyroid adenoma.
 Less common are parathyroid hyperplasia, parathyroid carcinoma (malignant tumor), and
adenomas in more than one gland.
Familial syndromes associated with primary hyperparathyroidism:

Investigations Diagnosis:

 The diagnosis of primary hyperparathyroidism is made by blood tests. Serum calcium


levels are elevated, and the parathyroid hormone level is abnormally high compared with
an expected low level in response to the high calcium.
 Serum parathyroid hormone concentration (PTH) in the presence of hypercalcaemia
confirms the diagnosis [e.g. bone metastases (breast, renal, thyroid carcinoma) have a low
(i.e. inhibited) PTH concentration].
 Corrected serum calcium is highly suggestive if unexplained but not diagnostic.
 Urinary cAMP is occasionally measured; this is generally elevated.
 High-resolution neck ultrasound may identify tumours.
 Sestamibi (radioisotope) scanning used to localize adenomas (accurate in 50%) and
allows a focused approach (minimally invasive parathyroidectomy).
 The most frequently applied approach is preoperative sestamibi scanning, followed by
direct excision of the identified gland and confirmation of cure by intraoperative PTH
measurement.
Preoperative imaging in patients with primary hyperparathyroidism:

Treatment:

Medical treatment

 Hypercalcaemic crisis needs aggressive rehydration.


 Establish large calibre IV access. Give 1L in first hour, further 4-6L in first 24h.
 Monitor urine output and central venous pressure until normalized.
 Frusemide can be added to increase urinary excretion of calcium once rehydration is
adequate.
 Bisphosphonates (e.g. IV pamidronate) should be avoided in PHPT when
parathyroidectomy is anticipated, since they impair the ability to maintain
normocalcaemia after the excision of an overactive parathyroid adenoma.
Surgical treatment:

Indications for parathyroidectomy in primaryhyperparathyroidism.


 Urinary tract calculi
 Reduced bone density
 High serum calcium
 ? All in younger age group <50 years
 Deteriorating renal function
 Symptomatic hypercalcaemia

Operation:

 Bilateral neck exploration, visualization of all four parathyroid glands with excision of
the enlarged one(s), has for many years been the standard treatment. It remains the
treatment for those with negative localization scans.
 When imaging studies identify reliably the position of the adenoma, patients can undergo
minimally invasive parathyroidectomy (MIP). This is a focused neck exploration through
a lateral cervical scar aiming to remove the adenoma visualized on scanning and not to
explore the other parathyroid glands.

Group – C

3. Write short notes on (any three)


3x5
a. Parotid abscess.
b. Patent ductus arteriosus.
c. Chordee
d. QUART.
e. Chronic subdural haematoma.

a. Answer. Parotid abscess

A parotid abscess is a buildup of pus and infected material in the parotid gland, the
largest of the salivary glands surrounding the jaw.

Patients at risk:
 This condition most commonly develops as a consequence of poor oral care, usually
when people are recovering from a medical condition and are less capable of following
a basic oral hygiene routine.
 Immunocompromised patients.
 Diabetics.

Bacteriology: Staphylococcus aureus, Klebsiella pneumoniae, Escherichia coli and Alpha-


haemolytic streptococci.

Symptoms:
 Symptoms of a parotid abscess include significant pain and swelling of one or
both parotid glands. The area behind the jaw can feel tight and hot, and the skin may be
red and shiny.

Diagnosis:
 A careful physical examination to rule out conditions with similar symptoms,
like mumps.
 Examination of the mouth may also show leakage of pus from the parotid gland into the
oral cavity and the patient may have foul-smelling breath as a result of the infection.

Treatment:
Treatment usually involves drainage, medications to treat infection, and monitoring for
signs of recurrence.

b. Answer. Patent ductus arteriosus.


Patent ductus arteriosus (PDA) is a congenital disorder in the heart wherein
a neonate's ductus arteriosus fails to close after birth.
Patent ductus arteriosus (PDA) is a persistent opening between two major blood vessels
leading from the heart. The opening, called the ductus arteriosus, is a normal part of a
baby's circulatory system before birth that usually closes shortly after birth. If it remains
open, however, it's called a patent ductus arteriosus.

Heart cross-section with PDA

Symptoms:
Patent ductus arteriosus symptoms vary with the size of the defect and whether the baby is
full-term or premature. A small PDA might cause no signs or symptoms and go
undetected for some time — even until adulthood. A large PDA can cause signs of heart
failure soon after birth.

A large PDA found during infancy or childhood might cause:

 Poor eating, which leads to poor growth


 Sweating with crying or eating
 Persistent fast breathing or breathlessness
 Easy tiring
 Rapid heart rate

Risk factors:

 Premature birth. .
 Family history and other genetic conditions. A family history of heart defects and other
genetic conditions, such as Down syndrome, increase the risk of having a PDA.
 Rubella infection during pregnancy.
 Being born at a high altitude.

Complications:

 High blood pressure in the lungs (pulmonary hypertension).


 Heart failure.
 Heart infection (endocarditis).

Investigations and diagnosis:

 Echocardiogram. This test also helps the doctor evaluate the heart valves and detect other
potential heart defects.
 Chest X-ray. An X-ray image helps the doctor see the condition of your baby's heart and
lungs. An X-ray might reveal conditions other than a heart defect, as well.
 Electrocardiogram (ECG). This test records the electrical activity of the heart, which can
help to diagnose heart defects or rhythm problems.
 Cardiac catheterization. This test isn't usually necessary for diagnosing a PDA alone, but it
might be done to examine other congenital heart defects found during an echocardiogram or if
a catheter procedure is being considered to treat a PDA.

Treatment:
Treatments for patent ductus arteriosus depend on the age of the person being treated. Options
might include:

 Watchful waiting. In a premature baby, a PDA often closes on its own. For full-term babies,
children and adults who have small PDAs that aren't causing other health problems,
monitoring might be all that's needed.
 Medications. In a premature baby, nonsteroidal anti-inflammatory drugs (NSAIDs) — such as
ibuprofen (Advil, Infant's Motrin, others) or indomethacin (Indocin) — might be used to help
close a PDA. NSAIDs block the hormone like chemicals in the body that keep the PDA open.
NSAIDs won't close a PDA in full-term babies, children or adults.
 Open-heart surgery. If medications aren't effective and child's condition is severe or causing
complications, open-heart surgery might be recommended.

Open-heart surgery might also be recommended for adults who have a PDA that's causing
health problems. Possible risks include hoarseness, bleeding, infection and a paralyzed
diaphragm.

 Catheter procedures. Premature babies are too small for catheter procedures. However, if the
baby doesn't have PDA-related health problems, the doctor might recommend waiting until the
baby is older to do a catheter procedure to correct the PDA. Catheter procedures can also be
used to treat full-term babies, children and adults.

Complications from catheter procedures include bleeding, infection, or movement of the plug
or coil from where it was placed in the heart.

c. Answer. Chordee

Introduction: Chordee is a condition in which the head of the penis curves downward or upward,
at the junction of the head and shaft of the penis. The curvature is usually most obvious during
erection, but resistance to straightening is often apparent in the flaccid state as well
.
Causes:

 A chordee may be caused by an underlying condition, such as a disorder of sex


development or an intersex condition.
 Complication of circumcision,
 Hypospadias.

Presentation:

It is usually considered a congenital malformation of unknown cause. Since at an early stage of


fetal development the penis is curved downward, it has been proposed that chordee results from
an arrest of penile development at that stage.
The curvature of a chordee can involve:
 Tethering of the skin with urethra and corpora of normal size;
 Curvature induced by fibrosis and contracture of the fascial tissue (Buck's fascia or dartos)
surrounding the urethra;
 Disproportionately large corpora in relation to the urethral length without other demonstrable
abnormality of either; or
 A short, fibrotic urethra that tethers the penis downward (the least common type).
Severe degrees of chordee are usually associated with hypospadias, but mild degrees of curvature
may occur in many otherwise normal males. When the curved penis is small and accompanied by
hypospadias, deficiency of prenatal androgen effect can be inferred.
Treatment:

 The principal treatment of chordee is surgery in infancy, usually by a pediatric urologist.


 With chordees caused by circumcision, the preferred method of surgical treatment is a z-
plasty.
 The preferred time for surgery is between the ages of 6 and 18 months and correction is
usually successful.

d. Answer. QUART= Conservative surgery and irradiation


It is quadrantectomy and radiotherapy for early stage breast cancer.
It is a modality used for breast conserving surgery (BCT).
A quadrantectomy, also referred to as a segmental or partial mastectomy, is a surgical
procedure for breast cancer in which one quarter of breast tissue is removed along with
muscles of the chest wall within a 2 to 3 centimeter radius of a tumor.

Quadrantectomy associated to axillary dissection with radiation therapy (QUART) or


without it (QUAD), likely shows three advantages:
1) Preservation of healthy breast with satisfying cosmetic appearance if reshaped by the
plastic surgeon;
2) Preservation of vascular supply and innervation of the nipple-areola complex except of
central quadrantectomy;
3) Maintenance of some average symmetry improvable with the reshaping reduction of
the contralateral breast.

Contraindications:

Absolute Relative
 Pregnancy, although such surgery may be done  Collagen vascular disease.
during the third trimester with irradiation  The presence of multiple gross tumors in the same
following delivery. quadrant.
 Two or more primary tumors in separate  Presence of a large tumor in a small breast in
quadrants of the breast or diffuse which an adequate resection would result in
microcalcifications. significant cosmetic alteration. In this
 Previous irradiation of the breast. circumstance, preoperative chemotherapy should
 Persistent positive tissue margins after surgery be considered.
 Large pendulous breast.

e. Answer. Chronic subdural haematoma

Introduction: A chronic subdural hematoma is an "old" collection of blood and blood breakdown
products between the surface of the brain and its outermost covering (the dura). The chronic
phase of a subdural hematoma begins several weeks after the first bleeding.

Causes:

A subdural hematoma develops when the tiny veins that run between the dura and surface of the
brain (bridging veins) tear and leak blood. This is usually the result of a head injury.
A collection of blood then forms over the surface of the brain. In a chronic subdural collection,
blood leaks from the veins slowly over time, or a fast hemorrhage is left to clear up on its own.

A subdural hematoma is more common in the elderly because of normal brain shrinkage that
occurs with aging. This shrinkage stretches and weakens the bridging veins. These veins are
more likely to break in the elderly, even after a minor head injury. You or your family may not
remember any injury that could explain it.

Risks include:

 Long-term heavy alcohol use.


 Long-term use of aspirin, anti-inflammatory drugs such as ibuprofen, or blood thinning
(anticoagulant) medication such as warfarin.
 Diseases associated with reduced blood clotting
 Head injury
 Old age

Symptoms:
In some cases, there may be no symptoms. However, depending on the size of the hematoma and
where it presses on the brain, any of the following symptoms may occur:

 Confusion or coma
 Decreased memory
 Difficulty speaking or swallowing
 Difficulty walking
 Drowsiness
 Headache
 Seizures
 Weakness or numbness of arms, legs, face

Investigations:
 Because the symptoms and signs are often subtle, if there is any suspicion of a
hematoma, a head CT or head MRI scan should be done.

Treatment:

 The goal of treatment is to control symptoms and reduce or prevent permanent damage to
the brain. Anticonvulsant medications such as carbamazepine, lamotrigine or
levetiracetam may be used to control or prevent seizures.
 Surgery may be needed. This may include drilling small holes in the skull to relieve
pressure and allow blood and fluids to be drained. Large hematomas or solid blood
clots may need to be removed through a larger opening in the skull (craniotomy).

 Hematomas that do not cause symptoms may not require treatment. Chronic subdural
hematomas often come back after being drained. Therefore, it is sometimes better to
leave them alone unless they are causing symptoms.

Group – D

4. Answer briefly on (any three): 3x5

a. Dentigerous cyst.
b. Local anaesthesia in inguinal hernia surgery.
c. Venous ulcer.
d. Transluminal USG.
e. Radio - active Iodine.

a. Dentigerous cyst: (See the answer of 3.c of Group-C of paper II of 2014)

b. Local anaesthesia in inguinal hernia surgery

Introduction: Specialist hernia centres with a dedicated hernia service have achieved remarkable
results for inguinal hernia repair with the use of local anaesthesia and set the standards for groin
hernia surgery.

The following are the different steps of the technique of local anesthesia:

 Subdermic infiltration.
 Intradermic injection
 Deep subcutaneous injection.
 Subfascial infiltration.
 Pubic tubercle and hernia sac injection.

Advantages:

 Inguinal hernia repair with local anaesthesia is quite safe and results in a good success
rate in elderly patients despite a higher rate of comorbidity typical of this kind of
patient.
 Can be used for day care cases.
 No hazards of general or spinal anaesthesia.

Disadvantages:

 A commonly perceived problem of local anaesthetic inguinal hernia repair is the pain
of infiltration.
 A potential problem of local anaesthetic in inguinal hernia repair is toxicity especially in
obese patients who require large volumes.

c.Venous ulcer.

Introduction: Chronic venous disease, including chronic venous insufficiency and chronic
venous ulceration, is a common and important medical problem that causes significant
morbidity. Venous ulcers are expensive to treat and adversely impact patient’s quality of life.
Venous ulcers occur more commonly in the elderly, the peak prevalence occurring between ages
60 and 80 years.

A venous leg ulcer can develop after a minor injury if there is a problem with the circulation of
blood in your leg veins. If this happens, the pressure inside the veins increases.

Risk factors:
 Obesity – this increases the risk of high pressure in the leg veins
 Not being able to move for a long period of time – this can weaken the calf muscles,
which can affect circulation in the leg veins
 Having previously had deep vein thrombosis (DVT)– blood clots that develop in the leg,
which can damage valves
 Varicose veins – swollen and enlarged veins caused by malfunctioning valves
 Previous injury to the leg, such as a broken or fractured bone, which may cause DVT
 Previous surgery to the leg, such as a hip replacement or knee replacement, which can
prevent you from moving about
 Increasing age – as people generally find it harder to move about as they get older

Pathophysiology: (go through):

 Venous hypertension: Deep vein thrombosis, perforator insufficiency, superficial and deep
vein insufficiencies, arteriovenous fistulas and calf muscle pump insufficiencies lead to
increased pressure in the distal veins of the leg and finally venous hypertension.
 Fibrin cuff theory: Fibrin gets excessively deposited around capillary beds leading to
elevated intravascular pressure. This causes enlargement of endothelial pores resulting in
further increased fibrinogen deposition in the interstitium. The "fibrin cuff" which surrounds
the capillaries in the dermis decreases oxygen permeability 20-fold. This permeability barrier
inhibits diffusion of oxygen and other nutrients, leading to tissue hypoxia causing impaired
wound healing.
 Inflammatory trap theory: Various growth factors and inflammatory cells, which get trapped
in the fibrin cuff promote severe uncontrolled inflammation in surrounding tissue preventing
proper regeneration of wounds. Leukocytes get trapped in capillaries, releasing proteolytic
enzymes and reactive oxygen metabolites, which cause endothelial damage. These injured
capillaries become increasingly permeable to various macromolecules, accentuating fibrin
deposition. Occlusion by leukocytes also causes local ischemia thereby increasing tissue
hypoxia and reperfusion damage.
 Dysregulation of various cytokines.
 Dysregulation of various pro-inflammatory cytokines and growth factors like tumor necrosis
factor-α (TNF-α), TGF-β and matrix metalloproteinases lead to chronicity of the ulcers.
 Miscellaneous: Thrombophilic conditions like factor V Leiden mutation, prothrombin
mutations, deficiency of antithrombin, presence of antiphospholipid antibodies, protein C and
S deficiencies and hyperhomocysteinemia are also implicated.

Clinical features:
Venous leg ulcers are open, often painful, sores in the skin that take more than four to six weeks
to heal. They most often develop on the inside of the leg, just above the ankle.
Symptoms:
 Swollen ankles (oedema)
 Discolouration and darkening of the skin around the ulcer

 Hardened skin around the ulcer, which may make your leg feel hard and resemble the
shape of an upside-down champagne bottle
 A heavy feeling in your legs
 Aching or swelling in your legs
 Red, flaky, scaly and itchy skin on your legs (varicose eczema)
 Swollen and enlarged veins on your legs (varicose veins)
 An unpleasant and foul-smelling discharge from the ulcer

Signs of an infection
A venous leg ulcer can be susceptible to bacterial infection. Symptoms of an infected leg ulcer
can include:
 worsening pain

 a green or unpleasant discharge coming from the ulcer


 redness and swelling of the skin around the ulcer
 a high temperature (fever)
Nonsurgical Treatment

A. Infected ulcers

 Necessitate treatment of the infection first.


 Staphylococcus aureus, Streptococcus pyogenes, and Pseudomonas species are
responsible for most infections.
 Usually treated with local wound care, wet-to-dry dressings, and oral antibiotics.
 Topical antiseptics should be avoided.
 Severe infections require intravenous antibiotics.

B. Leg elevation

Leg elevation can temporarily decrease edema and should be instituted when swelling occurs.
This should be done before a patient is fitted for stockings or boots.

C. Compression therapy

Compression therapy is the primary treatment for CVI.

 Elastic compression stockings


o Fitted to provide a compression gradient from 30 to 40 mm Hg, with the greatest
compression at the ankle.
o Donned on arising from bed and removed at bedtime.
o Effective in healing ulcers but can take months to obtain good results.
o Stockings do not correct the abnormal venous hemodynamics and must be worn after
the ulcer has healed to prevent recurrence.
o Principal drawback is patient compliance.
 Unna boots
o Paste gauze compression dressings that contain zinc oxide, calamine, and glycerin.
o Used to help prevent further skin breakdown.
o Provide nonelastic compression therapy.
o Changed once or twice a week.
o Healing time for ulcers is less than that of elastic compression alone.
 Pneumatic compression devices
o Provide dynamic sequential compression.
o Used primarily in the prevention of deep vein thrombi in hospitalized patients.
o Also used successfully to treat venous insufficiency.
D. Topical medications

 Largely ineffective as a stand-alone therapy for venous stasis ulcers.


 Topical therapy is directed at absorbing wound drainage and avoiding desiccation of the
wound.
 Antiseptics can be counterproductive. Hydrogen peroxide, povidone-iodine, acetic acid,
and sodium hypochlorite are toxic to cultured fibroblasts and should be used for the
shortest duration necessary to control ulcer infection.

Surgical Therapy:

 Skin grafting

o Occasionally used to speed healing of large ulcers.


o Ulcer bed should be dry and free of infection.
o Fenestrated split-thickness skin graft is preferred to allow for serous drainage.
o Bedrest is recommended until the ulcer has healed completely.
o Recurrence is common unless the underlying venous pathology is corrected or
conservative support (elastic compression) is initiated and maintained after skin grafting.
Cultured skin substitutes are now commonly used to minimize patient discomfort and
avoid possible donor-site complications.
o Stripping the greater saphenous vein.

d. Answer. Transluminal USG.

Transrectal Ultrasound:

A TRUS may be done to:


 Determine if the prostate is enlarged
 Diagnose prostate cancer
o TRUS is done if the doctor suspects cancer because of an increased prostate-
specific antigen (PSA) level, abnormalities felt during digital rectal examination
(DRE) or certain symptoms are present, such as problems with urination.
 Help guide the doctor when taking biopsy samples of the prostate
 Help diagnose the cause of infertility in a man
 Help guide the doctor during treatments for prostate cancer
o TRUS is used to guide the placement of implants during brachytherapy.
o It may help the doctor perform cryosurgery.
o TRUS may be used to help deliver high-intensity focused ultrasound (HIFU).

Transvascular ultrasound: Transvascular imaging is defined as the acquisition of anatomic and


functional information of structures lying beyond the confines of a vascular conduit within which
the imaging device resides.
Transesophageal echocardiogram: The transducer is inserted into the esophagus to obtain
images of the heart.
Transvaginal ultrasound: The transducer is inserted into a woman's vagina to view the
uterus and ovaries.

Endoluminal Ultrasound:
 Diagnostic procedure of choice in evaluation of anorectal disorders.
 Best imaging modality for staging of rectal cancer.
 Invaluable in workup for fecal incontinence and suppurative anorectal conditions.
 3D ultrasound recently developed. Currently being evaluated in efficacy compared to 2D
ultrasound and MRI.
 Equipment and Technique: Endocavitary probe with rotating transducer
7or 10 MHz Transducer
 Latex Balloon covering the probe which is filled with water to distend the rectum.
 Phosphossoda enema for bowel prep:
o Left lateral decubitus position
o DRE and proctoscopy
o Wide bore proctoscope to facilitate positioning of probe above lesion.
o Balloon is filled with 30-60 cc of water and real time image obtained.

 Image Interpretation:
5 Layer Model

Beynon and Colleagues


 Three hyperechoic (white) layers separated by two hypoechoic (black) layers.
 1st White layer: Interface between balloon and mucosa
 2nd black layer: mucosa and muscularis
 3rd white layer: submucosa
 4th black layer: Muscularis propria
 5th white layer: interface between muscularis propria & fat

Accuracy of ERUS:

 Accurate in determining depth of invasion


 Moderately accurate in assessment of Lymph Node involvement.
 Significant learning curve
 Highly operator dependent

Factors for Misinterpretation:

 Close proximity to anal verge


 Inproper balloon inflation
 Artifact from air or stool
 Refraction artifacts
 Post-biopsy or surgical changes
 Hemorrhage
 Pedunculated tumors

e. Radio - active Iodine.

Introduction: There are 37 known isotopes of iodine (I) from 108I to 144I, but only one, 127I, is
stable. Its longest-lived radioactive isotope, 129I.
131
I (half life 8 days) is one of the common radioactive fission-products of nuclear fission, and is
thus produced inadvertently in very large amounts inside nuclear reactors.

Radioiodines I-123, I-124, I-125, and I-131 in medicine and biology:


Of the many isotopes of iodine, only two are typically used in a medical setting: iodine-
123 and iodine-131.
 Since 131I has both a beta and gamma decay mode, it can be used for radiotherapy or for
imaging.
 123
I, which has no beta activity, is more suited for imaging (such as CT scans using iodinated
contrast imaging) and less damaging internally to the patient.
 There are some situations in which iodine-124 and iodine-125 are used in medicine, also.
 Due to preferential uptake of iodine by the thyroid, radioiodine is extensively used in
imaging of and, in the case of I-131, destroying dysfunctional thyroid tissues.
 I – 131 is used to treat hyperthyroidism and thyroid cancer.
 Other types of tissue selectively take up certain iodine-131-containing tissue-targeting and
killing radiopharmaceutical agents (such as MIBG).
 Iodine-125 is the only other iodine radioisotope used in radiation therapy, but only as an
implanted capsule in brachytherapy,where the isotope never has a chance to be released for
chemical interaction with the body's tissues.
Side effects:
 Large or cumulative doses of radioactive iodine may cause infertility, which is the inability
to produce a child, especially in men.
 It is recommended that women avoid pregnancy for at least one year after radioactive iodine
treatment.
 There is a risk of secondary cancer with the use of I-131
The West Bengal University of Health Sciences

M.B.B.S. 3rd Professional Part – II (Supplementary) Examination, 2013

Subject: Surgery Time: 21/2 hrs.

Paper: II Marks: 60

Group –A

1. How do you classify goiter? Give an outline of investigations and management of a


solitary nodular goiter. 5+5+5=15

Answer. Classification of goiter


Outline of investigation and management protocol of a solitary thyroid nodule
Group – B

2.
a. Classify renal Neoplasm. Write clinical manifestations, investigations and
management of renal cell carcinoma in a 40 years old patient. 3+5+3+4 = 15

Answer. Classification of renal tumours:

Renal neoplasms
(according to Bailey and Love’s Short Practice of Surgery)

Benign neoplasms
• ■ Adenoma
• ■ Angioma
• ■ Angiomyolipoma
Malignant neoplasms
• ■ Wilms’ tumour (nephroblastoma in children)
• ■ Grawitz’s tumour (adenocarcinoma,
hypernephroma)
• ■ Transitional cell carcinoma of the renal pelvis and
• collecting system
• ■ Squamous carcinoma of the renal pelvis

For undergraduate students this classification is enough.

Students who are interested and can remember for those:


WHO Classification of kidney tumours:
• Familial renal cancer
• Renal cell tumors
• Malignant
• Clear cell renal cell carcinoma
• Multilocular clear cell renal cell carcinoma
• Papillary renal cell carcinoma
• Chromophobe renal cell carcinoma
• Carcinoma of the collecting ducts of Bellini
• Renal medulary carcinoma
• Xp11 translocation carcinomas
• Carcinoma associated with neuroblastoma
• Mucinous tubular and spindle cell carcinoma
• Benign
• Papillary adenoma
• Oncocytoma
• Renal cell carcinoma unclassified
• Neuroendocrine tumors
• Carcinoid
• Neuroendocrine carcinoma
• Primitive neuroectodermal tumor
• Neuroblastoma
• Phaeochromocytoma
• Other tumors
• Mesenchymal tumors
• Haematopoietic and lymphoid tumors
• Germ cell tumors
• Metastatic tumors
• Paediatric patients – Wilm’s tumour.

• Metanephrictumors
• Metanephricadenoma
• Metanephricadenofibroma
• Metanephricstromaltumors
• Mixed mesenchymaland epithelial tumors
• Cystic nephroma
• Mixed epithelial and stromaltumor
• Synovial sarcoma
• Nephroblastic tumors
• Nephrogenic rests
• Nephroblastoma
• Cystic partially differentiated nephroblastoma
Clinical features:

Signs and symptoms:

i. The classically described triad of gross hematuria, flank pain, and a palpable mass
occurs in only 7–10%% of patients and is frequently a manifestation of advanced
disease.
ii. Patients may also present with hematuria, dyspnea, cough, and bone pain which are
typically symptoms secondary to metastases.
iii. With the routine use of CT scanning for evaluation of nonspecific findings,
asymptomatic renal tumors are increasingly detected incidentally (>50%).

Paraneoplastic Syndromes: RCC is associated with a wide spectrum of paraneoplastic


syndromes including erythrocytosis, hypercalcemia, hypertension, and nonmetastatic hepatic
dysfunction. Overall, these manifestations can occur in 10–40% of patients with RCC.

Paraneoplastic syndromes
Syndrome associated with RCC Cause

Anaemia Haematuria, chronic disease


Polycythaemia Ectopic secretion of erythropoeitin
Hypertension (25%) Ectopic secretion of renin, renal artery
compression, or A-V fistula
Hypoglycaemia Ectopic secretion of insulin
Cushing's syndrome Ectopic secretion of ACTH
Hypercalcaemia Ectopic secretion of parathyroid hormone-
like substance
Gynaecomastia, amenorrhoea, reduced Ectopic secretion of gonadotrophins
libido, baldness
Stauffer's syndrome: hepatic dysfunction, Unknown; resolves in 60 -70% of patients
fever, anorexia post-nephrectomy
Investigations:

The diagnosis of renal cell carcinoma is often made by CT (and, less frequently, by intravenous
urography) performed as an initial step in the workup of hematuria, an enigmatic metastatic
lesion, or suspicious laboratory findings.

i. Laboratory Findings:
 Microscopic urinalysis reveals hematuria in most patients.
 The erythrocyte sedimentation rate may be elevated but is nonspecific.
 Elevation of the hematocrit and levels of serum calcium, alkaline phosphatase, and
aminotransferases occur in less than 10% of patients. These findings nearly always resolve
with curative nephrectomy and thus are not usually signs of metastases.
 Anemia unrelated to blood loss occurs in 20% to 40% of patients, particularly those with
advanced disease.
ii. Imaging Studies:

 Ultrasonography: Abdominal ultrasonography can define the mass as a benign simple cyst
or a solid mass in 90% to 95% of cases. Abdominal ultrasound can also identify a vena
caval tumor thrombus and its cephalad extent in the cava.
 Isotope Scanning: Occasionally, a renal mass is suspected on intravenous urography but is
equivocal or not seen on ultrasound. In these cases, a renal cortical isotope scanning agent
such as technetium-99m DMSA is helpful.
 CT Scan: CT scan is the diagnostic procedure of choice when a solid renal mass is noted on
ultrasound.
o CT scan accurately delineates renal cell carcinoma in over 95% of cases.
o CT scan is also helpful in local staging and can reveal tumor penetration of perinephric
fat; enlargement of local hilar lymph nodes, indicating metastases; or tumor thrombi in
the renal vein or inferior vena cava.
o CT angiography can delineate the renal vasculature, which is helpful in surgical
planning for partial nephrectomies.
 MRI: MRI is not more accurate than CT and is much more expensive. It is, however, the
most accurate noninvasive means of detecting renal vein or vena caval thrombi.
Magnetic resonance angiography (MRA) has become particularly useful for mapping
the blood supply and the relationship to adjacent structures in candidates for partial
nephrectomy.

 Other Diagnostic or Staging Techniques:


o Isotopic bone scanning is useful in patients with bone pain, elevated alkaline phosphatase,
or known metastases.
o Chest x-ray is sufficient if negative, but if equivocal, then CT scan of the chest can be
used to detect metastases.
o Occasionally, aspiration cytology of the mass can be useful in an enigmatic case.
Previously, such procedures were discouraged. The diagnosis is most often made by
noninvasive means, and needle aspiration is required only in indeterminate cases (< 10%).

Treatment:

 Staging should be done for proper treatment


 If the disease is confined within the renal fascia (Gerota's fascia) or limited to nonadherent
renal vein or vena caval tumor thrombi (stages T1, T2, and T3a) are best treated by radical
nephrectomy. This involves en bloc removal of the kidney and surrounding Gerota fascia
(including the ipsilateral adrenal), the renal hilar lymph nodes, and the proximal half of the
ureter.
 Para-aortic node dissection has not been proven beneficial and is not routinely performed.
 If the tumour is very large with a normal contralateral kidney, radical nephrectomy is
recommended.
 If the renal vein or the inferior vena cava is invaded, early control of the cava above and
below the tumour extension is taken. If there is extension into the thorax, the cardiac team
may be needed to put the patient on cardiac bypass so that tumour can, if necessary, be
removed from the right side of the heart.

 Different surgical approaches:


Most common operation for renal cell carcinoma is nephrectomy.

i. The transperitoneal abdominal approach is usually considered superior in renal


malignancy for the better access it gives to the great vessels.
ii. For very large upper renal tumours, or for retroperitoneal sarcomas, a thoracoabdominal
approach, with or without liver mobilization provides very extensive access.
iii. Alternatively retroperitoneal loin approach may also be preferable.

Nephrectomy: may involve removing either an entire organ and surrounding tissue or a small
part of the kidney.

o Partial Nephrectomy: In partial nephrectomy, which is also referred to as kidney-sparing


surgery, only the diseased or infected portion of the kidney is removed. The healthy kidney
tissue is left in place as much as possible.

o Radical Nephrectomy: Radical nephrectomy involves the removal of the entire kidney, the
adrenal gland, the ureter and the fatty tissue that surrounds the kidney.
o Simple Nephrectomy: Simple Nephrectomy is done for a living donor transplant purposes
that requires the removal of the kidney and a section of the attached ureter.

o Open Nephrectomy: This is the traditional surgery and the donor or patient may be under
general anesthesia. A 10 to 20-inch incision is done in the side or abdomen and a lower rib
may be removed.

o Laparoscopic Nephrectomy: This is a minimally invasive surgery and entails less blood loss.

 Immunotherapy:

 Biologic response modifiers:

o Interferon alfa -has had a 15% to 20% response rate.


o Interleukin-2 (IL-2) - a T-cell growth factor.Recombinant IL-2 is the only agent approved
by the US Food and Drug Administration for patients with advanced renal carcinoma.

 Newer biologic agents:


o Bevacizumab is a monoclonal antibody that binds and inactivates VEGF A. It has shown
the ability to yield partial responses, delay disease progression, and improve survival in
patients with advanced renal cancer.
o Sunitinib: a multi-targeted receptor tyrosine kinase (RTK) inhibitor yields higher response
rates.
o Sorefenib is another agent that can inhibit VEGF and PDGF and has halted disease
progression.
o Temsirolimus is an agent which inhibits mTOR, a kinase involved in the VEGF pathway to
promote angiogenesis. Temsirolimus has been demonstrated to prolong survival in patients
with advanced renal cancer when used as first-line therapy either alone or in combination
with interferon.
Undergraduates have the luxury to forget the following classification:

TNM Classification System for Renal Cell Carcinoma


Or
b. Discuss the clinical features of pheochromocytoma. How will you diagnose this
condition? Give an outline of management. 5+5+5 = 15

Answer: Pheochromocytoma is said to follow the rule of ten:

o 10% are multifocal;


o 10% are bilateral;
o 10% are extra-adrenal;
o 10& are malignant;
o 10% occur in children.

Clinical features: Symptoms and signs are caused by catecholamine excess and are
typically intermittent.

 Excess catecholamine secretion leads to characteristic episodes of;


o headache;
o sweating;
o palpitations;
o paroxysmal hypertension, tachydysrhythmias, and a feeling of ‘impending
doom or death’ may also occur.
 Attacks can be triggered by activities causing mechanical pressure on the tumour (e.g.
physical exercise, defecation, intercourse), by ingestion of alcohol, labour, general
anaesthesia, and surgical procedures.
 Only 50% of patients have persistent hypertension. The other 50% have normal blood
pressure or are hypotensive between the acute episodes.

Clinical signs of phaeochromocytoma:

 Hypertension
o Paroxysmal
o Continuous
 Headache
 Sweating
 Palpitation
 Pallor
 Weight loss
 Hyperglycaemia
 Nausea
 Psychological effects
According to Sabiston’s Textbook of Surgery 19th edition:

Algorithm for the diagnosis, localization, and management of pheochromocytoma.


Diagnosis and investigations:

Consider the diagnosis in patients with characteristic paroxysmal episodes, in those with
unusually labile or intermitted hypertension, in those with a family history of
phaeochromocytoma or related conditions (see MEN syndromes),and in hypertensive children.

 24h urine collection and assessment for VMA and noradrenaline is most accurate for
diagnosis (97% sensitive).
 Clonidine suppression test (failure of urine levels to fall after clonidine dose) confirms the
diagnosis where urine levels are borderline.
 Provocative testing (e.g. stimulation with bolus IV glucagons) is rarely necessary and not
without risk.

Localizing studies:
 Thoraco-abdominal CT or MRI scanning: first-line test especially for adrenal and
sympathetic chain tumours.
 MIBG (meta-iodo-benzyl-guanidine) scanning localizes extra-adrenal sites not seen on CT
or MRI.

Treatment

Medical treatment

 It is imperative to control the blood pressure prior to contemplating any surgical


intervention.
 Alpha-blockers are given(e.g. phenoxybenzamine 10mg bd/tds up to the maximum dose
tolerated) until hypertension controlled.
 Beta-blockade (e.g. propranolol) can be added after hypertension is controlled to control the
beta-adrenergic effects (tachycardia).
 Alternative treatments with doxazosin (alpha/beta blocker) or calcium channel blockers
have been described but are not widely used.

Surgical treatment

 The principle of surgery is complete resection of the tumour (with clear negative margins if
suspected of malignancy).
 Laparoscopic adrenalectomy is the treatment of choice for smaller adrenal tumours (< 8-
10cm); open adrenalectomy for larger tumours.
 Local or radical excision are appropriate for extra-adrenal tumours.
Postoperative

 Patients should be observed for 24 hours in the intensive care or high dependency unit as
hypovolaemia and hypoglycaemia may occur.
 Lifelong yearly biochemical tests should be performed to identify recurrent, metastatic or
metachronous phaeochromocytoma.

Group – C

3. Write short notes on (any three)


3x5
a. Posterior urethral valve.
b. Hutchinson’s pupil.
c. Empyema thoracis.
d. Spinal anaesthesia.
e. Types of Renal stone.

a. Answer. Posterior urethral valves (PUVs)

Definition: Abnormal congenital mucosal folds in the prostatic (posterior) urethra causing lower
urinary tract obstruction.

Classification

 Type I (90-95%) Membranes arise from the distal lateral aspect of the verumontanum,1
which extend distally and anteriorly to fuse in the midline.
 Type II Longitudinal folds extending from the verumontanum to bladder neck.
 Type III A ring-like membrane found distal to the verumontanum.

Incidence: 1 in >5000 males

Aetiology: Normal male urethra has small, paired lateral folds (plicae colliculi) which are found
between the lateral, distal edge of verumontanum and lateral urethral wall. PUVs probably
represent a congenital overgrowth of these folds, due to abnormal insertion of Wolffian ducts
into the posterior urethra during fetal development.

Presentation

Prenatal US features
 Bilateral hydroureteronephrosis; dilated bladder and posterior urethra (keyhole sign);
thick walled bladder; oligohydramnios (reduced amniotic fluid); and renal dysplasia.
 Early features are associated with poor prognosis.

Newborn and infants


 Respiratory distress (secondary to pulmonary hypoplasia);
 Palpable abdominal mass (hydronephrotic kidney or distended bladder);
 Ascites;
 UTI;
 Electrolyte abnormalities;
 Failure to thrive.

Older children
 Milder cases may present later with recurrent UTI; poor urinary stream; incomplete
bladder emptying; poor growth and incontinence.
 Risk of renal failure, vesicoureteric reflux, and voiding dysfunction (overactive or
underactive bladder), also described as valve bladder syndrome.

Associated features:
 Urinary ascites.
 Vesicoureteral reflux with renal dysplasia.
 Formation of bladder diverticuli.

Investigation:

 Ultrasound scan of renal tract.


 Micturating cystourethrogram: distended posterior urethra (shield-shaped); partially filled
anterior urethra; bladder neck hypertrophy; lucencies representing valve leaflets; thick
walled bladder (±diverticuli); incomplete bladder emptying; reflux (50%).
 Isotope renal scan (MAG-3, DMSA): assesses renal function.
 Videourodynamics: allows diagnosis of associated voiding dysfunction.

Management;
 Commence prophylactic antibiotics immediately, check serum electrolyes, and drain the
bladder with a paediatric feeding tube.
 If there is improvement, cystoscopy and transurethral ablation of valve is recommended.
 If upper tracts remain dilated with raised creatinine after bladder drainage, a temporary
cutaneous vesicostomy is indicated (communicating stoma between the bladder dome and
suprapubic abdominal wall, allowing free drainage of urine). An alternative is
ureterostomy drainage.
 Valve ablation is performed at a later stage.
Prognosis:
35% poor renal function; 20% develop end-stage renal failure.
(Write if you can remember)

b. Hutchinson’s pupil.

Introduction:Hutchinson's pupil is a clinical sign in which the pupil on the side of an


intracranial mass lesion is dilated and unreactive to light, due to compression of the oculomotor
nerve on that side.
The sign is named after Sir Jonathan Hutchinson.
Cause: These can be due to concussion injury to the brain and is associated with subdural
haemorrhage and unconsciousness.

Pathophysiology: The parasympathetic fibers to the pupil are responsible for pupillary
constriction. The fibers pass through the periphery of the oculomotor nerve, and hence are the
first to be affected in case of compression of the nerve.

Stages:
 In Stage 1, the parasympathetic fibers on the side of injury are irritated, leading to
constriction of pupil on that side.
 In stage 2, the parasympathetic fibers on the side of injury are paralysed, leading to
dilatation of pupil. The fibers on the opposite oculomotor nerve are irritated, leading to
constriction on opposite side.
 In stage 3, the parasympathetic fibers on both sides are paralysed - leading to bilateral
pupillary dilatation. Pupils become fixed. This indicates grave prognosis.

c. Empyema thoracis.

Introduction:
Empyema thoracis is the end stage of pleural infection from any cause.

Conditions that predispose to empyema formation


When empyema presents de novo it usually follows pneumonia and three phases are described:

Stages Features Treatment


1. Exudative In the exudative phase, there is protein-  At this stage, antibiotics may be all
phase rich (>30 g/L) effusion. that is required.
If this becomes infected with the  Aspiration or drainage to dryness
organisms from the in addition is preferred.
lung (typically Streptococcus milleri and
Haemophilus influenza in children).
2. Fibropurulent Over the next days, the fluid thickens to  Drainage at this stage is prudent as
stage what is known as the fibrinopurulent antibiotics alone are unlikely to be
phase. curative.

3. Organising The organising phase causes the lung to be  Surgical management may be
phase trapped by a thick peel or ‘cortex’. required.

.
Diagnosis:

 The initial investigations for suspected empyema remains chest X-ray, although it cannot
differentiate an empyema from uninfected parapneumonic effusion.
 Ultrasound must be used to confirm the presence of a pleural fluid collection and can be used
to estimate the size of the effusion, differentiate between free and loculated pleural fluid and
guide.
 Thoracocentesis if necessary.
 Chest CT and MRI do not provide additional information in most cases and should therefore
not be performed routinely.

Treatment:

 Pleural fluid drainage.


 Antibiotic therapy.

d. Spinal anaesthesia.

Introduction: Spinal anaesthesia (or spinal anesthesia), also called spinal analgesia,spinal
block or subarachnoid block (SAB), is a form of regional anaesthesia involving injection of
a local anaesthetic into thes ubarachnoid space, generally through a fine needle.

Injected substances:
 Bupivacaine (Marcaine) is the local anaesthetic most commonly used.
 Although lidocaine (lignocaine), tetracaine, procaine,ropivacaine, levobupivicaine, prilocain
e and cinchocaine may also be used.
 Commonly opioids are added to improve the block and provide post-operative pain relief.
Examples: include morphine, fentanyl, diamorphine or buprenorphine.
 Non-opioids like clonidine may also be added to prolong the duration of analgesia.

Mechanism:
 Regardless of the anaesthetic agent (drug) used, the desired effect is to block the
transmission of afferent nerve signals from peripheral nociceptors. Sensory signals from the
site are blocked, thereby eliminating pain.
 The degree of neuronal blockade depends on the amount and concentration of local
anaesthetic used and the properties of the axon.
 Thin unmyelinated C-fibres associated with pain are blocked first, while thick, heavily
myelinated A-alpha motor neurons are blocked moderately.
 Heavily myelinated, small preganglionic sympathetic fibers are blocked first. The desired
result is total numbness of the area.
 A pressure sensation is permissible and often occurs due to incomplete blockade of the
thicker A-beta mechanoreceptors. This allows surgical procedures to be performed with no
painful sensation to the person undergoing the procedure.

Limitations:
 Spinal anaesthetics are typically limited to procedures involving most structures below the
upper abdomen.
 To administer a spinal anaesthetic to higher levels may affect the ability to breathe by
paralysing the intercostal respiratory muscles, or even the diaphragm in extreme cases
(called a "high spinal", or a "total spinal", with which consciousness is lost), as well as the
body's ability to control the heart rate via the cardiac accelerator fibres.
 Also, injection of spinal anaesthesia higher than the level of L1 can cause damage to the
spinal cord, and is therefore usually not done.

e. Types of renal stone:

Radio-opaque:

 Calcium phosphate stones are the most radiodense stones, being almost as dense as bone.
A phosphate calculus (calcium phosphate often with ammonium magnesium phosphate
(struvite)) is smooth and dirty white. Magnesium ammonium phosphate (struvite) stones
are less radiodense than calcium containing stones.

It grows in alkaline urine, especially when urea-splitting Proteus organisms are present.
The calculus may enlarge to fill most of the collecting system, forming a stag-horn
calculus.

 Calcium oxalate stones are slightly less radiodense. Oxalate stones are irregular with sharp
projections. A calcium oxalate monohydrate stone is hard and radiodense.

Relatively radiolucent
 Cystine stones are relatively radiodense because they contain sulphur.
An uncommon congenital error of metabolism leads to cystinuria.
Cystine stones are often multiple and may grow to form a cast of the collecting system.
They are resistant to ESWL.
Completely radiolucent:

 Uric acid: These are hard, smooth and often multiple and multifaceted. Pure uric acid stones
are radiolucent.
 Triamterene:
 Xanthine: Occurs in patients with xanthine oxidase deficiency.
 Indinavir: found in AIDS patients.

Group – D

4. Answer briefly on (any three): 3x5

a. Causes of scrotal swelling.


b. Pressure sore.
c. Hypospadius.
d. Keloid.
e. Tension pneumothorax.

a. Answer. Causes of scrotal swelling:


i. Inguinal Hernia.
ii. Hydrocele.
iii. Haematocele.
iv. Spermatocele.
v. Varicolele.
vi. Torsion of testes.
vii. Epididymitis.
viii. Epididymal cyst.
ix. Orchitis.
x. Mumps.
xi. Testicular tumour.
xii. Filariasis.
xiii. Congestive cardiac failure.
xiv. Operative interventions to genitalia.

b. Answer: Pressure sore/ bed sores.:

Bedsores — also called pressure sores or pressure ulcers — are injuries to skin and underlying
tissue resulting from prolonged pressure on the skin. Bedsores most often develop on skin that
covers bony areas of the body, such as the heels, ankles, hips and tailbone.
Bedsores fall into one of four stages based on their severity.

Stage I Stage II Stage III Stage IV

The skin is not The outer layer of skin The ulcer is a deep Ulcer shows large-
broken. (epidermis) and part wound. The ulcer scale loss of tissue
of the underlying layer looks crater-like.
of skin (dermis) is
damaged or lost.

The skin appears red The wound may be The loss of skin The wound may
on people with lighter shallow and pinkish or usually exposes some expose muscle, bone
skin color red. fat. or tendons.

On people with darker The wound may look The bottom of the The bottom of the
skin like a fluid-filled wound may have wound likely contains
blister or a ruptured some yellowish dead dead tissue that's
blister. tissue. yellowish or dark and
crusty.

The site may be tender The damage often


extends beyond the
primary wound below
layers of healthy skin.

Unstageable

A pressure ulcer is considered unstageable if its surface is covered with yellow, brown, black or
dead tissue. It’s not possible to see how deep the wound is.

Common sites of pressure sores

For people who use a wheelchair, pressure sores often occur on skin over the following sites:

 Tailbone or buttocks
 Shoulder blades and spine
 Backs of arms and legs where they rest against the chair
For people who are confined to a bed, common sites include the following:

 Back or sides of the head


 Rim of the ears
 Shoulders or shoulder blades
 Hip, lower back or tailbone
 Heels, ankles and skin behind the knees
Bedsores are caused by pressure against the skin that limits blood flow to the skin and nearby
tissues.

Three primary contributing factors are:

 Sustained pressure.

 Friction.

 Shear.

Risk factors:
People are at risk of developing pressure sores if they have difficulty moving and are unable to
easily change position while seated or in bed. Immobility may be due to:

 Generally poor health or weakness


 Paralysis
 Injury or illness that requires bed rest or wheelchair use
 Recovery after surgery
 Sedation
 Coma

Other factors that increase the risk of pressure sores include:

 Advancing Age.  Bowel incontinence. .


 Lack of sensory perception.  Medical conditions affecting blood flow.
 Weight loss.  Smoking.
 Poor nutrition and hydration  Limited alertness.
 Excess moisture or dryness.  Muscle spasms.
Complications of pressure ulcers include:

 Sepsis. Sepsis occurs when bacteria enter the bloodstream through broken skin and spread
throughout the body. It's a rapidly progressing, life-threatening condition that can cause organ
failure.
 Cellulitis. Cellulitis can lead to life-threatening complications.
 Bone and joint infections. An infection from a pressure sore can burrow into joints and
bones. Joint infections (septic arthritis) can damage cartilage and tissue. Bone infections
(osteomyelitis) may reduce the function of joints and limbs. Such infections can lead to life-
threatening complications.
 Cancer. Another complication is the development of a type of squamous cell carcinoma that
develops in chronic, nonhealing wounds (Marjolin ulcer). This type of cancer is aggressive
and usually requires surgery.
Evaluating a bedsore: To evaluate a bedsore:

 Determine the size and depth of the ulcer


 Check for bleeding, fluids or debris in the wound that can indicate severe infection
 Try to detect odors indicating an infection or dead tissue
 Check the area around the wound for signs of spreading tissue damage or infection
 Check for other pressure sores on the body

Investigations:

 Blood tests: complete haemogram, LFT, sugar, urea , creatininie, blood culture.
 Tissue cultures to diagnose a bacterial or fungal infection in a wound that doesn't heal with
treatment or is already at stage IV
 Tissue cultures to check for cancerous tissue in a chronic, nonhealing wound
Treatment:

Stage I and II bedsores usually heal within several weeks to months with conservative care of the
wound and ongoing, appropriate general care. Stage III and IV bedsores are more difficult to
treat.

Reducing pressure

The first step in treating a bedsore is reducing the pressure that caused it. Strategies include the
following:

 Repositioning. If you have a pressure sore, you need to be repositioned regularly and placed
in correct positions. If you use a wheelchair, try shifting your weight every 15 minutes or so.
Ask for help with repositioning every hour. If you're confined to a bed, change positions every
two hours.

 Using support surfaces. Use a mattress, bed and special cushions that help you lie in an
appropriate position, relieve pressure on any sores and protect vulnerable skin. If you are in a
wheelchair, use a cushion. Styles include foam, air filled and water filled. Select one that suits
your condition, body type and mobility.
Cleaning and dressing wounds

Care that helps with healing of the wound includes the following:

 Cleaning.
 Applying dressings. Removing damaged tissue

To heal properly, wounds need to be free of damaged, dead or infected tissue.

 Surgical debridement involves cutting away dead tissue.


 Mechanical debridement .
 Autolytic debridement .
 Enzymatic debridement .

Other interventions

Other interventions that may be used are:

 Pain management.
 Antibiotics.
 A healthy diet. To promote wound healing, your doctor or dietitian may recommend an
increase in calories and fluids, a high-protein diet, and an increase in foods rich in vitamins
and minerals. You may be advised to take dietary supplements, such as vitamin C and zinc.
 Management of incontinence.
 Muscle spasm relief. Muscle relaxants — such as diazepam (Valium), tizanidine (Zanaflex),
dantrolene (Dantrium) and baclofen (Gablofen, Lioresal) — may inhibit muscle spasms and
help sores heal.
 Negative pressure therapy (vacuum-assisted closure, or VAC).

Surgery:

A pressure sore that fails to heal may require surgery. The goals of surgery include improving
the hygiene and appearance of the sore, preventing or treating infection, reducing fluid loss
through the wound, and lowering the risk of cancer.
In general, most pressure sores are repaired using a pad of your muscle, skin or other tissue to
cover the wound and cushion the affected bone (flap reconstruction).

c. Answer: Hypospadius:

Definition
Hypospadias is a congenital deformity where the opening of the urethra (the meatus) occurs on
the underside (ventral) part of the penis, anywhere from the glans to the perineum.
o It is often associated with hoodedforeskin and chordee (ventral curvature of the penile
shaft).
o It occurs in 1 in 250 live male births. Classification
Hypospadias can be classified according to the anatomical location of the urethral meatus
 Anterior (or distal): glandular, coronal, and subcoronal (~50%)
 Middle: distal penile, midshaft, and proximal penile (~30%)
 Posterior (or proximal): penoscrotal, scrotal, and perineal (~20%)

Aetiology:

Hypospadias results from incomplete closure of urethral folds on the underside of the penis
during embryological development. This is related to a defect in production or metabolism of
fetal androgens, or the number and sensitivity of androgen receptors in the tissues. Chordee are
caused by abnormal urethral plate development, and the hoodedforeskin is due to failed
formation of the glandular urethra and fusion of the preputial folds (resulting in a lack of ventral
foreskin but an excess of dorsal tissue).

Diagnosis:

o A full clinical examination will make the diagnosis. However, it is also important to seek
out associated abnormalities which will need treatment (undescended testes, inguinal
hernias, and hydroceles).
o Patients with absent testes and severe hypospadias should undergo chromosomal and
endocrine investigation to exclude intersex conditions.

Treatment:

o Surgery is indicated where deformity is severe, interferes with voiding, or is predicted to


interfere with sexual function.
o Surgery is now performed between 6-12 months of age.
o Local application of testosterone for 1 month pre-operatively can help increase tissue
size.
o Surgery aims to correct penile curvature (orthoplasty), reconstruct a new urethra, and
bring the new meatus to the tip of the glans using urethroplasy, glanuloplasty, and
meatoplasty techniques. Severe cases may require staged procedures.
o Common operations for anterior hypospadias include meatal advancement and
glanuloplasty (MAGPI), meatal-based flaps (Mathieu procedure), and tubularization of
the urethral plate. Posterior defects require free grafts (buccal mucosa), onlay grafts, and
preputial transfer flaps.

Complications:

1.Bleeding, 4.Meatal 7.Urethral


2.Infection, Diverticulum,
3.Urethral 5.Stenosis,
Strictures And
6.Urethrocutaneous
Fistula, 8. Failed procedures
, requiring re-operation.
d. Answer. Keloid:

Introduction:
A keloid is the formation of a type of scar which, depending on its maturity, is composed mainly
of either type III (early) or type I (late) collagen. It is a result of an overgrowth of granulation
tissue (collagen type 3) at the site of a healed skin injury which is then slowly replaced by
collagen type 1.
Keloids are firm, rubbery lesions or shiny, fibrous nodules, and can vary from pink to the colour
of the patient's flesh or red to dark brown in colour.
Keloids should not be confused with hypertrophic scars, which are raised scars that do not grow
beyond the boundaries of the original wound.
Clinical features:
 Keloids expand in claw-like growths over normal skin.
 They have the capability to hurt with a needle-like pain or to itch without warning,
although the degree of sensation varies from person to person.
 If the keloid becomes infected, it may ulcerate.
Location:

 Keloids can develop in any place where skin trauma has occurred.
 They can be the result of pimples, insect bites, scratching, burns, or other skin injury.
Keloid scars can develop after surgery.
 They are more common in some sites, such as the central chest (from a sternotomy), the
back and shoulders (usually resulting from acne), and the ear lobes (from ear piercings).
They can also occur on body piercings.
 The most common spots are earlobes, arms, pelvic region, and over the collar bone.

Causes:

 Most skin injury types can contribute to keloid scarring. This includes acne scars, burns,
chickenpox scars, ear piercing, scratches, surgical cuts or vaccination sites.

 Keloid scarring is common in young people between the ages of 10 and 20, and among
African American, Asian, and Hispanic persons.
 Keloids tend to have a genetic component, which means you are more likely to have
keloids if one or both of your parents has them.
Treatment:

 The best treatment is prevention in patients with a known predisposition. This includes
preventing unnecessary trauma or surgery (including ear piercing, elective mole
removal), whenever possible. Any skin problems in predisposed individuals (e.g., acne,
infections) should be treated as early as possible to minimize areas of inflammation.
 The most effective treatment is superficial external beam radiotherapy (SRT), which can
achieve cure rates of up to 90%.
 Additionally, intralesional injection with a corticosteroid does appear to aid in the
reduction of inflammation and pruritis.
 Cryotherapy or Cryosurgery is an application of extreme cold to treat keloids. This
treatment method is easy-to-perform and has shown results with least chance of
recurrence.
e. Answer. Tension pneumothorax
Introduction: Pneumothorax is the presence of air outside the lung, within the pleural
space.
 Tension pneumothorax is when (independent of aetiology) there is a build-up of
positive pressure within the hemithorax, to the extent that the lung is completely
collapsed, the diaphragm is flattened and the mediastinum is distorted and,
eventually, the venous return to the heart is compromised.
 Any pleural breach is inherently valve-like because air will find its way out
through the alveoli but cannot be drawn back in because the lung tissue collapses
around the hole in the pleura.

Risk factors:
 Chest trauma.
 Ruptured bullae of lung.
 Patients being mechanically ventilated following trauma are at particular
risk.
Symptoms
 Sudden chest pain.
 Shortness of breath.
 Chest tightnes.
 Easy fatigue.
 Bluish color of the skin due to lack of oxygen.
 Rapid heart rate.
 Low blood pressure.
 Decreased mental alertness.
 Decreased consciousness.
 Rapid breathing.
 Bulging (distended) veins in the neck

Tests used to diagnose tension pneumothorax include:

 A chest x-ray will show - Deviation of the trachea away from the side of the tension.

Shift of the mediastinum.

Depression of the hemi-diaphragm

 Arterial blood gases


 Electrocardiogram

Treatment
 If the patient has symptoms of tension pneumothorax, get immediate medical treatment.
 Treatment is removal the air from the pleural space, allowing the lung to re-expand.
 Needle Thoracostomy: In an emergency, a small needle (such as a standard intravenous
needle) may be placed into the chest cavity through the ribs to relieve pressure.
 The standard treatment is insertion of a chest tube. This allows the lung to re-expand. As the
lung heals and stops leaking air, the vacuum is turned down and then the chest tube is
removed.
 Surgery may be needed if the problem happens again, or if the lung does not re-expand after
5 days with a chest tube in place.
The West Bengal University of Health Sciences
M.B.B.S. 3rd Professional Part – II Examination (Supplementary), 2012
Subject: Surgery Time: 21/2 hrs.
Paper: II Marks: 60
Group –A
1. a) What are the different types of nipple discharges with their clinical
importance?How would you manage stage I carcinoma breast in lady aged 40
years. 6+9 = 15
Answer. Discharges from the nipple:
Discharge Discharge from a single duct Discharge from more than one duct
from the
surface

Paget’s Blood-stained Serous Blood- Black Purulent Serous Milk


disease (any colour) stained or
green
Skin diseases
(eczema, Intraduct Fibrocystic Carcinoma Duct Infection Fibrocystic Lactation
psoriasis) papilloma disease ectasia disease Rare causes
Ectasia (hypothyroidis
Rare causes Intraduct Duct ectasia Duct ectasia m, pituitary
(e.g. chancre) carcinoma Fibrocystic Carcinoma tumour)
Carcinoma disease
Duct ectasia

Management of stage I carcinoma breast in lady aged 40 years.


Diagnosis should be confirmed by:

Triple assessment Metastatic work


 Clinical examination  Chest X-ray.
 FNAC/Core needle biopsy  USG – whole abdomen.
 Mammography  CT scan brain.

Treatment:
 Total mastectomy / lumpectomy with or without radiation therapy is the treatment of
stage I cancer. The incidence of ipsilateral breast cancer recurrence (in-breast
recurrence) is higher in the lumpectomy group not receiving radiation therapy.
 Currently, mastectomy with assessment of axillary lymph node status and breast
conservation (lumpectomy with assessment of axillary lymph node status and radiation
therapy) are considered equivalent treatments for stage I breast cancer.

Axillary lymphadenopathy or metastatic disease in a sentinel axillary lymph node


necessitates an axillary lymph node dissection.

Breast conservation is considered for all patients because of the important cosmetic
advantages.

Relative contraindications to breast conservation therapy include

 Prior radiation therapy to the breast or chest wall;


 Involved surgical margins or unknown margin status following re-excision;
 Multicentric disease; and
 Scleroderma or other connective-tissue disease..

 Adjuvant chemotherapy for early invasive breast cancer is considered for

 All node-positive cancers.


 All cancers that are larger than 1 cm in size.
 Node-negative cancers larger than 0.5 cm in size when adverse prognostic features
are present.

o Adverse prognostic factors include

 Blood vessel or lymph vessel invasion,


 High nuclear grade, high histologic grade,
 HER2/neu over - expression,
 Negative hormone receptor status.

 Tamoxifen therapy is considered for hormone receptor–positive women with cancers


that are larger than 1 cm in size.
 HER2/neu expression is determined for all newly diagnosed patients with breast cancer
to provide prognostic information in patients with node-negative breast cancer, predict
the relative efficacy of various chemotherapy regimens, and predict benefit from
Trastuzumab (Herceptin) in women with metastatic or recurrent breast cancer.

Or
b) A 65 years old man presents to the emergency with acute retention of urine.
How would you investigate and manage the patient? 7+8=15
Answer.
Initial management:
I. Check the blood pressure, pulse rate and hydration of the patient.
II. Palpate the bladder – whether full or not.
III. Drain the bladder – by Foley’s catheterisation - if failed – Suprapubic catheterisation.
IV. Antibiotics.
V. Analgesics.
VI. Try to find out the actual cause.

Causes of acute retention of urine in this patient:

Obstructive:
 Benign prostatic hyperplasia
 Urethral Strictures
 Bladder calculi
 Faecal Impactation
 Phimosis / paraphimosis
 Benign/malignant pelvic masses
 Meatal Stenosis
 Stone in the urethral meatus

Infectious and Inflammatory Causes:

 Prostatitis
 Prostatic abscess
 Balantitis
 Cystitis
 Bilharziasis
 Herpes simplex virus

Pharmacologic Causes:

 Drugs with anticholingeric properties eg: tricylic antidepressants (amitriptyline)


 Opioids
 Sympathomimetic drugs eg: oral decongestants containing Ephedrine.
 NSAIDs
 Antiparkinsonian agents (levodopa)
 Antipsychotics (chlopromazine)
 Muscle relaxants (Baclofen)
Neurologic Causes :
 Autonomic or peripheral nerve : Diabetes mellitus, Guillain-Barre syndrome,
Pernicious anaemia, radical pelvic surgery.
 Brain: CVA, MS, Tumour, Parkinson’s disease, concussion.
 Spinal cord : Haematoma / abscess / tumour, Cauda equine, spina bifida occulta.
Diagnosis:

 Proper history.
 Clinical examination: including digital rectal examination.
 Investigations:

Radiological investigations:
• KUB - to diagnose stone disese
• Abdominal US - to diagnose stone disese–, prostatic hypertrophy.
• IVP
• Helical CTU

Advantages over IVP:

» greater specificity (95%) and sensitivity (97%) for diagnosing ureteric


stones
» Can identify other, non-stone causes of flank pain.
» No need for contrast administration.
» Faster, taking just a few minutes
» the cost of CTU is equivalent to that of IVU
• MRI
 very accurate way of determining whether or not a stone is present
in the ureter
 very high cost
 Urethrography : for stricture.

Definitive treatment: treat the cause.

Group – B
2. Write short notes on (any three): 3x5

a. C.A.B.G
b. Tracheostomy.
c. Extradural Haemorrhage.
d. D.V.T.
e. Variants of melanoma
Answers.
a. C.A.B.G

Coronary artery bypass grafting or CABG is an open-heart procedure to relieve the blockages of
the arteries of the heart.

Indications for Coronary Artery Bypass Grafting (CABG) depend on various factors, mainly on
the individual's symptoms and severity of disease. Some of these include -

1. Left main artery disease or equivalent


2. Triple vessel disease
3. Abnormal Left Ventricular function.
4. Failed PTCA.
5. Immediately after Myocardial Infarction (to help perfusion of the viable myocardium).
6. Life threatening arrhythmias caused by a previous myocardial infarction.
7. Occlusion of grafts from previous CABGs.

Bypass grafting may be contraindicated in patients, for e.g. absent viable myocardium or the
artery that needs grafting is too small.

Choice of Grafts for Coronary Artery Bypass Grafting (CABG)

There are 3 types of grafts, venous grafts, arterial grafts and synthetic grafts.

Venous grafts:

During the operation, the surgical team will need to harvest and prepare a vein from elsewhere in
the body to use as a graft. The most commonly used vein is the long saphenous vein.

If the long saphenous vein is diseased with varicosities, the short saphenous vein can be used.

The saphenous vein graft is usally harvested by the assistant and can be taken through a long
skin incision or recently by the endoscope (EVH). Benefits of endoscopic vein harvesting
include more cosmetic incision and lower incidence of infection.

Arterial grafts:

Certain arteries in the body can be harvested without doing much problems to the areas they
were taken from. Here are examples of some of the arteries which can be harvested:

The left internal mammary artery: very commonly used vessel which is also called the left
internal thoracic artery. The very good thing about the left internal mammary artery is its
excellent patency rate in which there is about a 95 percent chance it will remain open for twenty
or more years later.
The right internal mammary: This artery usually reaches the right coronary, the left anterior
descending and some branches of the circumflex.

One of the disadvantages of harvesting the internal mammary arteries is that they are the arteries
which give blood supply to the breastbone. In some patients the healing of the breastbone, which
is opened during surgery, is delayed and can be infected as well.

Radial Artery:The patency rates with radial arteries so far indicate that it has a greater chance
of staying open longer than saphenous vein grafts but not quite as long as the left internal
mammary artery.

One of the disadvantages of the radial artery is that it can develop some spasm and narrowing
after harvesting. This can be overcome by bathing the artery in some solutions after harvesting
and by giving you special intravenous drugs in the ICU that will protect the artery from this
spasm

Gastroepiploic artery: The gastroepiploic artery can also be used as a free graft when both ends
are disconnected.

Synthetic grafts made of Dacron or other material have also been used. These grafts generally
work quite well in other areas of the body, particularly in the larger arteries and the aorta, but the
patency rates for coronary artery grafting have not been very good, and these synthetic arteries
are not routinely used.

A new concept, called total arterial revascularization, is now emerging in which arterial grafts
are only used because of their patency rates. However lot of surgeons still believe in using the
left internal mammary artery for bypassing the left anterior descending artery and using
saphenous vein grafts to bypass other coronary arteries.

Types:

On pump CABG

Off-Pump CABG (OPCAB).


Coronary Artery Bypass Surgery is a major surgical procedure and it carries some risks and
complications. These complications are higher in patients who are heavy smokers or if they have a
major problem with this other vital organs such as kidneys or lungs or have reduced blood supply to the
brain.

Infection and bleeding are examples of risks and complications any surgery may present.

Some of these complications may be uncommon. Complications after surgery include-

 Ankle swelling
 Heart Attack
 Stroke
 Emboli
 Arrhythmias
 Vein graft occlusion
 Kidney failure or temporary shutdown
 Death
 Stress and Depression - could be long-term problem.
 The risk of death from CABG is approx 1%. The main reason of death is heart attacks occuring
during or immediately after surgery.
 Heart attacks occurs in about 5% of the patients.
 Neurological complications are minimal and occurs more in women than men, especially
elderly.

b. Tracheostomy:

Definition: A tracheostomy is a surgical procedure to create an opening through the neck into the
trachea (windpipe). A tube is usually placed through this opening to provide an airway and to remove
secretions from the lungs.

Types of tubes:

 Cuffed or uncuffed - Most pediatric tubes do not have cuffs and inner cannulas due to smaller
diameter. Most adult tubes have inner cannula to allow for less frequent outer cannula changes.
 Metal (Jackson) or plastic (bivona, portex, shiley)
 Single or double cannula
 Fenestrated or non-fenestrated
 Short or long term
 Custom

Indications:

 facilitate prolonged mechanical ventilation and weaning


 by-pass upper airway obstruction (ex. sleep apnea, tumor …)
 maintain patent airway in severe head and neck injury or surgery
 airway anomalies
 secretion removal
 recurrent aspiration

Contraindications:

 coagulopathy
 enlarged thyroid
 abnormal airway anatomy
 lack of patient consent for procedure
 poor surgical candidate

Advantages:

 decreased work of breathing


 decreases the risk of upper airway complications due to endotracheal tube
 increase patient comfort and compliance
 improved oral hygiene
 oral movement for communication
 easier to stabilize and secure compared to endotracheal tubes
 increased mobility

Disadvantages:

 increased risk of infection


 impairs speech
 bypasses normal humidification system
 invasive surgical procedure
 may impair swallowing

c. Extradural Haemorrhage.
Epidural or extradural hematoma (haematoma) is a type of traumatic brain injury (TBI) in which
a buildup of blood occurs between the dura mater (the tough outer membrane of the central
nervous system) and the skull.
The dura mater also covers the spine, so epidural bleeds may also occur in the spinal column.
Often due to trauma, the condition is potentially deadly because the buildup of blood may
increasepressure in the intracranial space and compress delicate brain tissue.
The condition is present in one to three percent of head injuries.
Between 15 and 20% of patients with epidural hematomas die of the injury.
Clinical features:
o Epidural bleeds are extra-axial bleeds, occurring outside of the brain tissue.
o Epidural bleeding is rapid because it is usually from arteries, which are high pressure.
Epidural bleeds from arteries can grow until they reach their peak size at six to eight
hours post injury, spilling from 25 to 75 cubic centimeters of blood into the intracranial
space. As the hematoma expands, it strips the dura from the inside of the skull, causing
an intense headache.
o Epidural bleeds can become large and raise intracranial pressure, causing the brain to
shift, lose blood supply, or be crushed against the skull. Larger hematomas cause more
damage. Epidural bleeds can quickly expand and compress the brain stem, causing
unconsciousness, abnormal posturing, and abnormal pupil responses to light.
In the hallmark of epidural hematoma, patients may regain consciousness during what is
called a lucid interval, only to descend suddenly and rapidly into unconsciousness later.
o The lucid interval, which depends on the extent of the injury, is a key to diagnosing
epidural hemorrhage. If the patient is not treated with prompt surgical intervention, death
is likely to follow.
Imaging:
On images produced by CT scans and MRIs, epidural hematomas usually appear convex in
shape because their expansion stops at skull's sutures, where the dura mater is tightly
attached to the skull.
Thus they expand inward toward the brain rather than along the inside of the skull, as occurs
in subdural hematoma. The lens like shape of the hematoma leads the appearance of these
bleeds to be called "lentiform".
Epidural hematomas may occur in combination with subdural hematomas, or either may
occur alone. CT scans reveal subdural or epidural hematomas in 20% of unconscious
patients.

Causes:
o The interior of the skull has sharp ridges by which a moving brain can be injured.
o The main cause of epidural hematoma is usually traumatic, although spontaneous
hemorrhage is known to occur. Hemorrhages commonly result from acceleration-
deceleration trauma and transverse forces. 10% of epidural bleeds may be venous, due to
shearing injury fromrotational forces. Epidural hematoma commonly results from a blow
to the side of the head. The pterion region which overlies the middle meningeal artery is
relatively weak and prone to injury. Thus only 20 to 30% of epidural hematomas occur
outside the region of the temporal bone.
o The brain may be injured by prominences on the inside of the skull as it scrapes past
them.
o Epidural hematoma is usually found on the same side of the brain that was impacted by
the blow, but on very rare occasions it can be due to a contrecoup injury.
Treatment: As with other types of intracranial hematomas, the blood may be aspirated
surgically to remove the mass and reduce the pressure it puts on the brain.
The hematoma is evacuated through a burr hole or craniotomy. If transfer to a facility
with neurosurgery is prolonged trephination may be performed in the emergency
department.

d. D.V.T:
Causes and features

 May develop in association with abnormalities of the vein wall, blood flow, or
constituents of blood (Virchow's triad).
 May be due to vein compression or stasis (immobility, trauma, mass, bed rest, surgery,
paralysis, long distance travel including airline travel).
 May be due to inherited hypercoaguability (factor V Leiden, protein C, protein S, or
antithrombin insufficiency).
 May be due to acquired hypercoaguability (surgery, malignancy, polycythaemia,
smoking, hormone replacement therapy, OCP, dehydration).
 Severity may vary from isolated asymptomatic tibial/calf thrombosis to severe
iliofemoral segment thrombosis with phlegmasia caerulea dolens (venous gangrene).

Documented risk factors for DVT:

 Increasing Age  Cardiac Dysfunction  Cancer And Its Treatment


 Obesity  Paralysis  Nephrotic Syndrome
 Prolonged  Indwelling Central Venous Catheters  Major Surgery
Immobility  Previous Venous Thromboembolism  Pregnancy Or Estrogen Use.
 Varicose Veins  Inflammatory Bowel Disease  Trauma
 Stroke

Clinical features

 Clinical manifestations may be absent.


 Local features of venous engorgement and stasis:
 limb swelling;
 pain;
 erythema and warmth to the touch;
 mild fever and tachycardia resulting from release of inflammatory mediators;
 Homan's sign calf pain on dorsiflexion of the foot is very unreliable and should not be
performed.
 Complications:
 pulmonary embolism;
 venous gangrene (phlegmasia dolens).

Diagnosis and investigations

 Aim to confirm presence and extent of thrombosis (to decide on necessity and type of
treatment, risk of embolization).
 Fibrin, Fibrinogen Assays. The basis of fibrin or fibrinogen can be assayed by measuring the
degradation of intravascular fibrin. The D-dimer test measures cross-linked degradation
products, which is a surrogate of plasmin's activity on fibrin.
 Duplex scan: investigation of choice.
 VQ scan: If suspicion or evidence of pulmonary embolism.
 CT pulmonary angiography (CTPA): safest, most sensitive, and most specific investigation
for suspected pulmonary embolism.

Treatment

 Prophylaxis.
 Conservative measures: bed rest, elevation, and good hydration.
 Uncomplicated DVT: low molecular weight heparin (LMWH), initially in hospital; may
be given on an outpatient basis via a dedicated DVT clinic. Subsequent treatment is with
oral anticoagulation with warfarin for 3-6 months.
 Complicated DVT: initially with IV unfractionated heparin (UFH) whilst converting to
oral anticoagulation with warfarin.
 Thrombolysis or surgical thrombectomy are reserved for severe thrombosis with venous
gangrene.
 Vena caval filter percutaneously inserted via jugular vein into infrarenal IVC to catch
thromboemboli and prevent PE.

e. Variants of melanoma:

Superficial spreading Nodular melanoma Lentigo maligna Acral lentiginous


melanoma (SSM) melanoma melanoma
Most common form of Most aggressive form. Usually found in Occurs on the palms,
melanoma (80%), older patients as a soles, and nail beds.
large melanotic
freckle on the temple
or malar region
known as
Hutchinson freckle
Approximately one half Rapidly becoming a It is usually slow Occurs primarily in
arising from a pre-existing palpable, elevated, firm growing but darker-skinned
mole. nodule that may be dense becomes large, often people.
black or reddish blue- reaching 5 to 6 cm in
blackish. diameter.
The lesions usually are slow A distinct convex nodular Initially, it is flat, Metastasizes more
growing and brown, with development indicates but it becomes raised frequently than do
small discrete nodules of deep dermal invasion. and thicker, with other melanomas
differing colors. discrete brown to possibly related to
black nodules and later stage at
irregular edges. presentation.
SSM tends to spread laterally
Nodular melanomas arise
but can be slightly elevated.
from the epidermal- .
dermal junction and
invade deeply into the
dermis and subcutaneous
tissue.
SSM is found most commonly Approximately 5% are
on the back in men and amelanotic.
women and on the lower
extremities of women.

Group – C

3. Write short notes on (any three ) 3x5

a. Bladder changes in B.H.P.


b. Epididymal cyst.
c. E.S.W.L.
d. Nephroblastoma.
e. Haematuria – causes.

Answer.
a. Bladder changes in B.H.P.

I. Bladder outlet obstruction: BOO causes thickening of the wall of the bladder.
Microscopically smooth muscle cells enlarge and there is an increase in connective
tissue (collagen and elastin) between the smooth muscle bundles. In some cases this
may lead to poor compliance, with development of high bladder and intrarenal
pressures.
II. Overactive bladder:
 Leaking urine: during urge to urinate, a small amount of urine may leak. This is
called “urgency incontinence.”
 Frequent urination:
 Waking up at night to urinate.
III. Bladder stones: These are generally caused by an inability to completely empty the
bladder. Bladder stones can cause infection, bladder irritation, blood in the urine
and obstruction of urine flow.
IV. Bladder damage. A bladder that hasn't emptied completely can stretch and weaken
over time. As a result, the muscular wall of the bladder no longer contracts properly,
making it harder to fully empty your bladder.

b. Epididymal cyst:

Introduction: Smooth, extratesticular, spherical cysts in the head of the epididymis are not
uncommon in adult men. They are benign and do not usually require treatment.

Epidemiology: Epididymal cysts usually develop in adults around the age of 40. Epididymal
cysts are rare in children and when they occur, usually present around puberty

Presentation
Patients usually present having noticed a lump. This often causes a great deal of anxiety and
clinical examination can enable reassurance.

 Often epididymal cysts are multiple and may be bilateral.


 Small cysts may remain undetected and asymptomatic. Small cysts are tolerated by patients.
However, once epididymal cysts get large (with size equivalent to the size of a testicle) they
are, unsurprisingly, more likely to present for removal.
 As they are cystic and fluid-filled they are well defined, fluctuant and will transilluminate.
 As they arise in the epididymis, the testis is palpable quite separately from the cyst (unlike a
hydrocele where the testis is palpable within the fluid filled swelling).
 Extratesticular, fluctuant, cystic swellings which transilluminate and are readily palpable
separate from the body of the testis are epididymal cysts and do not usually need further
investigation.
 These can readily be distinguished from testicular tumours which arise from the testis.

Differential diagnosis:

 Spermatocele..
 Inflammatory thickening, either acute or chronic inflammation of the epididymis.
 Hydroceles.
 Varicoceles.
 Other scrotal masses, particularly extratesticular scrotal masses. These are predominantly
benign. Lipomas are the most common extratesticular tumours

Investigations:
o Scrotal ultrasound will assist diagnosis if there is uncertainty.

o Aspiration of fluid is rarely useful or necessary from either a diagnostic or treatment


perspective.

Associated diseases:

 Cystic fibrosis. Congenital absence of the vas deferens is the most common genital tract
anomaly in cystic fibrosis (99% of patients). Epididymal cysts are common
 Von Hippel-Lindau disease. Along with other more significant manifestations of this
syndrome, epididymal cysts are associated with the condition. It is a rare disease and
important to recognise early.
 Maternal exposure to diethylstilbestrol.
 Polycystic kidney disease (autosomal dominant).

Management:

 Treatment is not usually necessary and explanation and advice are all that is usually
required.
 Surgical excision is recommended in children with intractable scrotal pain or if the cyst size
does not seem to involute.
 Likewise excision may be offered to symptomatic adults. Percutaneous aspiration with
instillation of sclerosant is another option but the sclerosant is often poorly tolerated and the
recurrence rate is high.

Complications
o Torsion of the cyst can occur but is extremely rare with only a handful of cases reported.

o They do not become infected.

c. E.S.W.L: Extracorporeal short wave lithotripsy.

Introduction:

Extracorporeal shock wave lithotripsy (ESWL) is a non-invasive treatment of kidney


stones (urinary calculosis) and biliary calculi (stones in the gallbladder or in the liver)
using an acoustic pulse. It is also reported to be used for salivary stones.

Basic principle:

Every lithotriptor contains three essential components - the generator and focusing
system, the coupling system and the imaging system. The generator is the heart of a
lithotriptor: it is the source of shock waves. There are several commonly used
mechanisms for shock wave generation, including electrohydraulic, electromagnetic and
piezoelectric mechanisms. In order to focus the shock wave into a small target zone for
more effective stone fragmentation, a focusing mechanism is needed. Depending on the
type and configuration of shock wave generators, there are several focusing systems -
ellipsoid reflecting surface for electrohydraulic generator, converging acoustic lens for
planar electromagnetic generator etc. Finally, the imaging system, either fluoroscopy or
ultrasound, will help to locate the target stone and position it into the focal zone of the
generator.

Indications:
ESWL is a potential first-line treatment for ureteral and renal stones smaller than 2 cm.
Potential candidates for ESWL include the following:
 Individuals who work in professions in which unexpected symptoms of stone passage may
prompt dangerous situations (eg, pilots, military personnel, physicians) (In such individuals,
definitive management is preferred to prevent adverse outcomes.)
 Individuals with solitary kidneys in whom attempted conservative management and
spontaneous passage of the stone may lead to an anuric state
 Patients with hypertension, diabetes, or other medical conditions that predispose to renal
insufficiency.

Absolute contraindications to extracorporeal shockwave lithotripsy (ESWL) include the


following:
 Acute urinary tract infection or urosepsis
 Uncorrected bleeding disorders or coagulopathies
 Platelet aggregation inhibitors (e.g. ASS)
 Pregnancy
 Uncorrected obstruction distal to the stone
 Cystine stones.
 Aortic aneurysm

Relative contraindications include the following:


 Body habitus: Morbid obesity and orthopedic or spinal deformities .
 Renal ectopy or malformations (eg, horseshoe kidneys and pelvic kidneys)
 Complex intrarenal drainage (eg, infundibular stenosis)

 Poorly controlled hypertension (due to increased bleeding risk)
 Gastrointestinal disorders: In rare cases, these may be exacerbated after ESWL treatment.
 Renal insufficiency: Stone-free rates in patients with renal insufficiency (serum creatinine level
of 2–2.9 mg/dL) were significantly lower than in patients with better renal function (serum
creatinine level < 2 mg/dL).
 Distal obstruction.

Complications:

Immediate complications:
o Stone fragment related complications such as ureteric colic and steinstrasse.
o Other complications include haematuria, haematoma formation and adjacent organ
injury.
Long-term complications of ESWL:
o Increased incidence of new onset hypertension in the elderly.
o New onset diabetes.
d. Nephroblastoma/Wilm’s tumour:
Aetiology:

 Nephroblastoma, also known as Wilms tumor, is the most common solid renal tumor of
childhood.
 The peak age for presentation is during the third year of life, and there is no sex
predilection.
 The disease is seen worldwide with a similar age of onset and sex distribution.
 Tumors are commonly unicentric, but they occur in either kidney with equal frequency.
 In 5% of cases the tumors are bilateral.
 Wilms tumor occurs in familial and nonfamilial forms.

Clinical Presentation:
 Asymptomatic abdominal mass, microscopic hematuria.
 Malaise, weight loss, anemia, fever.
 Hypertension, intraperitoneal rupture of tumour with abdominal crisis, tumour thrombus
involving IVC with dilated abdominal veins, vericocele, cardiac thrombus.
 Malignant tumours frequently metastasize to regional lymph nodes, liver, and lungs.
Association:
 Hypospadias,Undescended testes, anirida, isolated hemihypertrophy,
 Denish-Drash syndrome, Beckwith-Wiedemann syndrome (macroglossia, visceromegaly,
hypoglycemia,umbilical defects), WAGR syndrome( Wilms, aniridia, genitourinary
malformation, mental retardation)
Pathology
 Gross: grayish white, fleshy cut surface, cystic areas, necrosis, hemorrhage.
Microscopically heterogenous tissue aberrant glomeruli, bone, cartilage, muscle, adipose
tissue.
 Three elements: blastemal, stromal, epithelial
 Favourable histology: Triphasic, monophasic
 Unfavourable histology: Anaplastic.

Investigation
 Urinalysis may show evidence of hematuria, and anemia may be present, particularly in
patients with evidence of subcapsular hemorrhage.
 Patients with liver metastases may have abnormal serum chemistries.
 Chest X-ray, abdominal ultrasonography, CT scan, MRI

Treatment: Combination of chemotherapy, surgery, and radiotherapy according to histology and


spread at diagnosis.
e. Haematuria – causes.
See the answer of question no – 2b of group – B of paper II of 2013

Group – D
4. Answer briefly on (any three): 3x5

a. Frozen shoulder - physiotherapy.


b. Dentigerous cyst.
c. Brachytherapy.
d. Epidural Anaesthesia.
e. Small bowel enemas.

Answers.

a. Frozen shoulder - physiotherapy.


Frozen shoulder (adhesive capsulitis) is stiffness, pain, and limited range of movement in the
shoulder.

Physiotherapy:
A physiotherapist can use a number of techniques to keep the movement and flexibility in frozen
shoulder shoulder.
Treatments are:
 Stretching exercises that use specific techniques to move the joint in all directions.

 Massage.
 Thermotherapy, with warm or cold temperature packs.

b. Dentigerous cyst. (See the answer of 3.c of Group-C of paper II of 2014)

c. Brachytherapy.

Introduction:

 Brachytherapy (brak-e-THER-uh-pee) is a procedure that involves placing radioactive


material inside the body.

 Brachytherapy is one type of radiation therapy that's used to treat cancer.

 Brachytherapy is sometimes called internal radiation.


Types:

Different types of brachytherapy can be defined according to (1) the placement of the
radiation sources in the target treatment area, (2) the rate or ‘intensity’ of the irradiation
dose delivered to the tumour, and (3) the duration of dose deliver.

Source placement:

 The two main types of brachytherapy treatment in terms of the placement of the
radioactive source are interstitial and contact.

 In the case of interstitial brachytherapy, the sources are placed directly in the target tissue of
the affected site, such as the prostate or breast.

 Contact brachytherapy involves placement of the radiation source in a space next to the
target tissue. This space may be a body cavity (intracavitary brachytherapy) such as
the cervix, uterus or vagina; a body lumen (intraluminal brachytherapy) such as
the trachea or oesophagus; or externally (surface brachytherapy) such as the skin. A
radiation source can also be placed in blood vessels (intravascular brachytherapy) for the
treatment of coronary in-stent restenosis.

Clinical applications:

 Brachytherapy is commonly used to treat cancers of the cervix, prostate,breast, and skin.
 Brachytherapy can also be used in the treatment of tumours of the brain, eye, head and
neck region (lip, floor of mouth, tongue, nasopharynx andoropharynx), respiratory tract
(trachea and bronchi), digestive tract (oesophagus, gall bladder, bile-ducts,rectum, anus),
urinary tract (bladder,urethra, penis), female reproductive tract (uterus, vagina, vulva),
and soft tissues.

Commonly used radiation sources (radionuclides) for brachytherapy

Radionuclide Type

Cesium-137 (137Cs) β−- particles

Cobalt-60 (60Co) β−- particles


Iridium-192 (192Ir) γ-rays

Iodine-125 (125I) Electron Capture, ε

Palladium-103 (103Pd) Electron Capture, ε

Ruthenium-106 (106Ru) β−- particles

Radium-226 (226Ra) β−- particles

Side effects: Side effects of brachytherapy are specific to the area being treated. Because
brachytherapy focuses radiation in a small treatment area, only that area is affected.

d. Epidural anaesthesia.(See the answer of 4.d of Group-D of paper II of 2014)

e. Small bowel enemas.

Small Bowel Enema or enteroclysis:

Indications:

 Suspected inflammatory bowel disease, including exclusion of small bowel disease


in Crohn’s colitis.
 Intermittent or partial small bowel obstruction.

Preparation - One day before the test - Have a normal breakfast, clear fluids only for lunch
(no solid foods or dairy products). After lunch take 2 Dulcolax tablets. Drink 1 glass of clear
liquid every hour until bed. At 6 p.m. take 1 bottle of Citromag followed by a glass of water.
On the day of the test, have nothing to eat or drink.

Time to perform - 60 to 90 minutes.

Procedure: This procedure involves inserting a thin tube through the mouth, esophagus and
past the stomach to inject barium,methylcellulose and water into the small bowel. This
allows for better visualization of the small bowel than can be seen during a small bowel
follow-through. The back of the throat is 'frozen' to allow easier passage of the tube.

Problem:

 Greater discomfort to the patient, a higher radiation dose and possibility that
gastroduodenal disease may not be seen.
 The investigation requires more experienced radiologists.
 Some radiologists advocate that small bowel enema with careful fluoroscopy and
vigorous manual palpation can produce acceptable results with less cost, radiation
exposure and discomfort to the patient.
The West Bengal University of Health Sciences
rd
M.B.B.S. 3 Professional Part – II Examination (Supplementary), 2011
Subject: Surgery Time: 21/2 hrs.
Paper: II Marks: 60
Group –A
1. a) Describe the management of a lady of 35 years presenting with Toxic multinodular
goiter. 6+9

Answer:
This patient should be investigated as follow:
 T4 and TSH levels to evaluate thyroid function.
 High resolution USG and FNA should be used for suspicious or dominant nodules where
malignancy is suspected. Predominantly solid nodules especially when they are hypoechoic
or display a sonoluscent rim surrounding the lesion(―halo sign‖) should be evaluated by
FNAC.
 X ray Neck AP and Lateral views to assess tracheal position, retro tracheal extension or
incipient compression.
 Chest X ray to evaluate for any features of retrosternal extension.
 CT/MRI are useful in select cases where a retrosternal extension is suspected. It provides
accurate delineation of the depth of goiter extension into the chest and its relation to the
trachea, esophagus and great veins.
 Thyroid scintigraphy is another option to confirm the extent and functional status of the
gland but it is not needed routinely. It does not provide as good anatomical detail as a
CT/MRI. It is especially useful in the hyperthyroid patient with a dominant nodule as it
defines the area of hyperactivity thereby allowing a proper choice of therapeutic modality

The indications for treatment in multinodular goiter are:


Strong
 Compressive symptoms
 Hyperthyroidism
 Suspected malignancy
Relative
 Cosmesis
 Potential for Tracheo-esophageal compression.

The various therapeutic options include


· Suppressive levothyroxine therapy
· Surgical
· Radioactive iodine therapy.

Suppressive Levothyroxine Therapy


 The results of T4 suppressive therapy are inconsistent and marginal. This therapy has a
limited role in management of patients with MNG.
 The aim of such therapy is to consistently suppress TSH levels to <0.5 mU/l. The problem of
this therapy is that a significant number of patients become hyperthyroid with time.
 The overall poor response to T4 supression therapy is because large amounts of thyroid tissue
is likely to be hormone insensitive. Another factor for inconsistent response is the variable
TSH dependencyof thyrocytes.
 Only small goiter would respond and that too partially.
 Goitres that respond do so within a period of 6 months

Radioactive Iodine Therapy


It is of limited value and is useful in only two classes of patients.
A) Those with small goiters may benefit.
B) For patients with substantially increased perioperative risk and reasonable thyroid gland
function.

 Several groups have reported successful RAI therapy with I131 with reduction of thyroid
volume by 40% at 1 year and 50-60% at 3-5 years. Whereas, radiation induced auto immune
thyroiditis develops in a few patients, a significant number of patients develop
hypothyroidism.
 RAI therapy is of no value in large multinodular goiters with poorly functioning nodules as
are commonly seen in India and other iodine deficient areas as the efficacy of RAI therapy
depends on the presence of reasonable gland activity all over the thyroid.

Surgery
 Surgical treatment is the modality of choice in the management of patients with
multinodular goitre.
 The results of surgery are immediate and tissue is available for histological confirmation
of the diagnosis and evaluation for any malignant change. Surgery is the only treatment
option in those with compressive symptoms or those with suspected malignancy.

 Surgical treatment options are between Subtotal and Total thyroidectomy.

Preoperative preparation:

Antithyroid medications are generally administered in preparation for radioactive iodine ablation
or surgery. The medications commonly used are propylthiouracil (PTU, 100 to 300 mg three
times daily) and methimazole (10 to 30 mg three times daily).

Both drugs reduce thyroid hormone production by inhibiting the organic binding of iodine and
the coupling of iodotyrosines (mediated by thyroid peroxidase). In addition, PTU also inhibits
the peripheral conversion of T4 to T3, making it useful for the treatment of thyroid storm. (Both
drugs can cross the placenta, inhibiting fetal thyroid function, and are excreted in breast milk)
Side effects of treatment include reversible granulocytopenia, skin rashes, fever, peripheral
neuritis, polyarteritis, vasculitis, and, rarely, agranulocytosis and aplastic anemia.

Surgery should be postponed until the granulocyte count reaches 1000 cells/m3.

The catecholamine response of thyrotoxicosis can be alleviated by administering beta-blocking


agents. These drugs have the added effect of decreasing the peripheral conversion of T 4 to T3.
Propranolol is the most commonly prescribed medication in doses of about 20 to 40 mg four
times daily. Higher doses are sometimes required because of increased clearance of the
medication.

Or
b) A 70 year old man presents with Acute retention of urine. How would you investigate the
patient? Outline the management in brief the benign hyperplasia of prostate. 7+8
Answer. See the answer of question 1.b of supplementary paper –II of 2012
(Also see the answer of question no.1.b of Group-A paper II of 2012)

Management of brnign hyperplasia of prostate:


Investigations:
Clinical examination:
 DRE: Digital rectal examination
Laboratory findings:
 A urine anlysis to exclude infection or hematuria and serum creatinine measurement to
assess renal function are required.
 Renal insufficiency may be observed in 10% of patients with prostatism and warrants
upper-tract imaging.
 Patients with renal insufficiency are at an increased risk of developing postoperative
complications following surgical intervention for BPH.
 Serum PSA.

Imaging:
 Upper-tract imaging (intravenous pyelogram or renal ultrasound) is recommended only in
the presence of concomitant urinary tract disease or complications from BPH (eg,
hematuria, urinary tract infection, renal insufficiency, history of stone disease).

Treatment:
o Watchful waiting.
o Medical therapy:
Alpha-blockers 5-alpha-reductase inhibitors

Nonselective Finasteride 5 mg daily


 Phenoxybenzamine 10 mg twice a day Dutasteride 0.5 mg daily
Alpha-1, short-acting Subcutaneous implant Yearly
 Prazosin 2 mg twice a day Triptorelin pamoate 3.75 mg every month
Alpha-1, long-acting
 Terazosin 5 or 10 mg daily
 Doxazosin 4 or 8 mg daily
Alpha-1a selective
 Tamsulosin 0.4 or 0.8 mg daily
 Alfuzosin 10 mg daily
o Surgical indications include
 Refractory urinary retention (failing at least one attempt at catheter removal),
 Recurrent urinary tract infection from BPH,
 Recurrent gross hematuria from BPH,
 Bladder stones from BPH, renal insufficiency from BPH, or
 Large bladder diverticula.

o Conventional Surgical treatment:


 Transurethral resection of the prostate: best option.
 Risks of TURP: include retrograde ejaculation (75%), impotence (5–10%), and
incontinence (<1%). Complications include bleeding, urethral stricture or bladder neck
contracture, perforation of the prostate capsule with extravasation, and if severe, TUR
syndrome resulting from a hypervolemic,hyponatremic state due to absorption of the
hypotonic irrigating solution.
 Clinical manifestations of the TUR syndrome include nausea, vomiting, confusion,
hypertension, bradycardia, and visual disturbances. The risk of the TUR syndrome
increases with resectiontimes >90 minutes.
 Treatment includes diuresis and, in severe cases, hypertonic saline administration.

 Transurethral incision of the prostate—Men with moderate to severe symptoms and a


small prostate often have posterior commissure hyperplasia (elevated bladder neck). These
patients will often benefit from an incision of the prostate. This procedure is more rapid
and less morbid than TURP.
 Open simple prostatectomy—When the prostate is too large to be removed
endoscopically, an open enucleation is necessary.Glands >100 g are usually considered for
open enucleation.
o A simple suprapubic prostatectomy is performed transvesically and is the operation of
choice in dealing with concomitant bladder pathology.
o In a simple retropubic prostatectomy, the bladder is not entered.

Minimally Invasive Therapy


 Laser therapy—Many different techniques of laser surgery for the prostate have been
described.
Two main energy sources of lasers have been utilized—Nd:YAG and holmium:YAG
.
Techniques:

 Visual contact ablative techniques are more time-consuming procedures.


 Interstitial laser therapy places fibers directly into the prostate, usually under
cystoscopic control.

 Transurethral electrovaporization of the prostate.


 Hyperthermia—Microwave hyperthermia is most commonly delivered with a
transurethral catheter.
 Transurethral needle ablation of the prostate.
 High-intensity focused ultrasound.
 Intraurethral stents.

Group – B
2. Write short notes on (any three): 3x5

a. Stove in chest.
b. Paraphimosis.
c. Bed sore.
d. Ulnar nerve injury.
e. Hydrocephalus.

Answers.

a. Stove in chest:
The stove-in chest is a rare form of flail chest in which there is collapse of a segment of
the chest wall, associated with a high immediate mortality.

Causes:
 High velocity road traffic accident.
 Direct blow to chest.

Clinical features:

 Pain.
 Paradoxical respiration.
 Respiratory distress.

Investigations:

 Chest X-ray.
 A.B.G.
 CECT – Thorax.

Treatment:
 Conservative treatment with effective analgesia, aggressive chest physiotherapy and
close observation of respiratory function.
 Pneumatic stabilisation with continuous positive airways pressure ventilation (CPAP)
and/or intermittent mandatory ventilation (IMV).
 Operative fixation of the chest wall injury.
b. Paraphimosis. (See the answer of question no.3.a of Group-C of paper II of 2012)

c. Bed sore. (See the answer of question no.4.b of Group-D of supplementary paper II of
2013)

d. Ulnar nerve injury:

Causes:
There are many causes of ulnar nerve injuries, including pressure, trauma and illness. In some
cases, ulnar nerve injuries may arise without a known cause.

 The most common cause of ulnar nerve injury is extended pressure on the ulnar nerve,
known as ulnar nerve entrapment. As the ulnar nerve travels from the shoulder to the hand, it
passes through two tunnels of tissue, the cubital tunnel behind the elbow and Guyon’s canal
in the wrist. Both tunnels are common locations at which the ulnar nerve can be compressed
and injured. The ulnar nerve may also be compressed at the neck or beneath the collarbone.

 Entrapment of the ulnar nerve may result from swelling of soft tissue, cysts, or damage to the
bones in the arms. Bone damage causing ulnar nerve injuries include arthritis, elbow
dislocations, elbow and wrist fractures, and bone spurs. Repetitive motions of the arm and
hand, extensive bending of the elbow, and long-term pressure on the palm of the hand may
also cause ulnar nerve injuries.

 Ulnar nerve injuries may also be the result of direct trauma to the nerve. Finally, any whole
body illness that is known to cause nerve damage, such as diabetes or hypothyroidism, can
affect the ulnar nerve.

Risk factors:

 Activities in which your elbow or wrist is bent or twisted for prolonged periods
 Alcohol abuse
 Brachial plexus injury (injury to the bundle of nerves that transmit signals from the spine
to the shoulder, arm and hand)
 Diabetes (chronic disease that affects your body’s ability to use sugar for energy)
 Elbow and wrist abnormalities
 Hypothyroidism (underactive thyroid)
 Nerve entrapment or compression, such as of the ulnar nerve in the arm
 Rheumatoid arthritis (chronic autoimmune disease characterized by joint inflammation)
 Sleeping positions that put pressure on the ulnar nerve.

Symptoms:
 Abnormal sensations in the little finger and part of the ring finger, usually on the palm
side.

 Loss of coordination of the fingers

 Numbness, decreased sensation

 Pain

 Tingling, burning sensation

 Weakness and clumsiness of the hand

Clinical examinations and Tests:

An exam of the hand and wrist may show:


 "Claw-like" deformity (in severe cases)

 Difficulty moving the fingers

 Wasting of the hand muscles (in severe cases)

 Weakness of hand flexing

Tests may be needed, depending on the history, symptoms, and findings from the physical exam.
These tests may include:

 Blood tests

 Imaging scans

 MRI of the neck

 Nerve ultrasound

 Nerve conduction tests

 Recording of the electrical activity in muscles (EMG)

 X-rays

Treatment:
The goal of treatment is to allow you to use the hand and arm as much as possible. The cause
should be identified and treated. Sometimes, no treatment is needed and you will get better on
your own.
Medications may include:

 Over-the-counter pain relievers or prescription pain medications to control pain


(neuralgia)

 Other medications, including gabapentin, phenytoin, carbamazepine, or tricyclic


antidepressants such as amitriptyline or duloxetine, to reduce stabbing pains

 Corticosteroids injected into the area to reduce swelling and pressure on the nerve.

A supportive splint at either the wrist or elbow can help prevent further injury and relieve the
symptoms. You may need to wear it all day and night, or only at night. If the ulnar nerve is
injured at the elbow, wearing a pad may help protect the nerve from further injury. Be careful to
avoid leaning on the elbow.

Surgery to relieve pressure on the nerve may help if the symptoms get worse, or if there is proof
that part of the nerve is wasting away.

Other treatments may include:

 Physical therapy exercises to help maintain muscle strength

 Occupational counseling or occupational therapy for changes you can make at work, or
retraining

e. Hydrocephalus.

Hydrocephalus, also known as "water on the brain", is a medical condition in which there is
an abnormal accumulation of cerebrospinal fluid (CSF) in the ventricles, or cavities, of
the brain. This may cause increased intracranial pressure inside the skull and progressive
enlargement of the head, convulsion, and mental disability. Hydrocephalus can also cause
death.
Signs and Symptoms:
Symptoms of increased intracranial pressure:
 Headaches, vomiting, nausea, papilledema, sleepiness, or coma.
 Elevated intracranial pressure may result in uncal and/or cerebellar tonsill herniation,
with resulting life threatening brain stem compression.
 The triad (Hakim triad) of gait instability, urinary incontinence and dementia is a
relatively typical manifestation of the distinct entity normal pressure
hydrocephalus (NPH). Focal neurological deficits may also occur, such as abducens
nerve palsy and vertical gaze palsy (Parinaud syndrome due to compression of
the quadrigeminal plate, where the neural centers coordinating the conjugated vertical eye
movement are located).
Normal pressure hydrocephalus:
The symptoms depend on the cause of the blockage, the person's age, and how much brain tissue
has been damaged by the swelling.
In infants with hydrocephalus, CSF fluid builds up in the central nervous system, causing the
fontanelle (soft spot) to bulge and the head to be larger than expected.

Early symptoms Symptoms that may occur in older children can include:
may also include:

 Eyes that appear to gaze  Brief, shrill, high-pitched cry  Loss of bladder control
downward (urinary incontinence)
 Changes in personality,
 Irritability memory, or the ability to  Loss of coordination and
reason or think trouble walking
 Seizures
 Changes in facial appearance  Muscle spasticity (spasm)
 Separated sutures and eye spacing
 Slow growth (child 0-5 years)
 Sleepiness  Crossed eyes or uncontrolled
eye movements  Slow or restricted movement
 Vomiting
 Difficulty feeding  Vomiting.

 Excessive sleepiness

 Headache

 Irritability, poor temper


control

Pathology:
Hydrocephalus is usually due to blockage of cerebrospinal fluid (CSF) outflow in
the ventricles or in the subarachnoid space over the brain. In a person without hydrocephalus,
CSF continuously circulates through the brain, its ventricles and the spinal cord and is
continuously drained away into the circulatory system. Alternatively, the condition may result
from an overproduction of the CSF fluid, from a congenital malformation blocking normal
drainage of the fluid, or from complications of head injuries or infections.
Clsssification:
Hydrocephalus can be caused by impaired cerebrospinal fluid (CSF) flow, reabsorption, or
excessive CSF production.

The most common cause of hydrocephalus is CSF flow obstruction, hindering the free passage of
cerebrospinal fluid through the ventricular system and subarachnoid space (e.g.,stenosis of
the cerebral aqueduct or obstruction of the interventricular foramina -foramina of
Monro secondary to tumors, hemorrhages, infections or congenital malformations).

Hydrocephalus can also be caused by overproduction of cerebrospinal fluid (relative obstruction)


(e.g., papilloma of choroid plexus).

Based on its underlying mechanisms, hydrocephalus can be classified


into communicating and non-communicating (obstructive). Both forms can be
either congenital or acquired.
Communicating:
Communicating hydrocephalus, also known as non-obstructive hydrocephalus, is caused by
impaired cerebrospinal fluid resorption in the absence of any CSF-flow obstruction between the
ventricles and subarachnoid space.
Various neurologic conditions may result in communicating hydrocephalus, including
subarachnoid/intraventricular hemorrhage, meningitis, Chiari malformation, and congenital
absence of arachnoidal granulations (Pacchioni's granulations).
Scarring and fibrosis of thesubarachnoid space following infectious, inflammatory, or
hemorrhagic events can also prevent resorption of CSF, causing diffuse ventricular dilatation.

 Normal pressure hydrocephalus (NPH) is a particular form of communicating


hydrocephalus, characterized by enlarged cerebral ventricles, with only intermittently
elevated cerebrospinal fluid pressure.

 Hydrocephalus ex vacuo also refers to an enlargement of cerebral ventricles and


subarachnoid spaces, and is usually due to brain atrophy (as it occurs in dementias), post-
traumatic brain injuries and even in some psychiatric disorders, such as schizophrenia. As
opposed to hydrocephalus, this is a compensatory enlargement of the CSF-spaces in
response to brain parenchyma loss - it is not the result of increased CSF pressure.

Non-communicating:
Non-communicating hydrocephalus, or obstructive hydrocephalus, is caused by a CSF-flow
obstruction ultimately preventing CSF from flowing into the subarachnoid space (either due to
external compression or intraventricular mass lesions).

 Foramen of Monro obstruction may lead to dilation of one or, if large enough (e.g.,
in Colloid cyst), both lateral ventricles.

 The aqueduct of Sylvius, normally narrow to begin with, may be obstructed by a


number of genetically or acquired lesions (e.g., atresia, ependymitis, hemorrhage, tumor) and
lead to dilation of both lateral ventricles as well as the third ventricle.

 Fourth ventricle obstruction will lead to dilatation of the aqueduct as well as the lateral
and third ventricles.

 The foramina of Luschka and foramen of Magendie may be obstructed due to


congenital failure of opening (e.g., Dandy-Walker malformation).

Congenital
The cranial bones fuse by the end of the third year of life. For head enlargement to occur,
hydrocephalus must occur before then. The causes are usually genetic but can also be acquired
and usually occur within the first few months of life, which include
1) intraventricular matrix hemorrhages in premature infants,
2) infections,
3) type II Arnold-Chiari malformation,
4) aqueduct atresia and stenosis, and
5) Dandy-Walker malformation.
In newborns and toddlers with hydrocephalus, the head circumference is enlarged rapidly and
soon surpasses the 97th percentile. Since the skull bones have not yet firmly joined together,
bulging, firm anterior and posterior fontanelles may be present even when the patient is in an
upright position.
The infant exhibits fretfulness, poor feeding, and frequent vomiting.
As the hydrocephalus progresses, torpor sets in, and the infant shows lack of interest in his
surroundings. Later on, the upper eyelids become retracted and the eyes are turned downwards
(due to hydrocephalic pressure on the mesencephalic tegmentum and paralysis of upward gaze).
Movements become weak and the arms may become tremulous.
Papilledema is absent but there may be reduction of vision. The head becomes so enlarged that
the child may eventually be bedridden.
About 80-90% of fetuses or newborn infants with spina bifida—often associated
with meningocele or myelomeningocele—develop hydrocephalus.

Acquired: This condition is acquired as a consequence of CNS infections, meningitis, brain


tumors, head trauma, intracranial hemorrhage (subarachnoid or intraparenchymal) and is usually
extremely painful.

Treatment:
Hydrocephalus treatment is surgical. It involves the placement of a ventricular catheter (a tube
made of silastic), into the cerebral ventricles to bypass the flow obstruction/malfunctioning
arachnoidal granulations and drain the excess fluid into other body cavities, from where it can be
resorbed.
Most shunts drain the fluid into the peritoneal cavity (ventriculo-peritoneal shunt), but alternative
sites include the right atrium (ventriculo-atrial shunt), pleural cavity (ventriculo-pleural shunt),
and gallbladder
An alternative treatment for obstructive hydrocephalus in selected patients is theendoscopic third
ventriculostomy (ETV), whereby a surgically created opening in the floor of the third ventricle
allows the CSF to flow directly to the basal cisterns, thereby shortcutting any obstruction, as
in aqueductal stenosis.

Group – C

3. Write short notes on (any three ) 3x5

a. Blood discharge per nipple.


b. Hydronephrosis.
c. Parotid fistula.
d. Hypospadias.
e. Oschner Sherren regime.

Answers:
a. Blood discharge per nipple.

Causes:
 Benign:
o Mastitis.
o Intraductal papilloma.
o Duct ectasia.
 Malignant:
o Intraductal carcinoma.
o Paget’s disease of nipple.
Discharge from a single duct Discharge from more than one duct
 Blood-stained  Blood-stained
o Intraduct papilloma o Carcinoma
o Intraduct carcinoma o Ectasia
o Duct ectasia o Fibrocystic disease

Breast symptoms that may occur along with bleeding nipple

Bleeding nipple may accompany other symptoms affecting the breast. These are related to
benign or malignant (cancerous) processes and include:

 Breast discomfort or pain


 Breast lump
 Breast swelling, redness or warmth
 Change in the size, shape or appearance of the breast
 Nipple retraction or turning inward
 Other nipple discharge such as pus
Other symptoms that may occur along with bleeding nipple

Bleeding nipple may accompany symptoms related to other body systems including:

 Bone pain
 Enlarged lymph nodes in the armpit
 Skin rash or ulcerations
 Sudden weight loss
 Swelling of the arm next to the affected nipple or breast

Symptoms that might indicate a serious condition:

 Bone pain
 High fever (higher than 101 degrees Fahrenheit)
 Nipple retraction or turning inward
 Other nipple discharge such as pus
 Skin ulceration
 Sudden weight loss
 Swelling of the arm next to the affected nipple or breast

Diagnosis by:
 Cytology
 Mammograghy
 USG
 Ductogram
 Biopsy.

Treatment:
 Microdochectomy.
 Cone excision of the major ducts (after Hadfield)-(subareolar resection)
 Simple and modified radical mastectomy.

b. Hydronephrosis. (See the answer of question no.1 of Group-A paper II of 2013)

c. Parotid fistula: A parotid fistula is a communication between the skin and a salivary duct or
gland, through which saliva is discharged.
Causes:
I. Parotid gland and duct injuries following surgery of parotid gland and temporomandibular
joint.
II. Various other causes of parotid injury are rupture of parotid abscess, inadvertent incision
of parotid abscess, complication of superficial Parotidectomy, gunshot wounds and
trauma.
Clinical features:
 Salivary extravasations into the tissues causing swelling over or adjacent to parotid gland.
 Expanding neck mass and cutaneous fistula formation.
 In glandular fistulas discharge is less and tends to heal spontaneously with conservative
treatment, where as ductal fistulas continuously discharge saliva and spontaneous healing is
very rare.
Diagnosis:
 Diagnosis is largely made from clinical signs and history.
 The fistula is often visible over the parotid region, or located upon clinical examination.
Discharge from the fistula, which must be differentiated from a draining sinus, often
results in a visibly wet face.
 The salivary discharge from a fistula is usually golden or blood tinged saliva, which will
be viscous and 'stringy' in appearance.
 The presence of mucous can be confirmed by using periodic acid-schiff stain if necessary.
 History of or current presence of a wound over the parotid regionis a very good indicator.

Treatment options:
 Conservative management.
 Aspiration and pressure dressings
 Anti-Sialogogues
 Radiation therapy
 Parasympathetic Denervation (Tympanic Denervation)
 Cauterization of the Fistulous
 Reconstruction of the duct
 Superficial or Total Parotidectomy.
d. Hypospadias. (See the answer of question no. 4.c of Group-D of supplementary paper II of
2013)

e. Oschner Sherren regime.


If an appendicular mass is present and the condition of the patient is satisfactory, the standard
treatment is the conservative Ochsner–Sherren regimen.
This strategy is based on the premise that the inflammatory process is already localised and
that inadvertent surgery is difficult and may be dangerous.
It may be impossible to find the appendix and, occasionally, a faecal fistula may form. Under
these circumstances, it is wise to observe a nonoperative management but to be prepared to
operate should clinical deterioration occur.
Components:
 Keep the patient nil per mouth.
 Careful recording of the patient’s condition and the extent of the mass should be made
and the abdomen regularly reexamined. It is helpful to mark the limits of the mass on
the abdominal wall using a skin pencil.
 A contrast-enhanced CT examination of the abdomen should be performed and
antibiotic therapy instigated.
 An abscess, if present, should be drained radiologically.
 Temperature and pulse rate should be recorded 4-hourly and a fluid balance record
maintained.
 Clinical deterioration or evidence of peritonitis is an indication for early laparotomy.
Clinical improvement is usually evident within 24–48 hours.
Failure of the mass to resolve should raise suspicion of a carcinoma or Crohn’s disease. Using
this regimen, approximately 90 per cent of cases resolve without incident.
The great majority of patients will not develop recurrence, and it is no longer considered
necessary to remove the appendix after an interval of 6–8 weeks.
Patients over the age of 40 should have colonoscopy and follow-up imaging to ensure resolution
as a small minority (less than 5 per cent) may have an underlying appendicular or colonic
malignancy.
Criteria for stopping conservative treatment of
an appendix mass
_ A rising pulse rate
_ Increasing or spreading abdominal pain
_ Increasing size of the mass

Group – D
4. Answer briefly on (any three): 3x5

a. Cervical traction.
b. Adamantinoma.
c. U.S.G for Hepato-biliary diseases.
d. Local anaesthesia.
e. Brachytherapy.
Answer:
a. Cervical traction.
Cervical traction is used to alleviate neck pain, as well as shoulder and upper arm pain
associated with cervical spine disorders. It can also relieve muscle spasms and nerve
compression, and can aid in achieving proper alignment of the cervical vertebrae.
There are two commonly used methods of applying cervical traction.
1. Supine cervical traction (in bed).
2. Overdoor cervical traction (vertical-sitting up)

Indications:
 Herniated or compressed intervertebral disc in the cervical spine.

 Radiculopathy.
 Osteoarthritis.
 Anytime it is favorable to stretch and mobilize the soft tissue.

Contraindications:
 Acute cervical injury.
 Osteomyelitis.
 Spinal instability.
 Spinal hypermobility.

 Tumours in the neck area.


 Rheumatoid arthritis.

b. Adamantinoma.

Introduction:
Adamantinoma is a primary low-grade, rare malignant bone tumor that is predominantly located
in the mid-portion of the tibia.

 Epidemiology

o incidence

 less than 300 cases have been documented

o demographics
 occurs in young adults (20 - 40 years of age)

 Associated conditions

o osteofibrous dysplasia

 historically, it was thought that osteofibrous dysplasia


(OFD) was a precursor to this adamantinoma, however current studies
have cast doubt on this theory

 Prognosis

o may metastasize to lungs (25%), therefore long-term followup is recommended

o recurrence is uncommon with negative margin excision

Presentation

 Symptoms

o pain of months to years duration

 Physical exam

o bowing deformity or a palpable mass of tibia is common

Imaging

Radiographs

o multiple sharply circumscribed lucent lesions ("soap bubble" appearance) with interspersed sclerotic bon
in mid tibia

 some lesions may destroy cortex

o may see bowing of the tibia

o radiographic evolution of lesions is helpful in the diagnosis as lesions may continue to grow and erode th
the cortex

o unlike other primary bone tumors, adamantinoma typically shows no periosteal reaction
Studies

 Histology

o characteristic fibrous and epithelial tissue in gland like pattern

o contains both epithelial and mesenchymal cells

 nests of epithelial-like cells arranged in palisading or glandular pattern

 background of fibrous stroma

Treatment

 Operative

o wide-margin surgical resection

 indications

 standard of care in most patients

 techniques

 often requires intercallary resection with allograft or intercallary megaprosthesis reconstruction

 as adamantinoma is a low-grade malignancy, radiotherapy and/or chemotherapy is not typical


used for local control of disease

Differentials & Treatment Groups

 Osteofibrous dysplasia

o differentiating between osteofibrous dysplasia and adamantinoma is critical

 osteofibrous dysplasia is benign and treated with observation

 adamantinoma is malignant and treat with surgical resection

c. U.S.G for Hepato-biliary diseases.


Diagnostic:

Liver:

USG is helpful in commenting about the following


1. Size:
 Lt. Lobe span - (5-10 cm).

 Rt. Lobe span - (8-15 cm).

2. Focal lesion:
 Single or Multiple

 Size

 Site (segmental anatomy)

 Echopattern

1. Echofree e.g. hepatic simple cyst, hydatid cyst.


2. Hypoechoic e.g. amoebic liver abscess, lymphoma.
3. Hyperechoic (echogenic) e.g. haemangioma .
4. Hetergenous e.g. cancer, secondary metastasis.
 Differential diagnosis

3. Diffuse liver disease .


 Schistosomal hepatic fibrosis: (Thickened portal tracts):

Portal tracts appear in US as portal vein radicles only. If the wall of these radicles
are thickened, we measure the portal tracts (outer-outer diameter). If the diameter
is more than 3 mm in more than 3 tracts “Periportal Thickening”.

 Liver cirrhosis: coarse echopattern with: (Miliary =echogenic fine liver dots).

o Irregular surface.

o Large caudate lobe

o Attenuated hepatic veins.

 Bright liver: Increase brightness “less dark”.

Normally, the echopattern of the liver is slightly brighter than the renal
parenchyma.
 D.D of Bright liver .

o Fatty liver (D.M.–Hyperlipidemia-obese patients)

o Chronic hepatitis

o Liver cirrhosis

4. Hepatic vasculature . ( portal vein & hepatic veins )


A- Portal Vein:
• The diameter is normally up to 12mm, in fasting adults.
• From 13-17mm in suspected cases of portal hypertension.
• >17 it is sure portal hypertension.
• In some cases of portal hypertension the P.V diameter is within normal due to the
presence of collaterals.
Portal Vein Thrombosis Occurs in association with:
• H.C.C.
• After sclerotherapy.
• After splenectomy
Collaterals:
The presence of any collaterals is a sure sign of Portal Hypertension
1. Para umbilical vein : seen in the falciform ligament.
2. Coronary vein : seen in the inferior surface of the left lobe.
Normally less than 5 mm.
It is related to oesophageal varices.
3. Splenic hilum collaterals: around splenic vein
Directed to the kidney - lienorenal collaterals (benign)
Directed to stomach - lienogastric: it is related to
fundal varices.
B- Hepatic Veins:
Importance of hepatic veins:
• Attenuated in Liver cirrhosis and veno-occlusive disease.
• Dilated in congested hepatomegaly.
• In segmented Anatomy.
5. Intrahepatic biliary radicles:

• Normally they are not seen, when dilated as in Obstructed Jaundice →“double barrel
sign” (portal vein tributary and intrahepatic bile radicle).
• When the obstruction is intrahepatic (e.g hilar cholangiocarcinoma) there is no dilatation
of C.B.D but when the obstruction is extra hepatic there is dilatation of C.B.D. more than
8 mm
CBD:
Detect CBD obstruction:
• Stones in the CBD, hepatic duct, or ampulla of vater
• Cancer head of pancreas, ampulla of vater, cholangiocarcinoma.
• Lesions in the porta hepatis as porta hepatis lymph node enlargement.
• Fasciola or ascaris.
Gall bladder:
Comment on the following:
1. Size: Long axis 6-12 cm , short axis 3-5 cm
• Contracted < 5 cm
• Distended > 12 cm when the patient is fasting
2. Wall thickness:
• Measured in the side in contact with the liver.
• Normally it is up to 3 mm.
• From 3-5 mm >>> suspect thick wall
• More than 5 mm >>> It is a thick wall gall bladder which is seen in:
o Cholecystitis (acute-chronic).

o Ascites.

o Hepatitis (viral).
o Schistosomiasis.

3. Contents:
 Stone.

 Parasites.

 Mud.

 Masses - polyp, cancer

Therapeutic:
1. Percutaneous drainage of liver abscess.
2. PAIR therapy in hydatid cyst of liver.

d. Local anaesthesia.

Introduction: Local anesthesia is any technique to induce the absence of sensation in part of the
body.
Local anesthetics vary in their pharmacological properties and they are used in various
techniques of local anesthesia such as:

 Topical anesthesia (surface)

 Infiltration

 Plexus block

 Epidural (extradural) block

 Spinal anesthesia (subarachnoid block)


Adverse effects depend on the local anesthetic agent, method, and site of administration and is
discussed in depth in thelocal anesthetic sub-article, but overall, adverse effects can be:

1. localized prolonged anesthesia or paresthesia due to infection, hematoma, excessive fluid


pressure in a confined cavity, and severing of nerves & support tissue during injection.

2. systemic reactions such as depressed CNS syndrome, allergic reaction, vasovagal


episode, and cyanosis due tolocal anesthetic toxicity.

3. lack of anesthetic effect due to infectious pus such as an abscess.


Indications: Anesthesia is indicated to reduce pain before surgical procedures.

Contraindications: Patient allergies may preclude the use of a particular anesthetic agent (see
Complications section below).

e. Brachytherapy. (See the answer of 4.c of Group-D of supplementary paper II of 2012)


The West Bengal University of Health Sciences
M.B.B.S. 3rd Professional Part – II Examination (Supplementary), 2010
Subject: Surgery Time: 21/2 hrs.
Paper: II Marks: 60
Group –A
1. a) Classify thyroid malignancies. How will you manage a case of Follicular carcinoma
thyroid? 5+10
Answer. See the question. 1. a of Group – A of paper – II of 2012
Follicular cell malignancies can be clinicopathologically divided into
Differentiated (DTC)
Papillary & variants
Follicular & variants
Hürthle cell
Undifferentiated
Anaplastic
The presentation may be:

with local pressure / infiltration effects viz hoarseness of voice, dysphagia, and superior
mediastinal syndrome.

or occult.
thyroidism. Development of malignancy in a nodule of toxic
goitre is well documented.
-existing goitre indicated by rapid increase in size or local pressure
effects.
: Solitary thyroid nodule has 30% chances of being malignant. Radionuclide scan is
helpful in planning management. Cold nodule must vigorously be pursued to rule out
malignancy.

The work up includes:


- short history (not necessary in WDTC) and recent rapid increase in size.
mptoms of local involvement viz change n voice, respiratory difficulty, dysphagia.
- hard consistency, local limitation of mobility, involvement of strap
muscles, and obliteration of carotid pulse all suggests malignancy. Pizello‘s method, Lahey‘s
method and Berry‘s sign are all contributory.

5% cases.
Investigations:
High Resolution Ultrasound: Halo sign, commet tail sign, and type 3 intranodular vascularity of
malignancy are helpful. Involvement of trachea, vessels, strap muscles can be made out.
Fine Needle Aspiration Cytology: Cellular diagnosis of DTC is now feasible; ultrastructural study and
histo/histocytochemistry are a great help.
Trucut Biopsy:Selected indication is in inoperable cases to plan out management. Also to differentiate
lymphoma from anaplastic carcinoma.
Monoclonal antibodies for lymphoma: This can differentiate anaplastic carcinoma from lymphoma.
Imprint smear: Better interpretation than FNAC/ Frozen section.
CT/MRI: Specific indication is to know extent of tumour spread and local tissue plane infiltration.
Scintiscan: Presently limited use. Cold nodule classically in malignancy. Still helpful in solitary nodule
thyroid.
Tumor markers: Very significant is TGB which if done before and after management helps in follow up
and can indicate recurrence/ mets. CEA, CD-34, MVD, p 53 protein have also correlation.
X-ray Chest and X-ray of bone met site: Demonstrates local secondary deposit.
Ultrasound Liver: Done to detect liver metastases wherever suspected.
Skeletal survey: Done by radionuclide scan (Gamma camera) wherever indicated can alter the
clinical staging of the disease.

Flow chart:

Surgery for Follicular Carcinoma:


 From clinicopathological stand point these carcinomas are divided into low risk and high risk.
 Three important risk factors viz vascular invasion, metastases and age greater than 45 years.
 The result of cytological and frozen section histology in this tumour is high in accuracy of both in
interpretation.
 In low risk patient with intrathyroidal (non invasive) lesion less than 2.5 cm in size (determined on
scan) a hemithyroidectomy with isthumusectomy suffices.
 Completion thyroidectomy is indicated if histology reveals a more invasive form of carcinoma
postoperatively.
 In all other follicular carcinoma, a total or near total thyroidectomy is indicated.
This is recommended in presence of even metastases (the incidence of which is fairly high with this
carcinoma) as it facilitates adjuvant treatment with I -131.
 If nodes are involved, ipsilateral functional block dissection with central compartment clearance is to
be undertaken.
Hürthle Cell Tumors:
 Hürthle cell, though modified follicular cell is now considered a separate tumor altogether, contrary to
earlier belief, majority HCN are benign.
 Capsular and vascular invasion on histology and metastases clinically characterize malignant HCN.
 There is an indeterminate form also. The potential incidence of indeterminate forms creates
controversy regarding extent of surgery.
 The undisputed benign forms are best treated by hemithyroidectomy.
 For ―aggressive‖ benign type, indeterminate type and frankly malignant HCN, a total or near
thyroidectomy is the standard.
 If malignancy is found on post operative histology in a pre operative diagnosed benign lesion, a
completion thyroidectomy is then indicated.
 The approach to lymph nodes is same as in follicular carcinoma; if nodes are involved, ipsilateral
functional block dissection with central compartment clearance is to be undertaken.

Or
b) A 40 year old patient presents with haematuria. Enumerate the differential diagnosis,
plan the investigations and treatment. 4+7+4
Answer. See the answer of [Question 2.b of Group – B of paper –II of 2013]
Also see the answer of [Question.1of Group – A of supplementary paper –II of 2014]
Group – B
1. Write short notes on (any three): 3x5

a. Mixed salivary tumour.


b. Sentinel lymph node biopsy.
c. Gas gangrene.
d. Primary hyper parathyroidism.
e. Hypersplenism.
a. Mixed salivary tumour. Also known as pleomorphic adenoma.
Aetiology:
 Site : MC: Parotid gland ( 90%)- MC –Tail of gland
Submandibular gland (7%)
Minor salivary glands-MC: Palate
 Occurs more commonly in females (F:M=3:1)
 Age: any age . MC : 40-50yrs
 Usually unilateral
Pathology:

 Benign tumour.
 Tumor capsule-well formed, but incomplete
 Tiny excrescences (pseudopods) project outside.
 Give rise to recurrences.
Microscopic appearance:
 2 groups of cells :
o Well differentiated epithelial cells-acini/cords/sheets
o Spindle/stellate cells
 Abundant intercellular mucoid material-resembles cartilage
 Pleomorphic stroma
 No necrosis
 Rarity of mitotic figures
Clinical features:

 Painless slow growing swelling  Deep lobe swelling passes through Patey’s
stylomandibular tunnel
 In the parotid both lobes involved. If only deep lobe  Raised ear lobule.
involved – DUMB BELL TUMOUR
 Dysphagia if deep lobe is involved  Cannot be moved abv zygomatic bone- CURTAIN
SIGN
 Deviation of uvula & pharyngeal wall towards  Facial nerve not involved.
midline-deep

Investigations:
 FNAC – Important and diagnostic
 Open biopsy – contraindicated
Due to -
o Chance of injury to facial nerve,
o Seedling & high chance of recurrence,
o Parotid fistula formation
 CT SCAN – Best imaging modality.
 MRI
Treatment:
 Tumor is radio resistant
 Surgery :
o Enucleation –avoided. High recurrence.
o Treatment of choice: Superficial parotidectomy –Patey’s operation( if superficial lobe alone is
involved)
o Total conservative parotidectomy (If both lobes involved)
o Facial nerve is preserved.
Complications:
 Recurrence (1 – 5 %)
 Malignancy
o 3-5 % in early tumors
o 10% in long duration( >15 yrs)
b. Sentinel lymph node biopsy.
Introduction:

 A sentinel lymph node is defined as the first lymph node to which cancer cells are most likely to spread
from a primary tumor. Sometimes, there can be more than one sentinel lymph node.

 A sentinel lymph node biopsy (SLNB) is a procedure in which the sentinel lymph node is identified,
removed, and examined to determine whether cancer cells are present.
A negative SLNB result suggests that cancer has not developed the ability to spread to nearby lymph
nodes or other organs. A positive SLNB result indicates that cancer is present in the sentinel lymph node
and may be present in other nearby lymph nodes (called regional lymph nodes) and, possibly, other
organs. This information can help a doctor determine the stage of the cancer (extent of the disease within
the body) and develop an appropriate treatment plan.

Benefits:
In addition to helping in the staging of cancers and estimate the risk that tumor cells have developed the
ability to spread to other parts of the body, SLNB may help some patients avoid more extensive lymph node
surgery. Removing additional nearby lymph nodes to look for cancer cells may not be necessary if the
sentinel node is negative for cancer. The potential adverse effects of lymph node surgery include the
following:

 Lymphedema <chronic lymphedema< lymphangiosarcoma.


 Seroma, or the buildup of lymph fluid at the site of the surgery.
 Numbness, tingling, or pain at the site of the surgery.
 Difficulty moving the affected body part.
Uses:
 SLNB is most commonly used to help stage breast cancer and melanoma.
 However, it is being studied with other cancer types, including colorectal cancer, gastric
cancer, esophageal cancer, head and neck cancer, thyroid cancer, and non-small cell lung cancer

In carcinoma breast:

 Sentinel lymph node biopsy is primarily used in women with early breast cancers (T1 and T2, N0).
 It also is accurate for T3 N0 cancers, but nearly 75% of these women will have nonpalpable axillary
lymph node metastases.
 In women undergoing neoadjuvant chemotherapy to permit conservation surgery, sentinel lymph
node biopsy may be used.

Contraindications to the procedure include

 Palpable lymphadenopathy.
 Prior axillary surgery, chemotherapy or radiation therapy.
 Multifocal breast cancers.

c. Gas gangrene.
Causative organism: This is caused by C. perfringens.
These Gram-positive, anaerobic, spore-bearing bacilli are widely found in nature, particularly
in soil and faeces.

Patients at risk:
 This is found in military and traumatic surgery and colorectal operations.
 Patients who are immunocompromised, diabetic or have malignant disease are at greate risk,
particularly if they have wounds containing necrotic or foreign material, resulting in
anaerobic conditions.

Military wounds provide an ideal environment as the kinetic energy of high-velocity missiles or shrapnel
causes extensive tissue damage. The cavitation which follows passage of a missile through the tissues
causes a ‘sucking’ entry wound, leaving clothing and environmental soiling in the wound in addition to
devascularised tissue.

Clinical features:
 Gas gangrene wound infections are associated with severe local wound pain and crepitus (gas in the
tissues, which may also be noted on plain radiographs).
 The wound produces a thin, brown, sweet-smelling exudate, in which Gram staining will reveal bacteria.
 Oedema and spreading gangrene due to release of collagenase, hyaluronidase, other proteases and alpha
toxin.
 Early systemic complications with circulatory collapse and multi organ systemic failure follow if prompt
action is not taken.

Treatment:
 Antibiotic prophylaxis should always be considered in patients at risk, especially when amputations are
performed for peripheral vascular disease with open necrotic ulceration.
 Once gas gangrene infection is established, large doses of intravenous penicillin and aggressive
debridement of affected tissues are required.

d. Primary hyper parathyroidism.


See question 2.b of Group – B of supplementary paper – II of 2014.
e. Hypersplenism. Hypersplenism is a clinical syndrome characterized by:
 Splenomegaly, although this may be only moderate
 Pancytopenia or a reduction in the number of one or more types of blood cells, neutropenia is less
common than anemia and thrombocytopenia
 Normal production or hyperplasia of the precursor cells in the marrow or a so called maturation arrest
 Decreased red blood cells survival and
 Decreased platelet survival.

Causes of Splenomegaly & hypersplenism:


Infectious causes:

 Viral infection: infectious mononucleosis, viral hepatitis, and HIV infection.

 Bacterial infection: enteric fever, bacterial endocarditis, brucellosis, and Tuberculosis.

 Parasitic infections: malaria, visceral leishmaniasis,and schistosomiasis

 Fungal infections: histoplasmosis

Hyperplastic splenomegaly: Hereditary spherocytosis, symptomatic elliptocytosis, thalasaemia,


polycythaemia Rubra vera, myelofibrosis, and Chronic myeloid leukaemia, Chronic Lymphocytic
leukaemia and Lymphoma.

Congestive splenomegaly: Liver cirrhosis, hepatic schistosomiasis "portal hypertension", hepatic vein
obstruction, portal vein obstruction, splenic vein obstruction, congestive heart failure with increased
venous pressure, and splenic artery aneurysm.

Infiltrative splenomegaly: Gaucher's disease, amyloidosis, Niemann-pick diease, histiocytosis, splenic


tumours and metastatic malignancy, Marble bone disease and Waldenstrom macroglobulinaemia.

Micellanous causes: Connective tissue disorders. Idiopathic non tropical splenomegaly, iron deficiency
anaemia, B12 deficiency, thyrotoxicosis, Breylliosis.
Diagnosis of hypersplenism:

Clinical findings:

The symptoms are of 3 types:


1. Symptoms related to the enlarged spleen such as abdominal fullness associated with feeling of heaviness
and discomfort and pain in the left upper quadrant of the abdomen.
2. Haematological symptoms: Symptoms related to thrombocytopenia are common, such as, bruising and
epistaxis. Symptoms related to anaemia are fatigue, weakness and pallor. Leucopenia leads to recurrent
infections and oral ulcerations.
3. Symptoms and signs of the underlying diseases.

Investigations:
i. Laboratory findings: Anaemia, thrombocytopenia and leucopenia.
ii. Evaluation of splenic size: with physical examination, abdominal Ultrasonography, CT and MRI.
iii. Evaluation of splenic function: reduced red cell or platelet survival can be measured by labelling the
patient's cells with Cr51, or the platelets with indiumand measuring the rate of disappearance of
radioactivity from the blood.

The diagnosis of hypersplenism is ultimately confirmed by response to splenectomy, although an immediate


remission may be followed in the longer term by relapse with return of cytopenia.

Points to remember

• Hypersplenism is a triad of splenomegaly, pancytopenia and normocellular


bone marrow.
• Spelenomgaly due to any cause can lead to hypersplenism, but Portal
hypertension and hematological disorders are the commonest causes of
hypersplenism.
• No direct relation between splenic size and hypersplenism, however
hypersplenism is more common among those who have gross
splenomegaly.
• Differential diagnosis of gross splenomegaly includes Portal hypertension,
visceral leishmaniasis, Tropical splenomegaly syndrome, Gaucher disease
in addition to myeolproliferative and lymphoproliferative disorders.
• Treatment of the underlying disease is best option for control of
hypersplenism, however splenectomy and splenic embolization can be
indicated in some patients.
Group – C

2. Write short notes on (any three ) 3x5

a. Meconium ileus.
b. Hydrocephalus.
c. Intravenous anaesthetics.
d. Post burn contracture.
e. FAST.
Answer.
a. Meconium ileus.
Introduction: Meconium is the material found in the intestine in a newborn. It consists of succus entericus
that is made up of bile salts, bile acids, and debris that is shed from the intestinal mucosa during intrauterine
life. It is normally evacuated within 6 hours after birth or sooner in utero as a result of a vagal response to
perinatal stress.
Normally, meconium is invisible on a radiograph. It occasionally has a mottled appearance on abdominal
radiographs during the first 2 days of life.
By convention, 4 GI conditions have the term meconium in their name: meconium ileus, meconium ileus–
equivalent syndrome, meconium peritonitis, and meconium plug syndrome.
Presentation:

 The meconium sometimes becomes thickened and congested in the intestines, a condition known
as meconium ileus. Most are associated with cystic fibrosis (but only 15% of infants with CF will have
meconium ileus)
 Meconium ileus is often the first sign of cystic fibrosis.
 Obstruction of bowel by thick tenacious meconium.
 30% of intestinal obstruction in neonates.
 Frequent cause of meconium peritonitis.
 Abdominal distention is typically present at birth
Diagnosis made with contrast enema
Treatment:

 Gastrograffin enema with aggressive hydration can be used to treat some.


 Operative evacuation of meconium.
 May require ostomy.
 Proximal bowel dilated and distal bowel may be very small (microcolon) and require time to dilate
with use.
b. Hydrocephalus. See the question 2.e of Group – B of supplementary paper – II of 2011.
c. Intravenous anaesthetics.
Classification of intravenous anaesthetics:

Properties of the ideal intravenous anaesthetic agent:

o Rapid onset – this is achieved by an agent which is mainly unionized at blood pH and which is
highly soluble in lipid; these properties permit penetration of the blood–brain barrier
o Rapid recovery – early recovery of consciousness is usually produced by rapid redistribution of
the drug from the brain into other well-perfused tissues, particularly muscle.
o Analgesia at subanaesthetic concentrations
o Minimal cardiovascular and respiratory depression
o No emetic effects
o No excitatory phenomena (e.g. coughing, hiccup, involuntary movement) on induction
o No emergence phenomena (e.g. nightmares)
o No interaction with neuromuscular blocking drugs
o No pain on injection
o No venous sequelae
o Safe if injected inadvertently into an artery
o No toxic effects on other organs
o No release of histamine
o No hypersensitivity reactions
o Water-soluble formulation
o Long shelf-life
o No stimulation of porphyria.

Indication Absolute contraindication


o Induction of anaesthesia o Airway obstruction – intravenous anaesthesia
o Maintenance of anaesthesia – thiopental is should not be used if there is anticipated difficulty
suitable only for short procedures because in maintaining an adequate airway, e.g. epiglottitis,
cumulation occurs with repeated doses oral or pharyngeal tumours.
o Treatment of status epilepticus o Porphyria – barbiturates may precipitate lower
o Reduction of intracranial pressure motor neurone paralysis or severe cardiovascular
collapse in patients with porphyria.
o Previous hypersensitivity reaction to a
barbiturate.

d. Post burn contracture.

Post-burn scars: Post-burn scars are inevitable even with the best of treatment because they depend upon
the depth of burn injury. Except for the superficial dermal burns, all deeper burns (2nd degree deep dermal
and full thickness) heal by scarring.This scarring can only be minimised by various physical therapy
measures and plastic surgical procedures but not eliminated completely.

Post-burn scar contractures


A burn patient who receives the best of treatment is expected to heal without any contractures. The
incidence of post-burn contractures is extremely high in our country. Quite often, they are not only multiple
in a given patient but also very severe and diffuse.
 The deeper tissues may be affected either due to their involvement in the initial burn injury (e.g.,
electrical burns) or secondary to the presence of a skin contracture over a prolonged period of many
years, which leads to shortening of musculo-tendinous units and neurovascular structures.
 The joints may be subluxated or dislocated, with joint capsule and ligaments becoming tight in the
direction of the contracture.
 The bones may be deformed, especially in growing children, e.g., mandibular deformity in cases of post-
burn contractures of the neck.
 Presence of one or more of above along with a contracture may alter the physical therapy and/or surgical
treatment of a contracture. For example, an unstable scar or chronic non-healing ulcer(s) will not heal
without surgical release of the contracture.
 Physical therapy prior to surgery may not be possible in these cases.
 Massive raw areas need wound closure with skin grafting before contracture can be subjected to
physical therapy.
 Wide excision of Marjolin’s ulcer has to be combined with release of contracting bands.
 A post-burn contracture associated with a hypertrophic or an atrophic scar or a depigmented area may all
need excision–release to achieve best results not only functionally but also aesthetically.
Prevention and treatment:
 The most important and effective method of controlling the wound contraction is to close the
wound at the earliest using split-skin grafts in deep dermal and full thickness burns.
 Contraction can be inhibited by applying grafts to fresh wounds (as in early excision) or over
healthy granulating areas (after eschar separation). Although full thickness skin grafts inhibit
contraction almost completely, it is not possible in a clinical setting.
 The split-skin grafts may also need expansion with meshing in extensive burns. Although this leads
to complete healing of the wound, the latter is largely covered with epithelium in the interstices of
the meshed graft.
 It is widely believed that thicker the graft, greater will be the inhibition of contraction. This holds
good only if the grafts are harvested from a given site.

Timing of surgery in post-burn contractures;


As a general “rule”, surgical intervention for post-burn contractures should not be undertaken during the
active phase of healing and scarring, i.e., as long as the scar is immature and highly vascular. This usually
takes 1 year or so.
With the passage of time, some mild contractures may improve with a better final result than if they had
been surgically managed.
There are several exceptions to this general rule of scar maturation before doing surgical intervention.

 Ectropion of the eyelids, especially the upper eyelid with constant danger of keratoconjunctivitis,
corneal ulceration, scarring or perforation with loss of vision
 Incapacitating contracture of the neck with inability to look forwards
 Severe microstomia causing interference with adequate nutrition and maintenance of orodental
hygiene.

Surgical intervention:

Release of contracture
Complete release of contracture should be done, avoiding damage to any important underlying structure,
e.g., arteries, nerves, tendons, etc.
In general, a contracture should be released by incision rather than by excision.
Excision may, however, be required in certain circumstances, e.g., (a) small adjoining depigmented or
hypertrophic areas, excision of which will add to the final aesthetic result (b) atrophic/unstable scars/chronic
non-healing ulcer(s)/ discharging sinuses should be excised along with release of contracture to obtain
healthy bed for split-skin graft “take” (c) scars may also be excised so as to apply the graft/flap in
accordance with principles of aesthetic units. Partial excision of hypertrophic scars may sometimes be done,
e.g., in a case of post-burn contracture of neck, the scars may extend from chin, neck onto the chest and
even abdomen.
In general, the contracture should be released completely on the table in one go. However, in severe long-
standing contractures, there is considerable shortening of musculotendinous units and neurovascular
structures. Hence, it may not be possible to achieve complete release.

 Crippling contractures of hand, especially dorsal contracture with metacarpophalangeal joints going
in hyperextension leading to permanent damage to extensor mechanism with various deformities .
 Contractures of both the knees, which force the patient to be on “all the fours” and endangers the
very dignity of being an upright human being
 Post-burn contractures with associated adjoining chronic raw areas needing skin cover
 Contractures with infected hypertrophic scars and abscesses, which need excision/drainage for their
recovery
 Any severe, incapacitating contracture unlikely to improve at all with physical therapy measures

After the complete release of a post-burn contracture, the recreated defect has to be covered using skin
grafts or a skin flap. Most commonly, the raw areas resulting after release of post-burn contractures are
covered with skin grafts. Flap covers are used in special situations.

e. FAST.
Introduction:
Focused assessment with sonography for trauma (commonly abbreviated as FAST) is a rapid
bedside ultrasound examination performed by surgeons, emergency physicians and certain paramedics as
a screening test for blood around the heart (pericardial effusion) or abdominal organs (hemoperitoneum)
after trauma.
Four primary views.
• The right upper quadrant (RUQ)
• The subxiphoid
• The left upper quadrant (LUQ)
• Suprapubic
 Other views are often used
 1/2 of the positive tests will reveal blood in the RUQ.
The four classic areas that are examined for free fluid are the:
o Perihepatic space (also called Morison's pouch or the hepatorenal recess),
o Perisplenic space,
o Pericardium, and
o Pelvis.
With this technique it is possible to identify the presence of intraperitoneal or pericardial free fluid. In
the context of traumatic injury, this fluid will usually be due to bleeding.
Advantages:
FAST is less invasive than diagnostic peritoneal lavage, involves no exposure to radiation and is cheaper
compared to computed tomography, but achieves a similar accuracy.
Pitfalls:
• Failure to scan Morison's pouch in the vertical plane, ideally from the midclavicular line. A
horizontal scanning plane in the patient's midaxillary line may miss free fluid.
• Excessive focus on the required views.
• Failure to scan systematically and slowly through the four areas in real time.
• Failure to identify clotted blood.
• Failure to consider ascites as a cause for free fluid.
• The only thing worse than a slow FAST is an inaccurate FAST.

Group – D
3. Answer briefly on (any three): 3x5

a. Tension pneumothorax.
b. Epulis.
c. Hospice.
d. Glasgow coma scale.
e. Differential diagnosis of intracranial space occupying lesion.
Answer.
a. Tension pneumothorax. See the question 4.e of Group – D of supplementary paper-II of 2013.

b. Epulis. See the question 4.b of Group – D of paper-II of 2011.

c. Hospice.
Introduction: Hospice care is a type of care and philosophy of care that focuses on the palliation of
a chronically ill, terminally ill or seriously ill patient's pain and symptoms, and attending to their
emotional and spiritual needs.
Hospice:
 Provides support and care for those in the last phases of life-limiting illness
 Recognizes dying as part of the normal process of living
 Affirms life and neither hastens nor postpones death
 Focuses on quality of life for individuals and their family caregivers.
 Provided to those with a terminal diagnosis and life expectancy of ≤ 6 months

Core aspects:
 Patient/family focused
 Interdisciplinary
 Provides a range of services:
• Interdisciplinary case management
• Pharmaceuticals
• Durable medical equipment
• Supplies
• Volunteers
• Grief support

Additional services:
 Hospices offer additional services, including:
• Hospice residential care (facility)
• Inpatient hospice care
• Palliative care
• Complementary therapies
• Specialized pediatric team
• Caregiver training classes

Hospice team members:

 The patient's personal physician  Social workers


 Hospice physician (medical director)  Clergy or other counselors
 Nurses  Trained volunteers
 Home health aides  Speech, physical, and occupational
therapists

The Hospice team:

 Develops the plan of care  Teaches the family how to provide care.
 Manages pain and symptoms  Advocates for the patient and family.
 Attends to the emotional, psychosocial and  Provides bereavement care and counseling.
spiritual aspects of dying and caregiving
Admission criteria:
 General:

 Life-limiting illness, prognosis is 6 months or less if disease takes normal course


 Live in service area
 Consent to accept services
Palliative care:
 Treatment that enhances comfort and improves the quality of an individual’s life during the last
phase of life.
 The expected outcome is relief from distressing symptoms, the easing of pain, and/or enhancing
the quality of life.
Curative and palliative care:
 Curative care
• Focuses on quantity of life and prolonging of life
 Palliative care
• Focuses on quality of life and death, and views death as a natural part of life.

d. Glasgow coma scale.


The Glasgow Coma Scale or GCS is a neurological scale that aims to give a reliable, objective way of
recording the conscious state of a person for initial as well as subsequent assessment. A patient is
assessed against the criteria of the scale, and the resulting points give a patient score between 3
(indicating deep unconsciousness) and either 14 (original scale) or 15 (the more widely used modified
or revised scale).
GCS was initially used to assess level of consciousness after head injury, and the scale is now used by
first aid, EMS, nurses and doctors as being applicable to all acute medical and trauma patients.
In hospitals it is also used in monitoring chronic patients in intensive care.

Glasgow coma scale (GCS)

 Eye response (E)


 Verbal response (V)
 Motor response (M)
Minimum score, 3; maximum score, 15.

Generally, brain injury is classified as:


–9
or 9–12 (controversial)
Minor, GCS ≥ 13.

 Generally when a patient is in a decline of their GCS score, the nurse or medical staff should assess the
cranial nerves and determine which of the twelve have been affected.
 Tracheal intubation and severe facial/eye swelling or damage make it impossible to test the verbal and
eye responses. In these circumstances, the score is given as 1 with a modifier attached e.g. "E1c" where
"c" = closed, or "V1t" where t = tube. A composite might be "GCS 5tc". This would mean, for example,
eyes closed because of swelling = 1, intubated = 1, leaving a motor score of 3 for "abnormal flexion".
Often the 1 is left out, so the scale reads Ec or Vt.
 The GCS has limited applicability to children, especially below the age of 36 months (where the verbal
performance of even a healthy child would be expected to be poor). Consequently the Pediatric Glasgow
Coma Scale, a separate yet closely related scale, was developed for assessing younger children.
e. Differential diagnosis of intracranial space occupying lesion.

Classification of intra cranial space occupying lesions:-


I. Congenital :- Dermoid, Epidermoid, Teratoma.
II. Traumatic :- Subdural & Extradural haematoma
III. Inflammatory :- Abscess, Tuberculoma, Syphiliti gumma, fungal granulomas.
IV. Parasitic :- Cysticercosis, Hydratid cyst, Amebic abscess, Schistosoma japonicum.
V. Neoplasms.

Causes of space occupying lesions

Malignancy Other space-occupying lesions

 Metastases, gliomas, meningiomas, pituitary  A haematoma may follow a head injury. Risk
adenomas, and acoustic neuromasaccount for factors include old age and anticoagulation.
95% of all brain tumours.  Hydrocephalus.
 Cerebral abscesses are uncommon but risk factors
In adults, two thirds of primary brain tumours include COPD that may be a source of infection to
are supratentorial, but in children, two thirds of the systemic circulation and a right-to-left shunt
brain tumours are infratentorial. that permits infection to bypass the lungs that
would normally filter it out. Cerebral abscesses are
 Primary cerebral tumours include astrocytomas, multiple in 25% of cases.
glioblastoma multiforme, oligodendrogliomas  Cysts that may occur in the brain include arachnoid
and ependymomas cysts (in the subarachnoid space), colloid cysts,
dermoid cysts and epidermoid cysts.
About 30% of brain tumours are metastatic and  Cerebral amoebiasis and cysticercosis are rare.
of these about 50% are multiple.  Both infection and lymphomas of the CNS are more
common with HIV infection.
 The most common primary is lung  Granuloma and tuberculoma can occur.
cancer followed by breast cancer, carcinoma of
the colon and malignant melanoma.

Differential diagnosis:
 Stroke  Vasculitis including systemic lupus  Encephalitis
erythematosus, syphilis, polyarteritis
nodosa andgiant cell arteritis
 Head injury  Multiple sclerosis  Post-ictal state (Todd's palsy)
The West Bengal University of Health Sciences

M.B.B.S. 3rd Professional Part – II Examination (Supplementary), 2009

Subject: Surgery Time: 21/2 hrs.

Paper: II Marks: 60

Group –A

1. a) Define hydronephrosis. What are the causes of unilateral hydronephrosis? Discuss


management of a patient with stone in middle third of ureter. 2+4+9

Answer. See the answer of question no. 1 of Group – A paper – II of 2013


Or

b) A 56 year old lady presents with a lump in upper and outer quadrant of Right breast.

Discuss briefly the management of such a patient. 15

Answer: The management will be as follow:

Relevant History in Women with Palpable Breast Masses

Breast lump Current medications Personal history of Age at menopause


characteristics breast cancer
Changes in size over History of hormone Previous breast Current lactation
time therapy masses and biopsies status
Change relative to Family history Recent breast trauma History of
menstrual cycle or surgery breastfeeding
Duration of mass History of breast Other exposure to Number of children
disease radiation
Pain or swelling Relationship to Personal Social history
patient characteristics
Redness, fever, or Relative’s age at Age at first Radiation and
discharge onset childbearing chemical exposure
Diet and medications Medical and surgical Age at menarche Smoking
history
Risk factors for breast cancer:

Well-established risk factors Probable risk factors Possible risk factors


 Age 50 or older  Alcohol consumption  Chemical exposure
 Benign breast disease, especially  Did not breast feed  Diet high in fat
cystic disease, proliferative types of  Elevated endogenous estrogen  Diet low in beta carotene,
hyperplasia, and atypical levels folate, and vitamins A and C
hyperplasia  High BMI  Diet low in fruits and
 Exposure to ionizing radiation  Hormonal contraception vegetables
 First childbirth after age 20 therapy
 Higher socioeconomic status  Increased mammographic
 History of breast cancer density of breast tissue
 History of breast cancer in a first-  Menarche before age 12
degree relative  Menopause after age 45
 Hormone therapy  Mutations in BRCA 1 and
 Nulliparity BRCA 2 genes
 Obesity (i.e., BMI ≥ 30 kg per m2)

Investigations:
Imaging:

Ultrasonography
Ultrasonography can effectively distinguish solid masses from cysts, which account for approximately
25 percent of breast lesions. Although ultrasonography is not considered a screening test, it is more
sensitive than mammography in detecting lesions in women with dense breast tissue. It is useful in
discriminating between benign and malignant solid masses, and it is superior to mammography in
diagnosing clinically benign palpable masses.

Diagnostic mammography
Diagnostic mammography can help physicians determine whether a lesion is potentially malignant, and
it also screens for occult disease in surrounding tissue.

Digital mammography
Digital mammography allows images to be enhanced and transmitted electronically. The ability to alter
contrast and brightness permits further evaluation of abnormal areas to identify features diagnostic of
benign and malignant disease. In addition to its usefulness in tele-mammography, digital mammography
may be more accurate than traditional mammography.

Magnetic resonance imaging


Magnetic resonance imaging (MRI) is being studied to determine its usefulness in diagnosing breast
masses. Gadolinium contrast is used to enhance the vascularity of malignant lesions. MRI is inferior to
mammography in detecting in situ cancers and cancers smaller than 3 mm, and it provides no cost
benefit over excisional biopsy for verifying malignancy.

MRI improves detection of early breast cancers in carriers of BRCA mutations, it has a lower
specificity than mammography, which requires additional evaluations. It also has a limited sensitivity in
detecting ductal carcinoma in situ.

Tissue Studies
Fine-needle aspiration
The first step in evaluating patients with palpable breast masses often is fine-needle aspiration (FNA).

FNA also is used with ultrasonography or stereotactic imaging to further assess poorly defined palpable
masses.

Core-needle biopsy
Core-needle biopsy (CNB) produces a larger tissue sample than FNA and may be used in conjunction
with ultrasonography or stereotactic imaging for small or difficult-to-palpate lesions.

Gold standard for tissue diagnosis.

Excisional biopsy:
Excisional biopsy is diagnostic and therapeutic: a completely removed mass with good margins of
normal tissue.

Triple Test
The triple test is the combination of results from CBE (complete breast examination), imaging, and
tissue sampling.

Screening for breast cancer:

Diagnostic evaluation process:


If the patient is found positive for breast cancer:

Look for BRCA1 and BRCA2 gene mutation

Stage the disease.

Look for metastatic disease.

Go for appropriate treatment.


Group – B

2. Write short notes on (any three): 3x5

a. Hypospadias.
b. Marjolin’s ulcer.
c. Collar stud abscess.
d. Venous ulcer.
e. Management of Hirschprung’s disease.

Answer.

a) Hypospadias. (See question 4.c of Group-D of supplementary paper II of 2013)


b) Marjolin’s ulcer. See the oquestion 4.e of Group - D of paper – II of 2013
c) Collar stud abscess.

Also known as Tuberculous lymphadenitis.

Tuberculous lymphadenitis (or tuberculous adenitis) is a chronic specific granulomatous inflammation of the lymph
node with caseation necrosis, caused by infection with Mycobacterium tuberculosis or Mycobacterium bovis. The
characteristic morphological element is the tuberculous granuloma(caseating tubercule). This consists of giant
multinucleated cells and (Langhans cells), surrounded by epithelioid cells aggregates, T
celllymphocytes and fibroblasts. Granulomatous tubercules eventually develop central caseous necrosis and tend to
become confluent, replacing the lymphoid tissue.
Stages of Tubercular Lymphadenitis -

1. Lymphadenitis
2. Periadenitis
3. Cold abscess
4. 'Collar stud' abscess
5. Sinus

Diagnosis:

 A high index of suspicion is needed for the diagnosis of mycobacterial cervical lymphadenitis.
 A thorough history and physical examination, tuberculin test, staining for acid-fast bacilli, radiologic
examination, and fine-needle aspiration cytology (FNAC) will help to arrive at an early diagnosis of
mycobacterial lymphadenitis.
 Smears: Smears can be obtained either from a draining sinus or by FNA. Ziehl-Neelsen staining of
the smears may reveal mycobacteria in the fresh specimens. Chance of finding AFB is higher in
patients with cold abscess.
Combination of FNA with culture or a Mantoux test further increases the diagnostic yield in
mycobacterial cervical lymphadenitis.
 Culture: Culture of mycobacterium is diagnostic for mycobacterial cervical lymphadenitis.
However, a negative culture result should not exclude the diagnosis of mycobacterial cervical
lymphadenitis. The presence of 10–100 bacilli per cubic millimeter of the specimen is enough for a
positive culture result.
 Tuberculin Test: This intradermal test (Mantoux test) is used to show delayed type hypersensitivity
reactions against mycobacterial antigen, in which the reagent is mostly protein purified derivative
(PPD). The test becomes positive 2–10 weeks after the mycobacterial infection.
o Positive reactions (>10-mm induration) can occur in M. tuberculosis infections.
o Intermediate reactions (5- to 9-mm induration) can occur after BCG vaccination,
M. tuberculosis infection or nontuberculous mycobacterial infections.
 Negative reactions (< 4-mm induration) represent a lack of tuberculin sensitization. False-negative
reactions can occur in about 20% of all persons with active tuberculosis. The test may be positive
in different conditions, like other infections, metabolic disease, malnutrition, live virus
vaccination, malignancy, immunosuppressive drugs, newborns, elderly people, stress, sarcoidosis
and inadequate test application.
 Polymerase chain reaction (PCR): It is a fast and useful technique for the demonstration of
mycobacterial DNA fragments in patients with clinically suspected mycobacterial lymphadenitis.
 Histopathology: Histopathologic examination is diagnostic of mycobacterial cervical
lymphadenitis.
 Radiology and imaging: Chest radiograph, ultrasound, CT and MRI of the neck can be performed
in mycobacterial lymphadenitis.
o Associated chest lesions as seen on chest radiography are very common in children but less
common in adults.
o Ultrasound of the neck can demonstrate singular or multiple hypoechoic and multiloculated cystic
lesions that are surrounded with thick capsule.
o On CT, the presence of conglomerated nodal masses with central luscency, a thick irregular rim
of contrast enhancement and inner nodularity, a varying degree of homogeneous enhancement in
smaller nodes, dermal and subcutaneous manifestations of inflammation, such as thickening of
the overlying skin, engorgement of the lymphatics and thickening of the adjacent muscles, and a
diffusely effaced fascial plane may suggest mycobacterial cervical lymphadenitis.

Treatment:

 Antituberculosis treatment is the mainstay in the management of TB lymphadenitis.


 In addition surgical treatment is more useful in cases where abscess has been formed.

d) Venous ulcer. See the question 4.c of Group – D of paper II of 2014.


e) Management of Hirschprung’s disease.
Introduction:

 Incidence 1 in 5000 live births.


 Commoner in males.

Pathological features:

 Due to incomplete migration of neural crest cells into the hindgut resulting in distal aganglionosis
and failure of coordinated peristaltic waves, abnormal anorectal relaxation, and loss of recto-anal
inhibitory reflexes.
 May involve:
o Just the anorectal junction (ultrashort segment): often presents in adult life;
o The rectum and rectosigmoid (short segment; 70%): presents in infancy or early childhood;
o Extensive colonic involvement (long segment): rare.
 The proximal (normal) bowel becomes progressively distended due to buildup of faecal matter.

Clinical features:

 There is failure to pass meconium within 24-48h, abdominal distension, and bile vomiting.
 It may be associated with Down's syndrome.
 It may present late with poor weight gain, offensive diarrhoea, or enterocolitis.

Management:

 Diagnosis
o History
 Neonates: abdominal distention, infrequent defecation, failure to pass meconium within the first 48
hours of life, or enterocolitis with sepsis.
 Older infants and children present with chronic constipation or failure to thrive.
o Radiology
 Plain abdominal radiographs commonly show a pattern of distal obstruction.
 Barium enema
 Usually demonstrates a transition zone between distal nondilated bowel and
proximal dilated bowel.
 Most common in the rectosigmoid, but may be seen anywhere in the colon.
 Total colonic aganglionosis: no transition zone.
o Pathology
 Rectal biopsy is essential for making the diagnosis.
 Full-thickness specimens are the ideal tissue samples to allow identification of the absence of
ganglion cells.
 In neonates, rectal suction biopsy often is sufficient for diagnosis.
 Preoperative management
o Colonic decompression to prevent enterocolitis. Saline enemas may be used to evacuate
impacted stool.
o Nasogastric tube should be placed if the child is vomiting.
 Operative treatment
o Goals of surgical management are removal of aganglionic bowel and reconstruction of the
intestinal tract by bringing the innervated bowel down to the anus while maintaining normal
sphincter function.
o Primary pull-through
 Swenson.
 Duhamel.
 Soave.
 Laparoscopic endorectal pull-through (Georgeson).
 Transanal endorectal pull-through (Langer).

Each of these operations has been modified to improve functional results and may be performed in the
newborn period, although surgery may delayed to allow for increased weight gain or resolution of
enterocolitis.

Diverting colostomy

 Performed proximal to aganglionic segment in patients who are unstable or who have massively
dilated bowel.
 Daily rectal irrigations may obviate the need for a colostomy.

Group – C

3. Write short notes on (any three ) 3x5

a. Cleft palate.
b. Target FNAC.
c. Complications of Radiotherapy.
d. Regional anaesthesia.
e. Empyema thoracis.

Answer.

a. Cleft palate.
Cleft palate is types of clefting congenital deformity caused by abnormal facial development during
gestation.
A cleft is a fissure or opening gap.
It is the non-fusion of the body’s natural structures that form before birth.
Clefts can also affect other parts of the face, such as the eyes, ears, nose, cheeks and forehead.
Anatomy:
Hard Palate
– Bones: Maxilla( Palatine Processes) + Palatine Bones(Horizontal Lamina)
– Blood Supply: Greater Palatine Artery
– Nerve Supply: Anterior Palatine Nerve

Soft Palate
– Fibromuscular shelf attached like a shelf to posterior portion of hard palate
– Tenses, elevates, contracts Passavant’s Ridge
– Muscles: Tensor Veli Palatini(CNV), Levator Veli Palatini(Primary Elevator), Musculus
Uvulae, Palatoglossus, Palatopharyngeus(CN IX and X)

Embryology:
Primary Palate- Triangular area of hard palate anterior to incisive foramen to point just lateral to
lateral incisor teeth
– Includes that portion of alveolar ridge and four incisor teeth.
Secondary Palate- Remaining hard palate and all of soft palate
Primary Palate
– Forms during 4th to 7th week of Gestation
– Two maxillary swellings merge
– Two medial nasal swelling fuse
– Intermaxillary Segment Forms: Labial Component(Philtrum)
– Maxilla Component(Alveolus + 4 Incisors) Palatal Component(Triangular Primary
Palate)

Secondary Palate

– Forms in 6th to 9th weeks of gestation


– Palatal shelves change from vertical to horizontal position and fuse
– Tongue must migrate antero-inferiorly.
Primary palate Secondary palate

 Clefts of the primary palate : result from a failure of mesoderm to penetrate into the grooves
between the medial nasal and maxillary processes, which prohibits their from fusion with each
another
 Clefts of the secondary palate: result from a failure of the palatine shelves to fuse with one
another. The cause for this is failure of the tongue to descend into the oral cavity.
Normal palate Cleft palate

Classification:
 Group I (A):-
Defects of the soft palate alone
 Group II (B)
Defects involving the hard and soft palates (not extending anterior to the alveolus)
 Group III (C)
Defects involving the palate through to the alveolus
 Group IV (D)
Complete bi-lateral clefts.
Complications:
 Feeding difficulties:
One of the most immediate concerns after birth is feeding as cleft palate make sucking difficult or
cause gagging or nasal regurgitation
To overcome this problem by using a special bottle nipple or a small artificial palate (obturator) that
fits into the roof of the mouth .
The upright sitting position allows gravity to help the baby swallow the milk more easily
 Ear infections and hearing loss:
Babies with cleft palate are especially susceptible to middle ear infections. Ear infections are often
due to a dysfunction of the tube that connects the middle ear and the throat.
Over time, repeated ear infections can damage hearing, but hearing loss may resolve with treatment.
It's important for children with cleft palate to be evaluated regularly by an audiologist.
Most children with clefts have tubes inserted in their ears to drain fluids and help prevent infections.
 Dental problems:
If the cleft extends through the upper gum, tooth development will likely be affected.
A pediatric dentist should monitor tooth development and oral health from an early age.
 Speech difficulties:
Because both the lip and palate are used in forming sounds, the development of normal speech can
be affected. A speech pathologist can evaluate your child and provide speech therapy.
 Psychological challenges:
Children with clefts may face social, emotional and behavioral problems due to differences in
appearance and the stress of intensive medical care.

Diagnosis:

 Prenatal ultrasounds can detect a cleft palate prior a child's birth.


 By detecting the cleft abnormality during a pregnancy, the expecting parents can have a prenatal
consultation with a plastic surgeon.
 The symptoms of these abnormalities are visible during the first examination after birth.

Treatment:

 Children born with a cleft palate may need the skills of several medical professionals to correct the
problems associated with the cleft
 Treatment usually requires a complex, lengthy treatment plan lasting until adulthood (see table
below).

Age Intervention

Prenatal Referred to cleft lip and palate


team
Birth-1 month Diagnosis and genetic
counseling
Address psychosocial issues
Provide feeding instructions
Make feeding plan
1-15 months Check feeding and growth
Repair cleft lip
Check ears and hearing

16-24 months Assess ears and hearing


Assess speech and language
Check development
2-5 years Assess , manage
velopharyngeal insufficency
Assess development and
psychosocial adjustment

6-11 years Orthodontic interventions


Alveolar bone graft

12-21 years Jaw surgery, rhinoplasty


as needed
Orthodontic bridges implants as
needed

Aims of repair:
 Separate the oral and nasal cavities, this separation involves the formation of a valve that is
necessary for normal speech.
Also the muscles at the back of the palate need to be put in their proper place across the
Adjunct tools:
 Cell blocks
 Histochemistry
 Immunohistochemistry
 Electron microscopy
 Flow cytometry
 Immuno electron microscopy
 Molecular pathology -In situ hybridization, PCR etc
Future directions:
 Aspirating non palpable lesions using MRI
 Molecular pathology eg In Situ Hybridization
 Replacing diagnostic surgical pathology?
 Combined with MRI - replacing autopsy?
 cleft so the child can learn to speak normally.

Time of operation:

 General agreement exists that surgical correction of a cleft palate should be accomplished when
patients are younger than 1 year, before significant speech development occurs.
b. Target FNAC.

Introduction:
 Aspiration of cells/ tissue fragments using fine needles ( 22 , 23, 25 G) ; external diameter 0.6 to
1.0 mm
 1.5 inches long needle ( radiologists use longer needles)
 Diagnostic materials in the needle and not in the syringe even in cystic lesions
Clinical skill required:
 Familiarity with general anatomy eg thyroid vs other neck swelling
 Ability to take a focused clinical history
 Sharp skill in performing physical examination eg solid vs cystic, benign vs maligant lesions
 Good knowledge in normal cellular elements from various organs and tissue and how they
appear on smears eg fats cells vs breast tumour cells
 Comprehensive knowledge of surgical pathology
 Ability to translate traditional tissue patterns of lesions to their appearance in smears
Limitations:
 Soft vs hard ( bone) lesions
 Solid vs cystic lesions
 Poor cellular yield vs poor technique
 Reactive vs specific diseases eg reactive lymphadenitis vs Hodgkins disease
 Diffuse vs nodular lymphoma.

Guided FNAC:

 USG guided:Ultrasound-Guided FNAC: this is the preferred method in some centres and is
particularly useful in the staging of head and neck lesions, non-palpable breast lesions and
thyroid lesions, in the case of the latter by helping to avoid surgery in 37% of cases.
 Endoscopy-Guided Ultrasound FNAC: the use of endoscopy-guided ultrasound (EUS) FNAC
(EUS-FNAC) of the pancreas, mediastinum, duodenum, bile ducts, hypopharynx, rectum, lung
and other sites accessible through the endoscope is increasing.
 Computed Tomography (CT)-Guided FNAC: CT-guided FNAC is associated with high
diagnostic accuracy and a low rate of complications,particularly in the diagnosis of pulmonary
lesions.
c. Complications of Radiotherapy. See the question 3.e of Group – C of paper – II of 2009.
d. Regional anaesthesia.
Introduction:
❏ Local anesthetic applied around a peripheral nerve at any point along the length of the nerve
(from spinal cord up to, but not including, the nerve endings) for the purposes of reducing or
preventing impulse transmission
❏ No CNS depression (unless overdose (OD) of local anesthetic); patient conscious
❏ Regional anesthetic techniques categorized as follows
• Epidural and spinal anesthesia
• Peripheral nerve blockades
• IV regional anesthesia

Physiology:
• Physiologic response to central blockade is determined by the effects of interrupting the afferent
and efferent innervation of somatic (sensory and motor innervation) and visceral (autonomic
nervous system).
• Somatic blockade:
Prevention of pain.
Skeletal muscle relaxation.

• Fibers blocked more easily small and myleinated.


• Less easily – large – unmylinated.

Indications Contraindications Complications


❏ Avoidance of some of the ❏ Allergy to local anesthetic ❏ Failure of technique
dangers of general anesthesia ❏ Patient refusal, lack of ❏ Systemic drug toxicity
(e.g. known difficult intubation, cooperation due to overdose or
severe respiratory failure, etc.) ❏ Lack of resuscitation intravascular injection
❏ Patient specifically requests equipment ❏ Peripheral neuropathy
regional anesthesia ❏ Lack of IV access due to intraneural injection
❏ For high quality post-op pain ❏Coagulopathy ❏ Pain or hematoma at
relief ❏ Certain types of preexisting injection site
❏ General anesthesia not neurological dysfunction
available ❏ Local infection at block site

Epidural and spinal anaesthesia :


 Anatomy of Spinal/Epidural Area
o The spinal cord lies within the spinal canal.
o Surround by meanings, dura mater sub archnoid space, then piamatter, end by hoarse tail
(Couda equina)
o The spinal cord receives blood supply from ant spinal artery, and posterior spinal artery.
o Spinal cord extends to L2, dural sac to S2
o Nerve roots (cauda equina) from L2 to S2
o Needle inserted below L2 should not encounter cord, thus L3-L4, L4-L5 interspace commonly
used
Structures penetrated:
• Skin, subcutaneous fat
• Supraspinous ligament
• Interspinous ligament
• Ligamentum flavum (last layer before epidural space)
• Dura + arachnoid for spinal anesthesia
 Spinal anaesthesia:

❏ Relatively small LA dose injected into subarachnoid space in the dural sac surrounding the spinal
cord + nerve roots
❏ LA solution may be made hyperbaric (of greater specific gravity (SG) than the cerebrospinal fluid
(CSF) by mixing with 10% dextrose, thus increasing spread of LA to the dependent (low) areas of
the subarachnoid space
Epidural anaesthesia:
❏ LA deposited in epidural space (potential space between ligamentum flavum and dura)
❏ Solutions injected here spread in all directions of the potential space; SG of solution does not
affect spread
❏ Initial blockade is at the spinal roots followed by some degree of spinal cord anesthesia as LA
diffuses into the subarachnoid space through the dura
❏ Larger dose of LA used
Spinal vs epidural anesthesia:
SPINAL EPIDURAL
 Easier to perform • Technically more difficult; greater failure rate
 Smaller dose of LA required (usually < • Larger volume/doses of LA (usually > toxic IV
toxic IV dose) dose)
 Rapid blockade (onset in 2-5 minutes) • Significant blockade requires 10-15 minutes
 Very effective blockade • Effectiveness of blockade can be variable
Hyperbaric LA solution - position of • Use of catheter allows for continuous infusion or
patient important repeat injections
• Slower onset of side effects
• Position of patient not as important
• SG of LA solution not as important

Complications:
Spinal Anaesthesia Epidural Anaesthesia
– Failure of technique – Failure of technique
– Hypotension, bradycardia if block reaches – Hypotension - common
T2-4 (sympathetic nervous system (SNS) – Bradycardia if cardiac sympathetics
block) blocked (only if T2-4 block)
– Post-spinal headache – Systemic toxicity of LA (accidental
– Extensive spread of anesthetic ("high intravenous)
spinal") – Accidental subarachnoid injection can lead
– Persistent paresthesias (usually transient) to total spinal anesthesia
– Epidural or subarachnoid hematoma – Catheter complications (shearing, kinking,
– Spinal cord trauma, infection vascular or subarachnoid placement)
– Epidural or subarachnoid hematoma

 Peripheral nerve blockade:


• The use of ultrasound for regional anesthesia is relatively new.
• Ultrasound guided regional anesthesia have largely focused on brachial plexus blockade in the
interscalene, supraclavicular, infraclavicular and axillary regions.
• Recent studies examining the efficacy of ultrasound guidance for femoral, sciatic, psoas
compartment, celiac plexus and stellate ganglion blocks are promising, while ultrasound
visualization of the epidural space can facilitate neuraxial blockade in children, adults and
parturients.

e. Empyema thoracis. See the question 3.c of Group – C of supplementary paper – II of 2013.

Group – D
4. Answer briefly on (any three): 3x5

a) Mammography.
b) Nerve injury.
c) Ultrasonic therapy.
d) Adamentinoma.
e) Solitary thyroid nodule.

Answer.

a) Mammography. Mammography is the process of using low-energy X-rays (usually around 30 kVp)
to examine the human breast, which is used as a diagnostic and screening tool. The goal
of mammography is the early detection of breast cancer, typically through detection of characteristic
masses and/or microcalcifications.

It is performed in the asymptomatic patient and consists of two standard views, mediolateral oblique (MLO)
and craniocaudal (CC).
Screening mammography:The current recommendation from the National Cancer Institute and American
College of Surgeons is annual screening mammography for women aged 40 years and older. Breast lesions
on mammograms are classified according to the American College of Radiology by BI-RADS (Breast
Imaging Reporting and Database System) scores:

0 = Needs further imaging; assessment incomplete.


1 = Normal; continue annual follow-up (risk of malignancy: 1/2,000).
2 = Benign lesion; no risk of malignancy; continue annual follow-up (risk of malignancy: 1/2,000).
3 = Probably benign lesion; needs 4 to 6 months follow-up (risk of malignancy: 1% to 2%).
4 = Suspicious for breast cancer; biopsy recommended (risk of malignancy: 25% to 50%).
5 = Highly suspicious for breast cancer; biopsy required (75% to 99% are malignant).
6 = Known biopsy-proven malignancy.

 Malignant mammographic findings


o New or spiculated masses.
o Clustered microcalcifications in linear or branching array.
o Architectural distortion.
 Benign mammographic findings
o Radial scar. Generally due to fibrocystic breast condition (FBC); associated with proliferative
epithelium in the center of the fibrotic area in approximately one third of cases. Appearance
often mimics malignancy; a biopsy is needed to rule out malignancy.
o Fat necrosis. Results from local trauma to the breast. It may resemble carcinoma on palpation
and on mammography. The fat may liquefy instead of scarring, which results in a characteristic
oil cyst. A biopsy may be needed to rule out malignancy.
o Milk of calcium. Associated with FBC; caused by calcified debris in the base of the acini.
Characteristic microcalcifications appear discoid on craniocaudal view and sickle-shaped on
mediolateral oblique view. These are benign and do not require biopsy.
o Cysts cannot be distinguished from solid masses by mammography; ultrasound is needed to
make this distinction.

Diagnostic mammography:Diagnostic mammograms are performed in the symptomatic patient or to


follow up on an abnormality noted on a screening mammogram. Additional views (spot-compression
views or magnification views) may be used to further characterize any lesion. The false-negative and
false-positive rates are both approximately 10%.

Mammography guided biopsy can also be taken.

a) Nerve injury.

Definition:
Partial or complete interruption of normal physiology of the nerve leading to a condition where
nerve conduction is affected.
Classification: Seddon described three basic types of peripheral nerve injury
Neurapraxia (Class I) Axonotmesis (Class II) Neurotmesis (Class III)

It is a temporary interruption of It involves loss of the relative It is a total severance or


conduction without loss of axonal continuity of the axon and its disruption of the entire nerve
continuity.In neurapraxia, there is a covering of myelin, but fiber. A peripheral nerve
physiologic block of nerve preservation of the connective fiber contains an axon (Or
conduction in the affected axons. tissue framework of the nerve long dendrite), myelin
( the encapsulating tissue, the sheath (if existence), their
epineurium and perineurium, schwann cells, and the
are preserved ). endoneurium. Neurotmesis
may be partial or complete.
Other characteristics: Other characteristics: Other characteristics:

 It is the mildest type of  Wallerian  Wallerian degeneration


peripheral nerve injury. degeneration occurs distal occurs distal to the site
 There are sensory-motor to the site of injury. of injury.
problems distal to the site of  There are sensory and  There is connective
injury. motor deficits distal to the tissue lesion that may be
 The endoneurium, perineurium, site of lesion. partial or complete.
and the epineurium are intact.  There is no nerve  Sensory-motor problems
 There is no wallerian conduction distal to the and autonomic function
degeneration. site of injury (3 to 4 days defect are severe.
 Conduction is intact in the after injury).  There is no nerve
distal segment and proximal  EMG shows fibrillation conduction distal to the
segment, but no conduction potentials (FP),and site of injury (3 to 4
occurs across the area of injury. positive sharp waves (2 to days after lesion).
 Recovery of nerve conduction 3 weeks postinjury).  EMG and NCV findings
deficit is full,and requires days  Axonal regeneration are as axonotmesis.
to weeks. occurs and recovery is  Because of lack of nerve
 EMG shows lack of fibrillation possible without surgical repair, surgical
potentials (FP) and positive treatment.Sometimes intervention is
sharp waves. surgical intervention necessary.
because of scar
tissue formation is
required.

Sunderland’s classification:
 First-degree (Class I)
Seddon's neurapraxia and first-degree are the same.
 Second-degree (Class II)
Seddon's axonotmesis and second-degree are the same.
 Third-degree (Class II)
Sunderland's third-degree is a nerve fiber interruption. In third-degree injury, there is a lesion of the
endoneurium, but the epineurium and perineurium remain intact. Recovery from a third-degree injury is
possible, but surgical intervention may be required.
 Fourth-degree (Class II)
In fourth-degree injury, only the epineurium remains intact. In this case, surgical repair is required.
 Fifth-degree (Class III)
Fifth-degree lesion is a complete transection of the nerve. Recovery is not possible without an
appropriate surgical treatment.
Aetiology:
• Acute: • Chronic:

o Fracture o Tight nerve passage


o Wrong posture o Tumors
o Surgery
o Electrical burn

Presentation:

• Pain
• Loss of sensation
• Loss of motion
• Loss of power
• Loss of reflexes
• Wasting
• Trophic changes (skin,sc,neurovascular,bones,muscles)
• Contractures.
Diagnostic aids:

• X-RAY
• EMG
• NCS – Nerve conduction study.
• MRI

Clinical examples:

• Erb’s palsy
• Carpal tunnel syndrome (median nv)
• Radial nerve injury
• Ulnar nerve injury
• Sciatic nerve injury
• Lateral popliteal nerve injury

Nerve regeneration therapy:

Electrical stimulation can promote nerve regeneration. The frequency of stimulation is an important
factor in the success of both quality and quantity of axon regeneration as well as growth of the
surrounding myelin and blood vessels that support the axon. Histological analysis and measurement of
regeneration showed that low frequency stimulation had a more successful outcome than high
frequency stimulation on regeneration of damaged sciatic nerves.
The use of autologous nerve grafting procedures that involve redirection of regenerative donor nerve
fibers into the graft conduit has been successful in restoring target muscle function. Localized delivery
of soluble neurotrophic factors may help promote the rate of axon regeneration observed within these
graft conduits..

b) Ultrasonic therapy. Therapeutic ultrasound is extensively used by physiotherapists and is popular


with patients.
 Ultrasound is a form of MECHANICAL energy
 Mechanical vibration at increasing frequencies is known as sound energy.
 Below 16Hz, these vibrations are not recognizable as sound
 The normal human sound range is from 16Hz to something approaching 15-20,000 Hz
 Beyond this upper limit, the mechanical vibration is known as ULTRASOUND.
 The frequencies used in therapy are typically between 1.0 and 3.0 MHz
 1MHz = 1 million cycles per second
 As the energy within the sound wave is passed to the material, it will cause oscillation
of the particles of that material.
 Any increase in the molecular vibration in the tissue can result in heat generation, and
ultrasound (US) can be used to produce thermal changes in the tissues
 All sound is produced by the vibration of a membrane
 Ultrasound (> 20,000 Hz) is produced by the vibration of synthetic crystals
 The crystal contracts and expands when exposed to alternating electric current
 These oscillations of the crystal produce pressure waves = ultrasound waves.

Ultrasound is applied using a transducer or applicator that is in direct contact with the patient's skin. Gel
is used on all surfaces of the head to reduce friction and assist transmission of the ultrasonic waves.
Therapeutic ultrasound in physical therapy is alternating compression and rarefaction of sound waves
with a frequency of >20,000 cycles/second. Therapeutic ultrasound frequency used is 0.7 to 3.3 MHz.
Maximum energy absorption in soft tissue occurs from 2 to 5 cm. Intensity decreases as the waves
penetrate deeper. They are absorbed primarily by connective tissue: ligaments, tendons, andfascia (and
also by scar tissue).

• Conditions for which ultrasound may be used for treatment include the follow examples:
Ligament Sprains, Muscle Strains,Tendonitis, Joint Inflammation, Plantar fasciitis, Metatarsalgia, Facet
Irritation, Impingement syndrome, Bursitis,Rheumatoid arthritis, Osteoarthritis, and Scar Tissue
Adhesion.

Indications

1. Decrease pain and muscle spasm


2. Increase extensibility of collagen
3. Increase blood flow
4. Increase metabolic rate
5. Increase tissue healing
6. Organization of healing tissues

Contraindications

● Over the heart


● Over the cordal spine
● Over epiphyseal area of growing bones
● Infected areas
● Bleeding areas
● Neoplasms
● Thrombophlebitis

Precautions

● Over analgesic areas


● Myositis ossificans

● Fibrotic myopathy
● Plastic and metal implants

● Over the carotid sinus

● Over a pregnant uterus

c) Adamentinoma. See the question 4.b of Group – D of supplementary paper – II of 2011


d) Solitary thyroid nodule. See the question 1.a of Group – A of paper –II of 2011.
The West Bengal University of Health Sciences
M.B.B.S. 3rd Professional Part – II Examination (Supplementary), 2008
Subject: Surgery Time: 21/2
hrs.

Paper: II Marks: 60
Group –A
1. a) Enumerate the causes of acute retention of urine. What are the pathological changes
associated with prostatic hypertrophy? How will you manage such a patient?
5+5+5

Answer. See the question 1.b of supplementary paper – II of 2012.


Pathological changes:
 BPH is a benign prostatic hyperplasia of the epithelial and stromal components of the
prostate, which leads to a nodular enlargement of the organ.
 The enlargement of the periurethral prostate can affect the so-called middle lobe, the lateral
lobes or a combination of both. The first changes of BPH take place in the periurethral
glands around the verumontanum. Because of the rigid prostatic capsule, the lumen of the
prostatic urethra is compressed due to the prostatic hyperplasia. Since the size of the
prostate does not correlate with the degree of subvesical obstruction, anatomical and
functional variations are in addition responsible for symptomatic BPH.
 Benign prostatic hyperplasia is a true hyperplasia with an increase in cell number. The first
hyperplastic changes involve the stroma, followed by a small nodular hyperplasia of the
glandular tissue. At the cellular level, some hypertrophy (cell enlargement) is also present.
The nuclei of the glands show no signs of malignancy. In the further course of the disease,
large nodular hyperplasia develops.
 The significant increase in the number of smooth muscle cells in the prostate stroma results
in a dynamic component of subvesical obstruction, which is caused by smooth muscle
contraction. The contraction of smooth muscle cells in the prostate is mediated via alpha-
1A-adrenoceptors. In addition, smooth muscle cells of the prostate and bladder express
type 4 and type 5 phosphodiesterase isoenzymes. Treatment of erectile dysfunction with
inhibitors of phophodiesterases has a beneficial effect on symptomatic BPH.
 The initial response of the bladder for subvesical obstruction is hypertrophy of the detrusor
muscle; this allows the bladder to empty with higher voiding pressure. After muscular
hypertrophy, an increased collagen deposition leads to trabeculation of the bladder and
decreased bladder compliance. High voiding pressures lead to the formation of
pseudodiverticula that increase the functional residual urine.
 The response of the bladder to subvesical obstruction causes detrusor instability resulting in
frequency and urgency. The increasing dilatation of the bladder weakens the detrusor
muscle and leads to urinary retention. Later on BPH leads to the decompensation of the
bladder, resulting in progressive bladder dilatation, chronic urinary retention and urinary
(overflow) incontinence.
 BPH may lead to the decompensation of the upper urinary tract with hydronephrosis,
caused by micturition under high pressures, chronic urinary retention and vesicoureteral
reflux. Postrenal kidney failure is a late symptom of BPH and is usually associated
with chronic urinary retention with incontinence. The consequences of untreated postrenal
kidney failure are uremia and death.
 The relief of the urinary tract often leads to massive polyuria. Polyuria is caused by the
excretion of accumulated osmotically active substances. Secondly, the continuous
perfusion of the renal medulla without urine flow wiped out the corticomedullary osmotic
gradient, which is important for concentration of the urine.

Or
b) Discuss clinical features of pheochromocytoma. How do you diagnose this condition?
What will you do to manage it? 5+5+5
Answer. See the question 2.b of Group – B of supplementary paper – II of 2013

Group – B
2. Write short notes on (any three): 3x5

a. Carcinoid tumour.
b. Raynaud’s phenomenon.
c. Ranula.
d. Abdominal compartment syndrome.
e. Desmoids tumour.
Answer.
a. Carcinoid tumour.
Introduction:

 Carcinoid tumours, because of their neuroendocrine origin, have neurosecretory


capabilities and hence the ability to secrete a variety of peptides and bioactive amines.
 Chief among these are 5-hydroxytryptamine( 5-HT) or serotonin, histamine,
norepinephrine, ACTH, etc.
 They arise from cells of diffuse neuroendocrine system. These cells are found throughout
the GI tract and bronchopulmonary system.
 Carcinoid tumours occur most frequently in the GI tract, in bronchopulmonary system, and
other locations like ovary, gallbladder, thymus, testis, cervix, breast etc.
Pathology:
 Grossly carcinoid tumours appear as solid, yellow-tan lesions. They are usually solitary,
except in stomach and ileum where they may be multicentric.
 They are often submucosal and they may cause ulceration. There is intense desmoplasia
within and surrounding the tumour.
 Histologically they appear as uniform, small, round cells with rare mitotic figures. Two
types of silver staining are commonly used to identify neuroendocrine cells—argyrophil
reactions and argentafin staining. Foregut and hindgut carcinoid are typically argyrophil
and midgut (serotonin secreting) carcinoids are argentafin.
 Immunohistochemical stains like chromogranins A, B, and C are reliable serum markers
for detection of neuroendocrine tumours. About 80% of carcinoids will have elevated
levels of chromogranin A. A high level of chromogranin A is a poor prognostic indicator in
individuals with metastatic disease.
Clinical Presentation:
 Carcinoid tumours are often discovered incidentally during surgery, endoscopic procedures
or imaging studies.
 They may also present either by causing symptoms due to mass effect or due to their
secretory products.
 Mass effect may occasionally cause pain but more commonly causes luminal obstruction
(due to tumour growth or scarring).
 Symptoms are often non specific consisting of malaise,vague abdominal pain or weight
loss.
 Carcinoid syndrome is an unusual presentation of GI carcinoids.
 The classic product serotonin may produce diarrhea. ACTH producing tumours will cause
Cushing’s syndrome, while gastrin producing tumours may cause ZE syndrome.
Classification:
 Carcinoid tumours have been classified according to their embryologic region of
origin: foregut, midgut, hind gut.
 Foregut carcinoids include those arising in esophagus, stomach, pancreas, and duodenum.
 Midgut carcinoids comprise of those arising in the distribution of superior mesenteric
artery distribution, including appendix and ileum.
 Hind gut carcinoids include those arising in the distribution of inferior mesenteric artery
distribution including the rectum.

Esophagus
 Esophageal carcinoids are extremely rare. Dysphagia is the most common presentation.
 Mostly treated by esophagogastrectomy.
Stomach
 Stomach is the most common foregut location for carcinoid tumours.
 Pernicious anaemia and chronic atrophic gastritis are risk factors for the development of
gastric ECL cell tumour. This may be secondary to the hypergastrinemia caused by the
absence of significant acid output in such patients.
 They are much more common in those patients with MEN- type 1.
 The average age of presentation of patients with gastric carcinoid tumours is 62 years.
 Patients are often asymptomatic, carcinoids often found incidentally during upper GI
endoscopy.
 When symptoms occur, they are usually dyspeptic.
 EUS can be helpful in defining the depth of invasion and providing deep submucosal
 biopsy specimen. CT and MRI may be helpful in finding hepatic metastasis. All patients
with gastric carcinoids have elevations of chromogranin A levels.
 Management of gastric carcinoid tumours is controversial.
 Carcinoids secondary to pernicious anemia/ atrophic gastritis and MEN- 1 tend to have a
more benign course, with occasional spontaneous regression.
Pancreas
 The differentiation between pancreatic carcinoids and other neuroendocrine tumours
(NETs) of pancreas is primarily a matter of definition.
 Pancreatic carcinoids have been defined as a tumour with the histologic features of a
neuroendocrine tumour and with obviously increased serotonin metabolism.
 Pancreatic carcinoids tend to present late. Abdominal pain, diarrhea, and weight loss are
the most common presenting symptoms.
 Most patients presents with metastasis. Treatment is by surgical resection.
 Pancreaticoduodenectomy is often required.
 It has better survival rates than those of other pancreatic malignancies but worse than those
of carcinoids in other sites.
Duodenum
 Carcinoids of duodenum represent approximately 2% of carcinoids.
 Duodenal carcinoids most commonly discovered during endoscopy.
 They rarely produce serotonin and therefore virtually no association with carcinoid
syndrome. Half of the cases produce gastrin.
 Ampullary carcinoids frequently present with jaundice. Approximately 25% of patients
 with periampullary carcinoids have von Recklinghausen’s disease.
 The risk of metastasis increases with increased size of the primary tumour.
 The treatment of duodenal and ampullary carcinoid tumour is by resection.
Small Intestine
 Ileum is the most common site for carcinoid tumours, accounting for almost 30 % of
all carcinoids.
 Carcinoids form approximately 28% to 38% of all small bowel cancers.
 Average age of presentation is 62 to 65 years. Small bowel carcinoids are multifocal
in 25% of cases.
 They present as vague abdominal pain, intermittent intestinal obstruction, infarction
of small bowel due to superior mesenteric artery occlusion.
 Overt carcinoid syndrome is uncommon as an initial presentation.
 Carcinoid syndrome usually indicates hepatic or retroperitoneal metastasis and signifies an
unfavourable outcome.
 If found early, primary tumour and the associated lymphatics should be resected surgically.
Appendix
 It is a less common site of GI carcinoids.
 The proportion of appendiceal tumours that are carcinoids is high.
 Majority are less than 1 cm in size.
 Frequency of distant metastasis increases with the size of the tumour.
 They cause symptoms early due to appendiceal luminal obstruction with consequent
appendicitis. Have a favourable prognosis.
 Simple appendicectomy is indicated for tumours less than 1 cm size and a right
hemicolectomy for tumours more than 2 cm. size.
Colon
 Colonic carcinoids account for less than 10% of carcinoid tumours.
 They occur more commonly on right side, about two thirds involving ascending or
proximal transverse colon.
 They tend to present as larger lesions than most oher carcinoids, averaging around 5 cm.
 Symptoms are usually due to bulky advanced lesions.
 Advanced lesions may cause pain or colonic obstruction.
 Surgical resection and lymph node clearance should be performed. No adjuvant treatment
is effective. Five year survival is worst
Rectum
 Constitutes 19 % of all carcinoids.
 More than 80 % are still localized at the time of diagnosis.
 Appear as local nodules. Tumours may produce symptoms as bleeding, pain, or decreased
stool caliber, weight loss, changing bowel habits, may be detected incidentally.
 Carcinoid syndrome is distinctly uncommon due to rectal lesions.
 Rectal carcinoids have been treated endoscopically, by local excision, (lesions less
than 2 cm) and by radical excision with either a low anterior resection or APR (lesions
larger than 2 cm).

Carcinoid Syndrome

Typical Carcinoid Syndrome:


Most common clinical pattern, caused by metastatic midgut carcinoids. Serotonin is released
into circulation. It is then converted into 5- hydroxyindoleacetic acid (5-HIAA),which is
excreted in urine.
Patients with typical carcinoid syndrome have elevated plasma and platelet serotonin and
increased 5-HIAA in urine.
Clinically it is characterized by flushing, diarrhea, non specific abdominal pain, bronchospasm,
pellagra like skin lesions and progressive right heart failure.

Atypical Carcinoid Syndrome:


Most often associated with foregut carcinoid tumours. It presents with a different pattern of
flushing, it occurs again in head and neck region but it appears patchier. It also shows central
clearing and increased pruritus.
Asthma and/ or signs of peptic ulceration are occasional presentations due to increased
systemic levels of histamine. Patients have high plasma levels of 5-HTP and normal levels of
serotonin.
Urinary levels of 5-HIAA are typically normal to slightly elevated.

Diagnosis:
Based on typical clinical history,
24 hour urinary 5-HIAA levels.

Carcinoid Crisis:
 Acute exacerbation of carcinoid syndrome may occur in a number of situations but most
commonly in the setting of surgical or anaesthetic stresses.
 Patients with known carcinoid tumours should receive somatostatin analogs before surgery
to suppress mediator release.
 Symptoms like hyperglycemia, hypertension, bronchospasm are treated by routine medical
means.

Imaging Studies
 Visualization of primary carcinoid tumour is more difficult than identification of
metastatic disease.
 CT scan and MRI are non specific and often non –localizing. They are useful when lesions
are more than 1 cm in size. May be used to identify metastatic disease.
 Somatostatin Receptor Scanning using radiolabelled somatostatin analogs (Indium¹¹¹
labeled pentetreotide, I123 labeled tyrosine) is helpful in localizing primary occult
tumours and in staging of patients with known metastatic disease.

Treatment of Metastatic Disease / Carcinoid Syndrome: Majority of such patients are not
the candidates for curative treatment. The focus of
therapy is palliation of symptoms.

1. Surgery: For hepatic metastasis: Resection , OLT. Cytoreductive therapy. Hepatic artery
embolization and chemoembolization.
2. Immunotherapy: ? Interferon ( controversial).
3. Chemotherapy : Marginal benefit.
4. Pharmacologic Therapy: Mainstay of palliative therapy.
 Octreotide, lantreotide(long acting)---- Somatostatin Analogs.
 Methysergide, cyproheptadine, ketanserin, ondansetron--- serotonin receptor blockers.
 Parachlorophenylalanine--- inhibitor of serotonin synthesis.
5. Radiotherapy: For palliation of metastasis to bone and central nervous system.

b. Raynaud’s phenomenon.

Definition: repetitive episodes of biphasic colour change (at least 2 of pallor, cyanosis,
erythema), in either cold or normal environment.
Clinical features or Raynaud’s

 Primarily affects fingers Initial ischaemia


 Can affect toes, thumbs, Pallor
nipples, nose, earlobes
 Episodes precipitated by
cold exposure and
emotional stress Cyanotic phase
 Episodes accompanied by Blue
pain +/- numbness
 Pulses present
 Necrosis / tissue damage
suggestive of secondary Hyperaemic phase
cause Red / purple

Pathogenesis:
Causes:
 Primary (~10-15% of healthy population, female predominance)
 Secondary
 Drugs e.g. Beta blockers
 Connective tissue disorders e.g. systemic sclerosis
 Eating disorders
 Haematological e.g. cold agglutinins
 Vascular occlusion e.g. vasculitis, thoracic outlet obstruction, Buerger’s disease
 Occupation e.g. vibrating tool use
 Others e.g. hypothyroidism, carpal tunnel syndrome.
Diagnosis:
 History
 Severity, age of onset, gender, symptoms of CTD etc
 Clinical examination
 Radial pulses
 Skin changes
 Nailfold changes
 Joint disease
 Carpal Tunnel Syndrome
 Laboratory investigations
 FBC, U&E, LFT, CRP, TSH
 Autoimmune profile
 Nailfold capillaroscopy
 Infrared thermography
Treatment:
General measures:

 Maintenance of core temperature

 Avoidance of cold exposure

 Cessation of vasoconstrictive Rx e.g. B blockers

 Gloves (heated)

 Smoking cessation
Definitive treatment:

Promoting vasodilation Preventing vasoconstriction Novel treatments


 Calcium channel blockers  ACEi and ARBs  Rho kinase inhibitors
o Dihydropyridine o e.g. losartan o Responsible for cold-induced
 Nifedipine better than o May be better in primary RP expression of alpha-2
amlodipine  Alpha adrenoceptor blockade adrenoceptors
 Nitrates o e.g. prazosin  Statins
o Transdermal or oral  SSRIs o In part due to Rho kinase
 Prostaglandins o e.g. fluoxetine inhibition
 IV  Endothelin receptor antagonists  Antiplatelet treatments?
 Phosphodiesterase V inhibitors o e.g. bosentan o Current trial

c. Ranula.
Introduction: Ranula refers to a collection of extraglandular and extraductal saliva in the
floor of the mouth originating from the sublingual salivary gland. It may rarely originate
from injury to the submandibular gland (SMG) duct. It is a pseudocyst, as it does not
contain an epithelial lining.
The term ranula originates from the Latin word for frog (rana) as the cyst is said to look
like the underbelly of a frog.
A plunging ranula extends into the submandibular triangle of the neck through a defect in
the mylohyoid muscle, or less commonly, by passing behind the posterior edge of the
muscle.
Clinical features:
 A ranula usually presents as a translucent blue, dome-shaped, fluctuant swelling in
the tissues of the floor of the mouth.
 If the lesion is deeper, then there is a greater thickness of tissue separating from the
oral cavity and the blue translucent appearance may not be a feature.
 A ranula can develop into a large lesion many centimeters in diameter, with
resultant elevation of the tongue and possibly interfering with swallowing
(dysphagia).
 The swelling is not fixed, may not show blanching and is non-painful unless it
becomes secondarily infected.
 The usual location is usually is lateral to the midline, which may be used to help
distinguish it from a midline dermoid cyst.
 A cervical ranula presents as a swelling in the neck, with or without a swelling in
the mouth. In common with other mucoceles, ranulas may rupture and then cause
recurrent swelling.
 Ranulas may be asymptomatic, although they can fluctuate rapidly in size,
shrinking and swelling, making them hard to detect.

Diagnostic criteria:

 Mostly seen in young children and adolescents, Both of sexes are equally affected.
Swelling in floor of mouth, which may be painful. Mostly unilateral, on one side of
frenulum.
 Shape is spherical
 Size varies from 1 - 5 cm in diameter
 Color is pale blue with characteristics semi transparent appearance.
 Surface is smooth and mucous membrane is mobile over the swelling.
 Tenderness is absent
 Fluctuation test is positive
 Transillumination test is positive
 Cervical lymph nodes are not enlarged.
 May or may not have prolongation in the neck.

Treatment:

 Treatment of ranulas usually involves removal of the sublingual gland. Surgery may not be
required if the ranula is small and asymptomatic.
 Marsupialization may sometimes be used, where the intra-oral lesion is opened to the oral
cavity with the aim of allowing the sublingual gland to re-establish connection with the oral
cavity, but it is often unsuccessful.
 Excision of sublingual sailavary gland is often needed.

d. Abdominal compartment syndrome.


Introduction:
Abdominal compartment syndrome occurs when the abdomen becomes subject to
increased pressure. Specific cause of abdominal compartment syndrome is not known,
although some causes can be sepsis and severe abdominal trauma. Increasing pressure
reduces blood flow to abdominal organs and impairs pulmonary, cardiovascular, renal,
and gastro-intestinal (GI) function, causing multiple organ dysfunction syndrome and
death.
Definition:

 Intra-abdominal Pressure (IAP): is the pressure within the abdominal cavity. Normal
IAP in a well child is 0 mmHg and in a child on positive pressure ventilation is 1 - 8
mmHg.
 Intra-abdominal hypertension (IAH): is defined as an IAP greater than 10mmHg.
 Abdominal compartment syndrome (ACS): is defined as an IAP >20mmHg and the
onset of new or worsening organ failure directly attributed to elevated IAP.
Types:
 Primary ACS is essentially organ dysfunction and IAH in the presence of direct injury
to the abdominal contents. The examples are trauma, peritonitis, ileus, and hemorrhage
etc.
 Secondary consists of elevated pressure and organ dysfunction caused by third space
edema and resuscitation. The examples are resuscitation of hemorrhagic shock patients,
burns etc.
 Recurrent ACS in which the patient has recovered from the ACS once but because of
secondary insults the cycle begins again. This verity is associated with very high
mortality rate.
Causes:
Primary and secondary conditions that cause intra-abdominal hypertension and
abdominal compartment syndrome:
Primary Secondary
 Blunt/penetrating trauma  Severe intra-abdominal infection
 Liver transplantation  Large-volume fluid replacement
 Ruptured abdominal aortic aneurysm  Ascites
Postoperative bleeding  Pancreatitis
 Retroperitoneal hemorrhage  Ileus
 Mechanical intestinal obstruction  Sepsis
 Postoperative closure of the abdomen  Major burns
under tension  Continuous ambulatory peritoneal
 Bleeding pelvic fractures dialysis Morbid obesity
 Pregnancy

Effect on different organ:

Cardiovascular
A rise in the intra-abdominal pressure leads to a fall in cardiac output, due mainly to
compression of the inferior vena cava and reduction in venous return to the heart. Cardiac
output is reduced despite apparent rises in central venous pressure, pulmonary artery occlusion
pressure and systemic vascular resistance.

Respiratory
Raised intra-abdominal pressure will effectively splint the diaphragm and lead to a rise in peak
airway pressure and intra-thoracic pressure and subsequently a reduced venous return to the
heart. The increase in airway pressures may also exacerbate barotrauma and contribute to the
development of acute respiratory distress syndrome.

Renal
An acute increase in intra-abdominal pressure leads to oliguria and anuria probably due to
compression of the renal vein and renal parenchyma. Renal blood flow, glomerular filtration
are decreased with a corresponding increase in renal vascular resistance.

Cerebral
The rise in intra-abdominal pressure, intrathoracic pressure leads to a rise in central venous
pressure which prevents adequate venous drainage from the brain, leading to a rise in
intracranial pressure and worsening of intracerebral oedema.

Diagnosis: Measuring the pressure

– Insert a Foley catheter and clamp the tube distal to the sample port
– Instill 5-1000mL of saline into the bladder so as to leave a continuous column of fluid
from the bladder to the sample port on the Foley
– Insert a 18g catheter into the sample port and connect to a CVP transducer
– Level the transducer at the symphysis pubis
Diagnostic features:
o Distended abdomen
o IAP > 20 mm of Hg
o Elevated peak airway pressure
o Massive I.V. fluids requirements
o Oliguria to anuria not responding to volume repletion
o Decreased cardiac output
o Hypoxemia refractory to increase FIO2 and PEEP
o Hypercarbia
o Hypercapnia
o Wide pulse pressure
o Acidosis
 CT and Other Imaging Studies:
Abdominal CT scanning can reveal many subtle findings. Pickhardt et al found the following
in patients with abdominal compartment syndrome:
 Round-belly sign: Abdominal distention with an increased ratio of anteroposterior-to-
transverse abdominal diameter (ratio >0.80)
 Collapse of the vena cava
 Bowel wall thickening with enhancement
 Bilateral inguinal herniation
Plain abdominal radiographic studies are often useless in identifying abdominal compartment
syndrome, although they may show evidence of free air or bowel obstruction. Abdominal
ultrasonography may reveal an aortic aneurysm, particularly with large aneurysms, but bowel
gas or obesity makes performing the study difficult.
The following lab studies may be indicated:
 Comprehensive metabolic panel (CMP)
 Complete blood cell count (CBC)
 Amylase and lipase assessment
 Prothrombin time (PT), activated partial thromboplastin time (aPTT) if the patient is
heparinized
 Cardiac marker assays
 Urinalysis and urine drug screen
 Measurement of serum lactate levels (at many institutions, the sample must be kept on ice)
 Arterial blood gas (ABG): This is a quick way to measure the pH, lactate, and base deficit
Treatment:
If abdominal compartment syndrome is suspected, the focus of prehospital care is to
immediately transport the patient to the emergency department. Remove any constricting
garments. Do not place anything on the patient's abdomen (eg, external defibrillators, bundles
of blankets, oxygen tanks).
Avoid overly aggressive fluid resuscitation, especially in extremity injuries. The over-
resuscitated patient is much more likely to develop abdominal compartment syndrome, and
often the prehospital setting is where this begins.
In the emergency department, the first priority of the ED physician is to consider the diagnosis
in any patient with the appropriate mechanisms of injury or pathology. Abdominal
compartment syndrome will be missed unless it is in the differential diagnosis.
Therapy should include fluid resuscitation and transfusion if needed, as well as surgical
consultation.
A comprehensive, evidence-based approach to the management of abdominal compartment
syndrome that includes early use of an open abdomen has been shown to reduce mortality.

The first step in the evaluation of an increased IAP, especially in the presence of agitation and
restlessness, is to sedate and, if necessary, chemically paralyze the patient. If the bladder
pressures are still high and/or systemic manifestations of IAH (as described above) are evident,
the appropriate treatment, in most instances, is abdominal decompression by laparotomy.

If the abdomen is already "open," this may involve the opening of the temporary closure
(mesh, plastic, or towel clips, etc.); evacuation of the abdomen of fluid and blood; removal of
abdominal packs (with repacking, if necessary); and enlargement of the abdominal space to
accommodate the edematous and swollen contents. Some centers have started evacuation of
ascitic fluid as a temporary measure of abdominal decompression after secondary ACS,
especially in burn patients. Evacuation of hemoperitoneum by ultrasound (US)-guided
paracentesis may be employed with success in patients with nonoperative management of
hepatic and splenic injuries. If the abdomen was previously closed by fascial sutures, there was
no previous laparotomy, or bedside decompression is not efficacious or possible, a formal
operating room celiotomy should be performed.

At celiotomy, the abdomen is decompressed, abdominal fluid and/or blood evacuated, and
hemostasis obtained or ensured. The results of abdominal decompression are usually rather
dramatic, with a decreased inspiratory pressure, improved gas exchange, lessened systemic
vascular resistance, and increased cardiac index and oxygen delivery. The gut appearance will
improve, and, if measured, oxygenation of the gut can be shown to have improved. A brisk
increase in urinary output occurs.

Once abdominal decompression is achieved, it is the usual practice to leave the abdominal
fascia and skin "open" with some type of foreign material at the skin level to prevent
evisceration. These materials include various absorbable or nonabsorbable types of mesh, an
artificial burr device, sterilized saline bags with or without a zipper (the Bogota bag), dressings
with moist gauze or Vi-drapes, "vacuum-pack" etc. All have in common the goals of
preventing evisceration, allowing enlargement of the abdominal cavity, keeping the IAP low,
and preventing reoccurrence of IAH and ACS.
Pharmacologic therapy is less effective than mechanical drainage. Pressors have a role but may
not be equally effective in treating abdominal compartment syndrome.
Inpatient care in acute abdominal compartment syndrome is directed by critical care physicians
and surgeons. If an ICU patient experiences decompensation, abdominal compartment
syndrome should be reexamined as a potential cause.
IAH may be an ongoing process in any patient with pathology producing intra-abdominal fluid
loss. Repeat or continuous IAP measurement is indicated. The abdomen should be clear of any
heavy objects.
e. Desmoids tumour
Introduction:
Desmoid tumors, also known as aggressive fibromatoses, are soft-tissue tumors that
arise from connective tissue and typically present as a single, slowly growing mass.

Aetiopathogenesis:
 Desmoid tumors are rare; they account for about 0.03 percent of all neoplasms and less
than 3 percent of all soft tissue tumors.
 Individuals between the ages of 15 and 60 are most commonly affected; desmoids are
rare in the young and in the elderly.
 They are slightly more common in women than in men and there is no significant racial
or ethnic predilection.
Most desmoids are sporadic tumors, but some occur in the setting of Gardner
syndrome, a variant of familial adenomatous polyposis.
 Clonal proliferation of fibroblasts with an abundant collagen matrix
 Occur in 10-20% of patients with Familial Polyposis:
Site:
o Mesentery
o Abdominal Wall/Trunk
o Extremities
o Multifocal
 Sporadic tumors are rare
Site:
o Extremity
o Abdominal Wall/Trunk
o Mesentery
Classification:
Desmoid tumors may be classified as extra-abdominal, abdominal wall, or intra-abdominal (the
last is more common in patients with FAP). It is thought that the lesions may develop in
relation to estrogen levels or trauma/operations.
A 3' APC mutation is the most significant risk factor for intra-abdominal desmoid development
amongst FAP patients. FAP patients presenting with an abdominal wall desmoid pre-
operatively are at an increased risk of developing an intra-abdominal desmoid post-operatively.
Desmoid tumours of the breast are rare. Although benign, they can mimic breast cancer on
physical examination, mammography and breast ultrasound and can also be locally invasive.
Even though they occur sporadically, they can also be seen as a part of Gardner's syndrome. A
high index of suspicion and a thorough triple examination protocol is necessary to detect rare
lesions like a desmoid tumour which can masquerade as breast carcinoma. Desmoid tumour of
the breast may present a difficulty in the diagnosis especially where imaging studies are not
conclusive and suggest a more ominous diagnosis.
Treatment:
 Treatment may consist of watching and waiting, complete surgical removal, radiation
therapy, antiestrogens, NSAIDs,chemotherapy or microwave ablation.
 A biopsy is always indicated as the definitive method to determine nature of the tumour.
 Management of these lesions is complex, the main problem being the high rates of
recurrence in FAP associated disease.
 Conversely, for intra-abdominal fibromatosis without evidence of FAP, although
extensive surgery may still be required for local symptoms, the risk of recurrence appears
to be lower.
 Wide surgical resection with clear margins is the most widely practiced technique with
radiation, chemotherapy, or hormonal therapy being used to reduce the risk of recurrence.
 Current experimental studies are being done with Imatinib and sorafenib for treatment of
desmoid tumors, and show promising success rates.
Group – C
3. Write short notes on (any three ) 3x5

a. Clinical features of Hirsprung’s disease.


b. Cleft lip management in children.
c. Tetanus prophylaxis.
d. Diagnostic use of ultrasound.
e. Empyema thoracis

Answer.
a. Clinical features of Hirsprung’s disease. See the question 2.e of Group – B of
supplementary paper – II of 2009.
b. Cleft lip management in children. See the question 3.a of Group – C of paper – II
of 2013.
c. Tetanus prophylaxis.

Tetanus is a serious disease characterized by muscle spasm and rigidity. The mortality rate
is approximately 20% and is due to spasm of the muscles of respiration. Tetanus is an
illness preventable through primary immunization and regular booster shots.

The Emergency Department patient encounter provides an ideal opportunity to screen for
adequate tetanus immunization and to provide it, when necessary.

Primary immunization involves a series of four toxoid injections for preschool children or
three toxoid injections if started at age 7 or older. Following primary immunization,
children receive a booster shot at age 5 and additional boosters every 10 years subsequent
to that.
Patients seen in the Emergency Department with clean, minor wounds are considered
adequately immunized if they have received primary immunization and have had a booster
within the past 10 years. If a wound is "dirty" (which includes wounds contaminated with
saliva, feces or dirt, and burn injuries) then a booster within the past 5 years is necessary to
ensure immunization.

If the patient has not received primary immunization, (or if the patient is unsure) then
passive immunity with tetanus immune globulin (T.I.G.) is provided. At the same time, but
with a different injection site, tetanus and diphtheria toxoid should be given. This initiates
primary immunization but adequate follow-up should be arranged to ensure completion of
the series. Note that the diphtheria toxoid is added to ensure adequate immunity to
diphtheria in the population.

Patients will occasionally present stating they have an allergy to the toxoid. Adverse
reactions such as local pain, erythemia, fever, malaise or rash are common but should not
preclude further immunization. A true anaphylactic or serious neurologic reaction to the
toxoid are the only contraindications to further immunization with the tetanus and
diphtheria toxoid.

If a patient has had a true serious reaction in the past, they should receive a T.I.G. in the
Emergency Department and then follow-up with an allergist to assess immunization status.
The table below summarizes the CDC guidelines for tetanus prophylaxis.

Tetanus Immunization:

For clean, minor wounds:

Give patient Tetanus Give patient Tetanus Immune


Diphtheria Toxoid? Globulin?
Unknown or less than 3 doses of absorbed
Yes No
Tetanus Toxoid
Greater than 3 doses of of absorbed No (unless >10 years since last
No
Tetanus Toxoid booster)

For all other wounds:

Give patient Tetanus Give patient Tetanus


Diphtheria Toxoid? Immune Globulin?
Unknown or less than 3 doses of
Yes Yes
absorbed Tetanus Toxoid
Greater than 3 doses of of absorbed No (unless >5 years since
No
Tetanus Toxoid last booster)
d. Diagnostic use of ultrasound.
 Ultrasound imaging is useful in diagnosing a wide variety of conditions affecting
the organs and soft tissues of the body.
v.i.z, heart and blood vessels, liver, gallbladder, spleen,pancreas, kidneys, bladder,
uterus, ovaries, eyes, thyroid, and testicles.
 Ultrasounds do have some diagnostic limitations, however; sound waves do not
transmit well through dense bone or parts of the body that may hold air or gas, such
as the bowel.
 View the uterus and ovaries of a pregnant woman and assess her fetus
 Evaluate flow in blood vessels
 Check a thyroid gland
 Diagnose some cancers
 Reveal genital and prostate abnormalities

 Breast ultrasound is a procedure that may be used to determine whether a lump is a


cyst (sac containing fluid) or a solid mass which might be cancer. If the lump is
found to be a cyst, fluid is typically withdrawn from it using a needle and syringe (a
process called aspiration). If clear fluid is removed and the mass completely
disappears, no further treatment or evaluation is needed.

 Ultrasound can also be used to precisely locate the position of a known tumor to
help guide the doctor during a biopsy or aspiration procedure. Ultrasound helps
confirm correct needle placement.

 Ultrasound testing works by transmitting high-frequency sound waves, inaudible to


the human ear, through the breast. The sound waves bounce off surfaces in
the breast (tissue, air, fluid) and these "echoes" are recorded and transformed into
video or photographic images.
 Use during medical procedures: Ultrasound imaging can help doctors during
procedures such as needle biopsies.
 Diagnostic ultrasound is a safe procedure that uses low-power sound waves. There
are no known risks.

e. Empyema thoracis. See the question 3.c of Group – C of supplementary paper – II


of 2013.

Group – D
4. Answer briefly on (any three): 3x5

a. Epulis.
b. Wax bath.
c. Extradural haematoma.
d. Care of a paraplegic patient.
e. Axial flap.
Answer
a. Epulis. See the 4.b of Group – D of paper – II of 2011.

b. Wax bath. See the question 4.b of Group – D of paper – II of 2013.

c. Extradural haematoma. See the question 2.c of Group – B of supplementary paper –


II of 2012.

d. Care of a paraplegic patient.


 Choose a well-functioning and high-quality wheelchair. A wheelchair is an essential item
for a paraplegic, because it will enable him to get around more easily as well as help him
regain some of his independence. Involve the paraplegic in the selection process so he
can try a few wheelchairs and choose the one that works best for him,
 Move the paraplegic patient so he does not develop sores from inactivity. Sores can
develop on the back and on the back of the legs, due to sitting or being in a prone position
most of the time. Help the patient roll onto his side or stomach for longer periods of time
to prevent sores from developing,
 Inspect the paraplegic patient's catheter on a regular basis to ensure that it is sanitary.
Urinary tract infections can be serious for paraplegic patients, notes Salter, and taking
proper care of the catheter can help prevent them.

 A physical therapist teaches the patient the exercises to help improve movement and
strength. Physical therapists help you learn ways to stay active. They will teach you how
to use a wheelchair and how to move from a bed to a chair or toilet.

 An occupational therapist teaches the patient the skills to help with your daily activities,
such as getting dressed or bathing. Occupational therapists also teach work-related skills.

 Bowel and bladder programs help to manage when the patient urinate or has a bowel
movement.

e. Axial flap.
Axial flaps are supplied by a named artery and vein. This allows for a larger area to be freed
from surrounding and underlying tissue, leaving only a small pedicle containing the vessels.
Mathes and Nahai classification

 One vascular pedicle (eg, tensor fascia lata)


 Dominant pedicle(s) and minor pedicle(s) (eg, gracilis)
 Two dominant pedicles (eg, gluteus maximus)
 Segmental vascular pedicles (eg, sartorius)
 One dominant pedicle and secondary segmental pedicles (eg, latissimus dorsi)

Advantages: Disadvantages Principles

 Have defined cutaneous blood supply  Don’t reach distal  Dissect deep to panniculus or
 Radiation/infection resistant extremities. subcutis
 Durable, full thickness skin  Large donor site  Trans-illuminate
 Maintain hair growth closure  Bridging incision
 “Instant” closure  Variable cosmesis  Tacking sutures
 Closed suction drainage

You might also like